Site Loader

Содержание

Какие есть способы соединения приемников электрической энергии

Какие есть способы соединения приемников электрической энергии

Приемник электрической энергии — устройство, в котором происходит преобразование электрической энергии в другой вид энергии для ее использования.

При одновременном включении нескольких приемников электроэнергии в одну и ту же сеть, эти приемники можно легко рассматривать просто как элементы единой цепи, каждый из которых обладает собственным сопротивлением.

В ряде случаев такой подход оказывается вполне приемлемым: лампы накаливания, электрические обогреватели и т. п. — можно воспринимать как резисторы. То есть приборы можно заменить на их сопротивления, и легко произвести расчет параметров цепи.

Способ соединения приемников электроэнергии может быть одним из следующих: последовательный, параллельный или смешанный тип соединения.

Последовательное соединение

Когда несколько приемников (резисторов) соединяются в последовательную цепь, то есть второй вывод первого присоединяется к первому выводу второго, второй вывод второго соединяется с первым выводом третьего, второй вывод третьего с первым выводом четвертого и т.

д., то при подключении такой цепи к источнику питания, через все элементы цепи потечет ток I одной и той же величины. Данную мысль поясняет приведенный рисунок.

Заменив приборы на их сопротивления, рисунок преобразуем в схему, тогда сопротивления с R1 по R4, соединенные последовательно, примут каждый на себя определенные напряжения, которые в сумме дадут значение ЭДС на зажимах источника питания. Для простоты здесь и далее изобразим источник в виде гальванического элемента.

Выразив падения напряжений через ток и через сопротивления, получим выражение для эквивалентного сопротивления последовательной цепи приемников: общее сопротивление последовательного соединения резисторов всегда равно алгебраической сумме всех сопротивлений, составляющих эту цепь. А поскольку напряжения на каждом из участков цепи можно найти из закона Ома (U = I*R, U1 = I*R1, U2 = I*R2 и т. д.) и E = U, то для нашей схемы получаем:

Напряжение на клеммах источника питания равно сумме падений напряжений на каждом из соединенных последовательно приемников, составляющих цепь.

Так как ток через всю цепь течет одного и того же значения, то справедливым будет утверждение, что напряжения на последовательно соединенных приемниках (резисторах) соотносятся между собой пропорционально сопротивлениям. И чем выше будет сопротивление, тем выше окажется и напряжение, приложенное к приемнику.

Для последовательного соединения резисторов в количестве n штук, обладающих одинаковыми сопротивлениями Rk, эквивалентное общее сопротивление цепи целиком будет в n раз больше каждого из этих сопротивлений: R = n*Rk. Соответственно и напряжения, приложенные к каждому из резисторов цепи будут между собой равны, и окажутся в n раз меньше напряжения, приложенного ко всей цепи: Uk = U/n.

Для последовательного соединения приемников электроэнергии характерны следующие свойства: если изменить сопротивление одного из приемников цепи, то напряжения на остальных приемниках цепи при этом изменятся; при обрыве одного из приемников ток прекратится во всей цепи, во всех остальных приемниках.

В силу этих особенностей последовательное соединение встречается редко, и используют его лишь там, где напряжение сети выше номинального напряжения приемников, в отсутствие альтернатив.

К примеру напряжением 220 вольт можно запитать две последовательно соединенные лампы равной мощности, каждая из которых рассчитана на напряжение 110 вольт. Ежели данные лампы при одинаковом номинальном напряжении питания будут обладать различной номинальной мощностью, то одна из них будет перегружена и скорее всего мгновенно перегорит.

Параллельное соединение

Параллельное соединение приемников предполагает включение каждого из них между парой точек электрической цепи с тем, чтобы они образовывали параллельные ветви, каждая из которых питается напряжением источника. Для наглядности опять заменим приемники их электрическими сопротивлениями, чтобы получить схему, по которой удобно вести расчет параметров.

Как уже было сказано, в случае параллельного соединения каждый из резисторов испытывает действие одного и того же напряжения.

И в соответствии с законом Ома имеем: I1=U/R1, I2=U/R2, I3=U/R3.

Здесь I – ток источника. Первый закон Кирхгофа для данной цепи позволяет записать выражение для тока в неразветвленной ее части: I = I1+I2+I3.

Отсюда общее сопротивление для параллельного соединения между собой элементов цепи можно найти из формулы:

Величина обратная сопротивлению называется проводимостью G, и формулу для проводимости цепи, состоящей из нескольких параллельно соединенных элементов, также можно записать: G = G1 + G2 + G3. Проводимость цепи в случае параллельного соединения образующих ее резисторов равна алгебраической сумме проводимостей этих резисторов. Следовательно, при добавлении в цепь параллельных приемников (резисторов) суммарное сопротивление цепи уменьшится, а суммарная проводимость соответственно возрастет.

Токи в цепи состоящей из параллельно соединенных приемников, распределяются между ними прямо пропорционально их проводимостям, то есть обратно пропорционально их сопротивлениям. Здесь можно привести аналогию из гидравлики, где поток воды распределяется по трубам в соответствии с их сечениями, тогда большее сечение аналогично меньшему сопротивлению, то есть большей проводимости.

Если цепь состоит из нескольких (n) одинаковых резисторов, соединенных параллельно, то общее сопротивление цепи будет ниже в n раз, чем сопротивление одного из резисторов, а ток через каждый из резисторов будет меньше в n раз, чем общий ток: R = R1/n; I1 = I/n.

Цепь, состоящая из параллельно соединенных приемников, подключенная к источнику питания, отличается тем, что каждый из приемников находится под напряжением источника питания.

Для идеального источника электроэнергии справедливо утверждение: при подключении или отключении параллельно источнику резисторов, токи в остальных подключенных резисторах не изменятся, то есть при выходе из строя одного или нескольких приемников параллельной цепи, остальные будут продолжать работать в прежнем режиме.

В силу данных особенностей параллельное соединение обладает значительным преимуществом перед последовательным, и по этой причине именно соединение параллельное наиболее распространено в электрических сетях. Например, все электроприборы в наших домах предназначены для параллельного подключения к бытовой сети, и если отключить один, то остальным это ничуть не навредит.

Сравнение последовательных и параллельных цепей

Смешанное соединение

Под смешанным соединением приемников понимают такое их соединение, когда часть или несколько из них соединены между собой последовательно, а другая часть или несколько — параллельно. При этом вся цепь может быть образована из разных соединений таких частей между собой. Для примера рассмотрим схему:

Три последовательно соединенных резистора подключены к источнику питания, параллельно одному из них подключены еще два, а третий — параллельно всей цепи. Для нахождения полного сопротивления цепи идут путем последовательных преобразований: сложную цепь последовательно приводят к простому виду, последовательно вычисляя сопротивление каждого звена, и так находят общее эквивалентное сопротивление.

Для нашего примера. Сначала находят общее сопротивление двух резисторов R4 и R5, соединенных последовательно, затем сопротивление параллельного соединения их с R2, потом прибавляют к полученному значению R1 и R3, и после — вычисляют значение сопротивления всей цепи, включая параллельную ветвь R6.

Различные способы соединения приемников электроэнергии применяют на практике для различных целей, чтобы решать конкретные поставленные задачи. Например, смешанное соединение можно встретить в схемах плавного заряда электролитических конденсаторов в мощных блоках питания, где нагрузка (конденсаторы после диодного моста) сначала получает питание последовательно через резистор, затем резистор шунтируется контактами реле, и нагрузка оказывается подключенной к диодному мосту параллельно.

Ранее ЭлектроВести писали, что львовянка Оксана Денис подключила свою квартиру к солнечным панелям и ветроэнергетическим установкам.

По материалам: electrik.info.

Параллельное соединение резисторов таблица.

Параллельное соединение сопротивлений в электрической цепи. Параллельное соединение конденсаторов и катушек

В каждой электрической схеме присутствует резистор, имеющий сопротивление электрическому току. Резисторы бывают двух типов: постоянные и переменные. Во время разработки любой электрической схемы и ремонта электронных изделий часто приходится применять резистор, обладающий необходимым номиналом.

Несмотря на то что для резисторов предусмотрены различные номиналы

, может случиться так, что не будет возможности найти необходимый или же вообще ни один элемент не сможет обеспечить требуемый показатель.

Решением этой проблемы может стать применение последовательного и параллельного соединения. Ознакомившись с этой статьей, вы узнаете об особенностях выполнения расчета и подбора различных номиналов сопротивлений.

Часто при изготовлении какого-либо устройства используют резисторы, которые соединяются в соответствии с последовательной схемой. Эффект от применения такого варианта сборки сводится к увеличению общего сопротивления цепи. Для данного варианта соединения элементов создаваемое ими сопротивление рассчитывается как сумма номиналов. Если же сборка деталей выполняется по параллельной схеме, то здесь потребуется рассчитать сопротивление , используя нижеописанные формулы.

К схеме параллельного соединения прибегают в ситуации, когда стоит задача по снижению суммарного сопротивления, а, помимо этого, увеличения мощности для группы элементов, подключенных по параллельной схеме, которое должно быть больше, чем при их отдельном подключении.

Расчет сопротивления

В случае подключения деталей друг с другом, с применением параллельной схемы для расчета суммарного сопротивления, будет использоваться следующая формула:

R(общ)=1/(1/R1+1/R2+1/R3+1/Rn).

  • R1- R3 и Rn – резисторы, подсоединенные по параллельной схеме.

Причем, если цепь создается на основе только двух элементов, то для определения суммарного номинального сопротивления следует использовать такую формулу:

R(общ)=R1*R2/R1+R2.

  • R(общ) – суммарное сопротивление;
  • R1 и R2 – резисторы, подсоединенные по параллельной схеме.

Универсальная схема расчета

Применительно к радиотехнике следует уделить внимание одному важному правилу: если подключаемые друг к другу элементы по параллельной схеме имеют одинаковый показатель , то для расчета суммарного номинала необходимо общее значение разделить на число подключенных узлов:

  • R(общ) – суммарное значение сопротивления;
  • R – номинал резистора, подсоединенного по параллельной схеме;
  • n – число подключенных узлов.

Особое внимание следует обратить на то, что конечный показатель сопротивления в случае использования параллельной схемы подключения обязательно будет меньше по сравнению с номиналом любого элемента, подключаемого в цепь.

Пример расчета

Для большей наглядности можно рассмотреть следующий пример: допустим, у нас есть три резистора, чьи номиналы соответственно равны 100, 150 и 30 Ом. Если воспользоваться первой формулой для определения общего номинала, то получим следующее:

R(общ)=1/(1/100+1/150+1/30)=1/(0,01+0,007+0,03)=1/0,047=21,28Ом.

Если выполнить несложные расчеты, то можно получить следующее: для цепи, включающей в себя три детали, где наименьший показатель сопротивления составляет 30 Ом, результирующее значение номинала будет равно 21,28 Ом. Этот показатель будет меньше минимального значения номинала в цепи практически на 30%.

Важные нюансы

Обычно для резисторов параллельное соединение применяется тогда, когда стоит задача по созданию сопротивления большей мощности. Для ее решения потребуются резисторы, которые должны иметь равные показатели сопротивления и мощности. При таком варианте определить общую мощность можно следующим образом : мощность одного элемента необходимо перемножить с суммарным числом всех резисторов, из которых состоит цепь, подсоединенных друг с другом в соответствии с параллельной схемой.

Скажем, если нами будут использоваться пять резисторов, чей номинал составляет 100 Ом, а мощность каждого равна 1 Вт, которые присоединены друг к другу в соответствии с параллельной схемой, то суммарный показатель сопротивления будет равен 20 Ом, а мощность составит 5 Вт.

Если взять те же резисторы, но подсоединить их в соответствии с последовательной схемой, то конечная мощность составит 5 Вт, а суммарный номинал будет равен 500 Ом.

Заключение

Параллельная схема подключения резисторов очень востребована по той причине, что часто возникает задача по созданию такого номинала, которого невозможно добиться при помощи простого параллельного соединения. При этом процедура расчета этого параметра отличается достаточной сложностью , где необходимо учитывать разные параметры.

Здесь важная роль отводится не только количеству подключаемых элементов, но и рабочим параметрам резисторов — прежде всего, сопротивлению и мощности. Если один из подключаемых элементов будет иметь неподходящий показатель, то это не позволит эффективно решить задачу по созданию требуемого номинала в цепи.

На практике нередко встречается задача нахождения сопротивления проводников и резисторов при различных способах соединения. В статье рассмотрено, как рассчитывается сопротивление при и некоторые другие технические вопросы.

Сопротивление проводника

Все проводники имеют свойство препятствовать течению электрического тока, его принято называть электрическим сопротивлением R, оно измеряется в омах. Это основное свойство проводниковых материалов.

Для ведения электротехнических расчётов применяется удельное сопротивление — ρ Ом·м/мм 2 . Все металлы — хорошие проводники, наибольшее применение получили медь и алюминий, гораздо реже применяется железо. Лучший проводник — серебро, оно применяется в электротехнической и электронной промышленности. Широко распространены сплавы с высоким значением сопротивления.

При расчёте сопротивления используется известная из школьного курса физики формула:

R = ρ · l/S, S — площадь сечения; l — длина.

Если взять два проводника, то их сопротивление при параллельном соединении станет меньше из-за увеличения общего сечения.

и нагрев проводника

Для практических расчётов режимов работы проводников применяется понятие плотности тока — δ А/мм 2 , она вычисляется по формуле:

δ = I/S, I — ток, S — сечение.

Ток, проходя по проводнику, нагревает его. Чем больше δ, тем сильнее нагревается проводник. Для проводов и кабелей разработаны нормы допустимой плотности, которые приводятся в Для проводников нагревательных устройств существуют свои нормы плотности тока.

Если плотность δ выше допустимой, может произойти разрушение проводника, например, при перегреве кабеля у него разрушается изоляция.

Правилами регламентируется производить расчёт проводников на нагрев.

Способы соединения проводников

Любой проводник гораздо удобнее изображать на схемах как электрическое сопротивление R, тогда их легко читать и анализировать. Существует всего три способа соединения сопротивлений. Первый способ самый простой — последовательное соединение.


На фото видно, что полное сопротивление равно: R = R 1 + R 2 + R 3 .

Второй способ более сложный — параллельное соединение. Расчёт сопротивления при параллельном соединении выполняется поэтапно. Рассчитывается полная проводимость G = 1/R, а затем полное сопротивление R = 1/G.


Можно поступить и по-другому, прежде рассчитать общее сопротивление при R1 и R2, после этого повторить операцию и найти R.

Третий способ соединения наиболее сложный — смешанное соединение, то есть присутствуют все рассмотренные варианты. Схема приведена на фото.


Для расчёта этой схемы её следует упростить, для этого заменяют резисторы R2 и R3 одним R2,3. Получается несложная схема.

R2,3,4 = R2,3 · R4/(R2,3 + R4).


Схема становится ещё проще, в ней остаются резисторы, имеющие последовательное соединение. В более сложных ситуациях используется этот же метод преобразования.

Виды проводников

В электронной технике, при производстве проводники представляют собою тонкие полоски медной фольги. Ввиду малой длины сопротивление у них незначительно, им во многих случаях можно пренебречь. Для этих проводников сопротивление при параллельном соединении уменьшается вследствие увеличения сечения.

Большой раздел проводников представляют обмоточные провода. Они выпускаются разных диаметров — от 0,02 до 5,6 миллиметра. Для мощных трансформаторов и электродвигателей выпускаются медные шинки прямоугольного сечения. Иногда при ремонте заменяют провод большого диаметра на несколько параллельно соединённых меньшего размера.


Особый раздел проводников представляют провода и кабели, промышленность предоставляет широчайший выбор марок для самых различных нужд. Нередко приходится заменять один кабель на несколько, меньшего сечения. Причины этого бывают самые различные, например, кабель сечением 240 мм 2 очень трудно прокладывать по трассе с крутыми изгибами. Его заменяют на 2×120 мм 2 , и проблема решена.

Расчёт проводов на нагрев

Проводник нагревается протекающим током, если его температура превысит допустимую, наступает разрушение изоляции. ПУЭ предусматривает расчёт проводников на нагрев, исходными данными для него являются сила тока и условия внешней среды, в которой проложен проводник. По этим данным из таблиц в ПУЭ выбирается рекомендуемое проводника или кабеля).

На практике встречаются ситуации, когда нагрузка на действующий кабель сильно возросла. Существует два выхода ‒ заменить кабель на другой, это бывает дорого, или параллельно ему проложить ещё один, чтобы разгрузить основной кабель. В этом случае сопротивление проводника при параллельном соединении уменьшается, следовательно падает выделение тепла.

Чтобы правильно выбрать сечение второго кабеля, пользуются таблицами ПУЭ, важно при этом не ошибиться с определением его рабочего тока. В этой ситуации охлаждение кабелей будет даже лучше, чем у одного. Рекомендуется рассчитать сопротивление при параллельном соединении двух кабелей, чтобы точнее определить их тепловыделение.

Расчёт проводников на потерю напряжения

При расположении потребителя R н на большом расстоянии L от источника энергии U 1 возникает довольно большое на проводах линии. К потребителю R н поступает напряжение U 2 значительно ниже начального U 1 . Практически в качестве нагрузки выступает различное электрооборудование, подключаемое к линии параллельно.

Для решения проблемы производят расчет сопротивления при параллельном соединении всего оборудования, так находится сопротивление нагрузки R н. Далее следует определить сопротивление проводов линии.

R л = ρ · 2L/S,

Здесь S — сечение провода линии, мм 2 .

Параллельное соединение резисторов — одно из двух видов электрических соединений, когда оба вывода одного резистора соединены с соответствующими выводами другого резистора или резисторов. Зачастую или параллельно для того, чтобы создать более сложные электронные схемы.

Схема параллельного соединения показан на рисунке ниже. При параллельном соединении резисторов, напряжение на всех резисторах будет одинаковым, а протекающий через них ток будет пропорционален их сопротивлению:

Формула параллельного соединения резисторов

Общее сопротивление нескольких резисторов соединенных параллельно определяется по следующей формуле:

Ток, протекающий через отдельно взятый резистор, согласно , можно найти по формуле:

Параллельное соединение резисторов — расчет

Пример №1

При разработке устройства, возникла необходимость установить резистор с сопротивлением 8 Ом. Если мы просмотрим весь номинальный ряд стандартных значений резисторов, то мы увидим, что резистора с сопротивлением в 8 Ом в нем нет.

Выходом из данной ситуации будет использование двух параллельно соединенных резисторов. Эквивалентное значение сопротивления для двух резисторов соединенных параллельно рассчитывается следующим образом:

Данное уравнение показывает, что если R1 равен R2, то сопротивление R составляет половину сопротивления одного из двух резисторов. При R = 8 Ом, R1 и R2 должны, следовательно, иметь значение 2 × 8 = 16 Ом.
Теперь проведем проверку, рассчитав общее сопротивление двух резисторов:

Таким образом, мы получили необходимое сопротивление 8 Ом, соединив параллельно два резистора по 16 Ом.

Пример расчета №2

Найти общее сопротивление R из трех параллельно соединенных резисторов:

Общее сопротивление R рассчитывается по формуле:

Этот метод расчета может быть использованы для расчета любого количества отдельных сопротивлений соединенных параллельно.

Один важный момент, который необходимо запомнить при расчете параллельно соединенных резисторов – это то, что общее сопротивление всегда будет меньше, чем значение наименьшего сопротивления в этой комбинации.

Как рассчитать сложные схемы соединения резисторов

Более сложные соединения резисторов могут быть рассчитаны путем систематической группировки резисторов. На рисунке ниже необходимо посчитать общее сопротивление цепи, состоящей из трех резисторов:



Для простоты расчета, сначала сгруппируем резисторы по параллельному и последовательному типу соединения.
Резисторы R2 и R3 соединены последовательно (группа 2). Они в свою очередь соединены параллельно с резистором R1 (группа 1).

Последовательное соединение резисторов группы 2 вычисляется как сумма сопротивлений R2 и R3:

В результате мы упрощаем схему в виде двух параллельных резисторов. Теперь общее сопротивление всей схемы можно посчитать следующим образом:

Расчет более сложных соединений резисторов можно выполнить используя законы Кирхгофа.

Ток, протекающий в цепи параллельно соединенных резисторах

Общий ток I протекающий в цепи параллельных резисторов равняется сумме отдельных токов, протекающих во всех параллельных ветвях, причем ток в отдельно взятой ветви не обязательно должен быть равен току в соседних ветвях.

Несмотря на параллельное соединение, к каждому резистору приложено одно и то же напряжение. А поскольку величина сопротивлений в параллельной цепи может быть разной, то и величина протекающего тока через каждый резистор тоже будет отличаться (по определению закона Ома).

Рассмотрим это на примере двух параллельно соединенных резисторов. Ток, который течет через каждый из резисторов (I1 и I2) будет отличаться друг от друга поскольку сопротивления резисторов R1 и R2 не равны.
Однако мы знаем, что ток, который поступает в цепь в точке «А» должен выйти из цепи в точке «B» .

Первое правило Кирхгофа гласит: «Общий ток, выходящий из цепи равен току входящий в цепь».

Таким образом, протекающий общий ток в цепи можно определить как:

Затем с помощью закона Ома можно вычислить ток, который протекает через каждый резистор:

Ток, протекающий в R1 = U ÷ R1 = 12 ÷ 22 кОм = 0,545 мА

Ток, протекающий в R 2 = U ÷ R2 = 12 ÷ 47 кОм = 0,255 мА

Таким образом, общий ток будет равен:

I = 0,545 мА + 0,255 мА = 0,8 мА

Это также можно проверить, используя закон Ома:

I = U ÷ R = 12 В ÷ 15 кОм = 0,8 мА (то же самое)

где 15кОм — это общее сопротивление двух параллельно соединенных резисторов (22 кОм и 47 кОм)

И в завершении хочется отметить, что большинство современных резисторов маркируются цветными полосками и назначение ее можно узнать .

Параллельное соединение резисторов — онлайн калькулятор

Чтобы быстро вычислить общее сопротивление двух и более резисторов, соединенных параллельно, вы можете воспользоваться следующим онлайн калькулятором:

Подведем итог

Когда два или более резистора соединены так, что оба вывода одного резистора соединены с соответствующими выводами другого резистора или резисторов, то говорят, что они соединены между собой параллельно. Напряжение на каждом резисторе внутри параллельной комбинации одинаковое, но токи, протекающие через них, могут отличаться друг от друга, в зависимости от величины сопротивлений каждого резистора.

Эквивалентное или полное сопротивление параллельной комбинации всегда будет меньше минимального сопротивления резистора входящего в параллельное соединение.

Проверим справедливость показанных здесь формул на простом эксперименте.

Возьмём два резистора МЛТ-2 на 3 и 47 Ом и соединим их последовательно. Затем измерим общее сопротивление получившейся цепи цифровым мультиметром. Как видим оно равно сумме сопротивлений резисторов, входящих в эту цепочку.


Замер общего сопротивления при последовательном соединении

Теперь соединим наши резисторы параллельно и замерим их общее сопротивление.


Измерение сопротивления при параллельном соединении

Как видим, результирующее сопротивление (2,9 Ом) меньше самого меньшего (3 Ом), входящего в цепочку. Отсюда вытекает ещё одно известное правило, которое можно применять на практике:

При параллельном соединении резисторов общее сопротивление цепи будет меньше наименьшего сопротивления, входящего в эту цепь.

Что ещё нужно учитывать при соединении резисторов?

Во-первых, обязательно учитывается их номинальная мощность. Например, нам нужно подобрать замену резистору на 100 Ом и мощностью 1 Вт . Возьмём два резистора по 50 Ом каждый и соединим их последовательно. На какую мощность рассеяния должны быть рассчитаны эти два резистора?

Поскольку через последовательно соединённые резисторы течёт один и тот же постоянный ток (допустим 0,1 А ), а сопротивление каждого из них равно 50 Ом , тогда мощность рассеивания каждого из них должна быть не менее 0,5 Вт . В результате на каждом из них выделится по 0,5 Вт мощности. В сумме это и будет тот самый 1 Вт .

Данный пример достаточно грубоват. Поэтому, если есть сомнения, стоит брать резисторы с запасом по мощности.

Подробнее о мощности рассеивания резистора читайте .

Во-вторых, при соединении стоит использовать однотипные резисторы, например, серии МЛТ. Конечно, нет ничего плохого в том, чтобы брать разные. Это лишь рекомендация.

Каждый в этой жизни сталкивался с резисторами. Люди с гуманитарными профессиями, как и все, изучали в школе на уроках физики проводники электрического тока и закон Ома.

С резисторами также имеют дело студенты технических университетов и инженеры различных производственных предприятий. Перед всеми этими людьми, так или иначе, вставала задача расчёта электрической цепи при различных видах соединения резисторов. В данной статье речь пойдёт о расчёте физических параметров, характеризующих цепь.

Виды соединений

Резистор — пассивный элемент , присутствующий в каждой электрической цепи. Он предназначен для того, чтобы сопротивляться электрическому току. Существует два вида резисторов:

  1. Постоянные.
  2. Переменные.

Зачем же спаивать проводники друг с другом? Например, если для какой-то электрической цепи нужно определённое сопротивление. А среди номинальных показателей нужного нет. В таком случае необходимо подобрать элементы схемы с определёнными значениями сопротивления и соединить их. В зависимости от вида соединения и сопротивлений пассивных элементов мы получим какое-то определённое сопротивление цепи. Оно называется эквивалентным. Его значение зависит от вида спайки проводников. Существует три вида соединения проводников:

  1. Последовательное.
  2. Параллельное.
  3. Смешанное.

Значение эквивалентного сопротивления в цепи считается достаточно легко. Однако, если резисторов в схеме очень много, то лучше воспользоваться специальным калькулятором, который считает это значение. При ведении расчёта вручную, чтобы не допускать ошибок, необходимо проверять, ту ли формулу вы взяли.

Последовательное соединение проводников

В последовательной спайке резисторы идут как бы друг за другом. Значение эквивалентного сопротивления цепи равно сумме сопротивлений всех резисторов. Особенность схем с такой спайкой заключается в том, что значение тока постоянно . Согласно закону Ома, напряжение в цепи равно произведению тока и сопротивления. Так как ток постоянен, то для вычисления напряжения на каждом резисторе, достаточно перемножить значения. После этого необходимо сложить напряжения всех резисторов, и тогда мы получим значение напряжения во всей цепи.

Расчёт очень простой. Так как с ним имеют дело в основном инженеры-разработчики, то для них не составит труда сосчитать всё вручную. Но если резисторов очень много, то проще воспользоваться специальным калькулятором.

Примером последовательного соединения проводников в быту является ёлочная гирлянда.

Параллельное соединение резисторов

При параллельном соединении проводников эквивалентное сопротивление в цепи считается по-другому. Немного сложнее, чем при последовательном.

Его значение в таких цепях равняется произведению сопротивлений всех резисторов, делённому на их сумму. А также есть и другие варианты этой формулы. Параллельное соединение резисторов всегда снижает эквивалентное сопротивление цепи. То есть, его значение всегда будет меньше, чем наибольшее значение какого-то из проводников.

В таких схемах значение напряжения постоянно . То есть значение напряжения во всей цепи равно значениям напряжений каждого из проводников. Оно задаётся источником напряжения.

Сила тока в цепи равна сумме всех токов, протекающих через все проводники. Значение силы тока, протекающего через проводник. равно отношению напряжения источника к сопротивлению этого проводника.

Примеры параллельного соединения проводников:

  1. Освещение.
  2. Розетки в квартире.
  3. Производственное оборудование.

Для расчёта схем с параллельным соединением проводников лучше пользоваться специальным калькулятором. Если в схеме много резисторов, спаянных параллельно, то гораздо быстрее вы посчитаете эквивалентное сопротивление с помощью этого калькулятора.

Смешанное соединение проводников

Этот вид соединения состоит из каскадов резисторов . Например, у нас есть каскад из 10 проводников, соединённых последовательно, и после него идёт каскад из 10 проводников, соединённых параллельно. Эквивалентное сопротивление этой схемы будет равно сумме эквивалентных сопротивлений этих каскадов. То есть, по сути, здесь последовательное соединение двух каскадов проводников.

Многие инженеры занимаются оптимизацией различных схем. Её целью является уменьшение количества элементов в схеме за счёт подбора других, с подходящими значениями сопротивлений. Сложные схемы разбиваются на несколько небольших каскадов, ведь так гораздо проще вести расчёты.

Сейчас, в двадцать первом веке, инженерам стало гораздо проще работать. Ведь несколько десятилетий назад все расчёты производились вручную. А сейчас программисты разработали специальный калькулятор для расчёта эквивалентного сопротивления цепи. В нём запрограммированы формулы, по которым ведутся расчёты.

В этом калькуляторе можно выбрать вид соединения, и потом ввести в специальные поля значения сопротивлений. Через несколько секунд вы уже увидите это значение.

Параллельное и последовательное соединение резисторов

Автор Alexey На чтение 5 мин. Просмотров 428 Опубликовано Обновлено

В электротехнике и электронике очень широко используются резисторы. Применяются они в основном для регулирования в схемах тока и напряжения. Основные параметры : электрическое сопротивление (R) измеряется в Омах, мощность (Вт) , стабильность и точность их параметров  в процессе эксплуатации. Можно вспомнить ещё множество его параметров , — ведь это обычное промышленное изделие.

Последовательное соединение

Последовательное соединение  — это такое соединение, при котором каждый последующий резистор подключается к предыдущему, образуя неразрывную цепь без разветвлений. Ток I=I1=I2 в такой цепи будет одинаковым в каждой её точке. Напротив, напряжение U1, U2 в различных её точках будет разным, причём работа по переносу заряда через всю цепь, складывается из работ по переносу заряда в каждом из резисторов, U=U1+U2. Напряжение U по закону Ома равно току, умноженному на сопротивление, и предыдущее выражение можно записать так:

IR=IR1+IR2,

где R — общее сопротивление цепи. То есть по простому идет падение напряжения в точках соединения резисторов и чем больше подключенных элементов , тем больше происходит падение напряжения

Отсюда следует, что  , общее значение  такого соединения определяется суммированием сопротивлений последовательно . Наши рассуждения справедливы для любого количества последовательно соединяемых участков цепи.

Параллельное соединение

Объединим начала нескольких резисторов (точка А). В другой точке (В) мы соединим все их концы. В результате получим участок цепи, который называется параллельным соединением и состоит из некоторого количества параллельных друг другу ветвей (в нашем случае – резисторов). При этом электрический ток между точками А и B распределится по каждой из этих ветвей.

Напряжения на всех резисторах будут одинаковы: U=U1=U2=U3, их концы — это точки А и В.

Заряды, прошедшие за единицу времени через каждый резистор, в сумме образуют заряд, прошедший через весь блок. Поэтому суммарный ток через изображенную на рисунке цепь I=I1+I2+I3.

Теперь, использовав закон Ома, последнее равенство преобразуется к такому виду:

U/R=U/R1+U/R2+U/R3.

Отсюда следует, что для эквивалентного сопротивления R справедливо:

1/R=1/R1+1/R2+1/R3

или после преобразования формулы мы можем получить другую запись, такого вида : .

Чем большее количество резисторов (или других звеньев электрической цепи, обладающих некоторым сопротивлением) соединить по параллельной схеме, тем больше путей для протекания тока образуется, и тем меньше общее сопротивление цепи.

Следует отметить, что обратная сопротивлению величина называется проводимостью. Можно сказать, что при параллельном соединении участков цепи складываются проводимости этих участков, а при последовательном соединении – их сопротивления.

Примеры использования

Понятно, что при последовательном соединении, разрыв цепи в одном месте приводит к тому, что ток перестает идти по всей цепи. Например, ёлочная гирлянда перестаёт светить, если перегорит всего одна лампочка, это плохо.

Но последовательное соединение лампочек в гирлянде даёт возможность использовать большое количество маленьких лампочек, каждая из которых рассчитана на напряжение сети (220 В), делённое на количество лампочек.

Последовательное соединение резисторов на примере 3-х лампочек и ЭДС

Зато при последовательном подключении предохранительного устройства его срабатывание (разрыв плавкой вставки) позволяет обесточить всю электрическую цепь, расположенную после него и обеспечить нужный уровень безопасности, и это хорошо. Выключатель в сеть питания электроприбора включается также последовательно.

Параллельное соединение также широко используется. Например, люстра – все лампочки соединены параллельно и находятся под одним и тем же напряжением. Если одна лампа перегорит, — не страшно, остальные не погаснут, они остаются под тем же самым напряжением.

Параллельное соединение резисторов на примере 3-х лампочек и генератора

При необходимости увеличения способности схемы рассеивать тепловую мощность, выделяющуюся при протекании тока, широко используются и последовательное, и параллельное объединение резисторов. И для последовательного, и параллельного способов соединения некоторого количества резисторов одного номинала общая мощность равна произведению количества резисторов на мощность одного резистора.

Смешанное соединение резисторов

Также часто используется смешанное соединение . Если ,например необходимо получить сопротивление  определенного номинала, но его нет в наличии можно воспользоваться одним из выше описанных способов или воспользоваться смешанным соединением.

Отсюда , можно вывести формулу которая и даст нам необходимое значение:

Rобщ.=(R1*R2/R1+R2)+R3

В нашу эпоху развития электроники и различных технических устройств в основе всех сложностей лежать простые  законы, которые поверхностно рассматриваются на данном сайте и думаю, что вам они помогут успешно применять в своей жизни. Если например взять ёлочную гирлянду , то соединения лампочек идет друг за другом , т.е. грубо говоря это отдельно-взятое сопротивление.

Не так давно гирлянды стали соединятся смешанным способом. Вообще , в совокупности все эти примеры с резисторами взяты условно , т.е. любым элементом сопротивления может быть  ток проходящий через элемент с падением напряжения и выделением тепла.

Что такое последовательное и параллельное соединение. Параллельное соединение резисторов. Калькулятор для расчета. Параметры цепи при параллельном соединении

Содержание:

Резистор представляет собой устройство, обладающее устойчивым, стабильным значением сопротивления. Это позволяет выполнять регулировку параметров на любых участках электрической цепи. Существуют различные виды соединений, в том числе и смешанное соединение резисторов. От использования того или иного способа в конкретной схеме, напрямую зависит падение напряжений и распределение токов в цепи. Вариант смешанного соединения состоит из последовательного и параллельного подключения активных сопротивлений. Поэтому в первую очередь нужно рассматривать эти два вида соединений, чтобы понять, как работают другие схемы.

Последовательное соединение

Последовательная схема подключения предполагает расположение резисторов в схеме таким образом, что конец первого элемента соединяется с началом второго, а конец второго — с началом третьего и т.д. То есть все резисторы поочередно следуют друг за другом. Сила тока при последовательном соединении будет одинаковой в каждом элементе. В виде формулы это выглядит следующим образом: I общ = I 1 = I 2 , где I общ является общим током цепи, I 1 и I 2 — соответствуют токам 1-го и 2-го резистора.

В соответствии с законом Ома, напряжение источника питания будет равно сумме падений напряжения на каждом резисторе: U общ = U 1 + U 2 = I 1 r 1 + I 2 r 2 , в которой U общ — напряжение источника электроэнергии или самой сети; U 1 и U 2 — значение падений напряжения на 1-м и 2-м резисторах; r 1 и r 2 — сопротивления 1-го и 2-го резисторов. Поскольку токи на любом участке цепи имеют одинаковое значение, формула приобретает вид: Uобщ = I(r 1 + r 2).

Таким образом, можно сделать вывод, что при последовательной схеме включения резисторов, электрический ток, протекающий через каждый из них равен общему значению тока во всей цепи. Напряжение на каждом резисторе будет разное, однако их общая сумма составит значение, равное общему напряжению всей электрической цепи. Общее сопротивление цепи также будет равно сумме сопротивлений каждого резистора, включенного в эту цепь.

Параметры цепи при параллельном соединении

Параллельное соединение представляет собой включение начальных выходов двух и более резисторов в единой точке, и концов этих же элементов в другой общей точке. Таким образом, фактически происходит соединение каждого резистора непосредственно с источником электроэнергии.

В результате, будет одинаковым с общим напряжением цепи: U общ = U 1 = U 2 . В свою очередь, значение токов будет разным на каждом резисторе, их распределение становится прямо пропорциональным сопротивлению этих резисторов. То есть, при увеличении сопротивления, сила тока уменьшается, а общий ток становится равен сумме токов, проходящих через каждый элемент. Формула для данного положения выглядит следующим образом: I общ = I 1 + I 2 .

Для расчетов общего сопротивления используется формула: . Она используется при наличии в цепи только двух сопротивлений. В тех случаях, когда сопротивлений в цепи подключено три и более, применяется другая формула:

Таким образом, значение общего сопротивления электрической цепи будет меньше, чем самое минимальное сопротивление одного из резисторов, подключенных параллельно в эту цепь. На каждый элемент поступает напряжение, одинаковое с напряжением источника электроэнергии. Распределение тока будет прямо пропорциональным . Значение общего сопротивления резисторов, соединенных параллельно, не должно превышать минимального сопротивления какого-либо элемента.

Схема смешанного соединения резисторов

Схема смешанного соединения обладает свойствами схем резисторов. В этом случае элементы частично подключаются последовательно, а другая часть соединяется параллельно. На представленной схеме резисторы R 1 и R 2 включены последовательно, а резистор R 3 соединен параллельно с ними. В свою очередь резистор R 4 включается последовательно с предыдущей группой резисторов R 1 , R 2 и R 3 .

Расчет сопротивления для такой цепи сопряжен с определенными трудностями. Для того чтобы правильно выполнить расчеты используется метод преобразования. Он заключается в последовательном преобразовании сложной цепи в простейшую цепь за несколько этапов.

Если для примера вновь использовать представленную схему, то в самом начале определяется сопротивление R 12 резисторов R 1 и R 2 , включенных последовательно: R 12 = R 1 + R 2 . Далее, нужно определить сопротивление резисторов R 123 , включенных параллельно, по следующей формуле: R 123 =R 12 R 3 /(R 12 +R 3) = (R 1 +R 2)R 3 /(R 1 +R 2 +R 3). На последнем этапе выполняется расчет эквивалентного сопротивления всей цепи, путем суммирования полученных данных R 123 и сопротивления R 4 , включенного последовательно с ним: R эк = R 123 + R 4 = (R 1 + R 2) R 3 / (R 1 + R 2 + R 3) + R 4 .

В заключение следует отметить, что смешанное соединение резисторов обладает положительными и отрицательными качествами последовательного и параллельного соединения. Это свойство успешно используется на практике в электрических схемах.

Здравствуйте, уважаемые читатели сайта . Очень часто в практике радиолюбителя при повторении или наладке радиоэлектронных устройств не всегда под рукой оказывается с нужным сопротивлением, хотя резисторов с другими сопротивлениями имеются в достаточном количестве.

В такой ситуации поступают просто: берут несколько резисторов (два или три) с разными сопротивлениями и, соединяя их последовательно или параллельно , подбирают нужное сопротивление.

В этой статье Вы узнаете, как применяя то или иное соединение можно подобрать необходимое сопротивление.

Последовательным называют соединение, при котором резисторы следуют друг за другом и образуют электрическую цепь из нескольких элементов, в которой конец одного резистора соединен с началом другого и т.д.

В последовательной цепи электрической ток поочередно протекает по всем резисторам и преодолевает сопротивление каждого из них. При этом ток в этой цепи одинаков. И если последовательно соединить два резистора R1 и R2 , их общее (полное) сопротивление Rобщ будет равно сумме их сопротивлений . Это условие справедливо для любого числа резисторов, где:

Например.
При соединении двух резисторов с номиналами R1 = 150 Ом и R2 = 330 Ом их общее сопротивление составит Rобщ = 150 + 330 = 480 Ом.

При соединении трех резисторов R1 = 20 кОм, R2 = 68 кОм и R3 = 180 кОм их общее сопротивление составит Rобщ = 20 + 68 + 180 = 268 кОм.

Запомните . Из нескольких соединенных последовательно резисторов их общее сопротивление Rобщ определяет тот, у которого сопротивление больше по отношению к другим резисторам в этой цепи.

При параллельном соединении резисторов соединяются их одноименные выводы: начальные выводы соединяются в одной точке, а конечные выводы в другой. Такой способ включения облегчает прохождение электрическому току, потому что он разветвляясь, одновременно протекает по всем соединенным таким образом резисторам.

При параллельном соединении резисторов складываются не сопротивления, а их электрические проводимости (величины, обратные сопротивлениям, т.е. 1/R), поэтому общее (полное) сопротивление Rобщ уменьшается и всегда меньше сопротивлений любого резистора в этой цепи. Формула для определения полного сопротивления имеет вид:

Если параллельно включены два резистора с сопротивлениями R1 и R2 , тогда основную формулу немного упрощаем и получаем:

При включении трех резисторов расчет общего сопротивления будет таким:

Например.
При соединении двух резисторов с номиналами R1 = 47 кОм и R2 = 68 кОм их общее сопротивление составит Rобщ = 47 68 / (47 + 68) = 27,8 кОм.

При соединении трех резисторов R1 = 10 Ом, R2 = 15 Ом и R3 = 33 Ом их общее сопротивление равно Rобщ = 10 15 33 / (15 33) + (10 33) + (10 15) = 5,07 Ом.

На заметку . При соединении двух резисторов с одинаковыми номиналами их общее сопротивление Rобщ равно половине сопротивления каждого из них.

Из приведенных примеров можно сделать вывод, что если необходим резистор с большим сопротивлением, применяют последовательное соединение. Если же резистор необходим с меньшим сопротивлением, применяют параллельное соединение.

Параллельным соединением сопротивлений называется такое соединение, когда начала сопротивлений соединены в одну общую точку, а концы — в другую.

Для параллельного соединения сопротивлений характерны следующие свойства:

Напряжения на зажимах всех сопротивлений одинаковы:
U 1 = U 2 = U 3 = U;
— проводимость всех параллельно соединённых сопротивлений равна сумме проводимостей отдельных сопротивлений:
1/R = 1/R 1 + 1/R 2 + 1/R 3 = R 1 R 2 + R 1 R 3 + R 2 R 3 /R 1 R 2 R 3 ,
где R — эквивалентное (равнодействующее) сопротивление трёх сопротивлений (в данном случае R 1 , R 2 и R 3).

Чтобы получить сопротивление такой цепи, надо перевернуть дробь, определяющую величину её проводимости. Следовательно, сопротивление параллельного разветвления из трёх резисторов:
R = R 1 R 2 R 3 /R 1 R 2 + R 2 R 3 + R 1 R 3 .

Эквивалентным сопротивлением называется такое сопротивление, которым можно заменить несколько сопротивлений (включенных параллельно или последовательно), не изменяя величины тока в цепи.

Чтобы найти эквивалентное сопротивление при параллельном соединении, необходимо сложить проводимости всех отдельных участков, т.е. найти общую проводимость. Величина, обратная общей проводимости, и является общим сопротивлением.

При параллельном соединении эквивалентная проводимость равна сумме проводимостей отдельных ветвей, следовательно, эквивалентное сопротивление в этом случае всегда меньше наименьшего из параллельно включенных сопротивлений.

На практике могут быть случаи, когда цепь состоит из более, чем трёх параллельных ветвей. Все полученные соотношения остаются справедливыми и для цепей, состоящих из любого числа параллельно соединённых резисторов.

Найдём эквивалентное сопротивление двух параллельно включенных сопротивлений R 1 и R 2 (см. рис.). Проводимость первой ветви равна 1/R 1 , проводимость второй ветви — 1/R 2 . Общая проводимость:
1/R = 1/R 1 + 1/R 2 .

Приведём к общему знаменателю:
1/R = R 2 + R 1 /R 1 R 2 ,
отсюда эквивалентное сопротивление
R = R 1 R 2 /R 1 + R 2 .

Эта формула и служит для расчётов общего сопротивления цепи, состоящей из двух параллельно включенных сопротивлений.

Таким образом, эквивалентное сопротивление двух параллельно включенных сопротивлений равно произведению этих сопротивлений, делённому на их сумму.

При параллельном соединении n равных сопротивлений R1 эквивалентное сопротивление их будет в n раз меньше, т.е.
R = R 1 /n.

На схеме, изображённой на последнем рисунке, включено пять сопротивлений R 1 по 30 Ом каждое. Следовательно, общее сопротивление R будет
R = R 1 /5 = 30/5 = 6 Ом.

Можно сказать, что сумма токов, подходящих к узловой точке А (на первом рисунке), равна сумме токов, от неё отходящих:
I = I 1 + I 2 + I 3 .

Рассмотрим, как происходит разветвление тока в цепях с сопротивлениями R 1 и R 2 (второй рисунок). Так как напряжение на зажимах этих сопротивлений одинаково, то
U = I 1 R 1 и U = I 2 R 2 .

Левые части этих равенств одинаковы, следовательно, равны и правые части:
I 1 R 1 = I 2 R 2 ,
или
I 1 /I 2 = R 2 /R 1 ,
т.е. ток при параллельном соединении сопротивлений разветвляется обратно пропорционально сопротивлениям ветвей (или прямо пропорционально их проводимостям). Чем больше сопротивление ветви, тем меньше ток в ней, и наоборот.

Таким образом, из нескольких одинаковых резисторов можно получить общий резистор с бОльшей мощностью рассеивания.

При параллельном соединении неодинаковых резисторов в наиболее высокоомном резисторе выделяется наибольшая мощность.

Пример 1. Имеются два сопротивления, включенных параллельно. Сопротивление R 1 = 25 Ом, а R 2 = 50 Ом. Определить общее сопротивление цепи R общ.

Решение. Rобщ = R 1 R 2 /R 1 + R 2 = 25 x 50 / 25 + 50 ≈ 16, 6 Ом.

Пример 2. В ламповом усилителе имеются три лампы, нити накала которых включены параллельно. Ток накала первой лампы I 1 = 1 ампер, второй I 2 = 1, 5 ампера и третьей I 3 = 2, 5 ампера. Определить общий ток цепи накала ламп усилителя I общ.

Решение. I общ = I 1 + I 2 + I 3 = 1 + 1, 5 + 2, 5 = 5 ампер.

Параллельное соединение резисторов часто встречается в радиотехнической аппаратуре. Два или более резисторов включается параллельно в тех случаях, когда ток в цепи слишком большой и может вызвать чрезмерный нагрев резистора.

Примером параллельного соединения потребителей электрической энергии может служить включение электрических ламп обычной осветительной сети, которые соединяются параллельно. Достоинство параллельного соединения потребителей заключается в том, что выключение одного из них не влияет на работу других.

Обычно все затрудняются ответить. А вот загадка эта в применении к электричеству решается вполне определенно.

Электричество начинается с закона Ома.

А уж если рассматривать дилемму в контексте параллельного или последовательного соединений — считая одно соединение курицей, а другое — яйцом, то сомнений вообще нет никаких.

Потому что закон Ома — это и есть самая первоначальная электрическая цепь. И она может быть только последовательной.

Да, придумали гальванический элемент и не знали, что с ним делать, поэтому сразу придумали еще лампочку. И вот что из этого получилось. Здесь напряжение в 1,5 В немедленно потекло в качестве тока, чтобы неукоснительно выполнять закон Ома, через лампочку к задней стенке того же элемента питания. А уж внутри самой батарейки под действием волшебницы-химии заряды снова оказались в первоначальной точке своего похода. И поэтому там, где напряжение было 1,5 вольта, оно таким и остается. То есть, напряжение постоянно одно, а заряды непрерывно движутся и последовательно проходят лампочку и гальванический элемент.

И это обычно рисуют на схеме вот так:

По закону Ома I=U/R

Тогда сопротивление лампочки (с тем током и напряжением, которые я написал) получится

R = 1/U , где R = 1 Ом

А мощность будет выделяться P = I * U , то есть P=2,25 Вm

В последовательной цепи, особенно на таком простом и несомненном примере, видно, что ток, который бежит по ней от начала до конца, — все время один и тот же. А если мы теперь возьмем две лампочки и сделаем так, чтобы ток пробегал сначала по одной, а потом по другой, то будет опять то же самое — ток будет и в той лампочке, и в другой снова одинаковым. Хотя другим по величине. Ток теперь испытывает сопротивление двух лампочек, но у каждой из них сопротивление как было, так и осталось, ведь оно определяется исключительно физическими свойствами самой лампочки. Новый ток вычисляем опять по закону Ома.

Он получится равным I=U/R+R,то есть 0,75А, ровно половина того тока, который был сначала.

В этом случае току приходится преодолевать уже два сопротивления, он становится меньше. Что и видно по свечению лампочек — они теперь горят вполнакала. А общее сопротивление цепочки из двух лампочек будет равно сумме их сопротивлений. Зная арифметику, можно в отдельном случае воспользоваться и действием умножения: если последовательно соединены N одинаковых лампочек, то общее их сопротивление будет равно N, умноженное на R, где R — сопротивление одной лампочки. Логика безупречная.

А мы продолжим наши опыты. Теперь сделаем нечто подобное, что мы провернули с лампочками, но только на левой стороне цепи: добавим еще один гальванический элемент, точно такой, как первый. Как видим, теперь у нас в два раза увеличилось общее напряжение, а ток стал снова 1,5 А, о чем и сигнализируют лампочки, загоревшись снова в полную силу.

Делаем вывод:

  • При последовательном соединении электрической цепи сопротивления и напряжения ее элементов суммируются, а ток на всех элементах остается неизменным.

Легко проверить, что это утверждение справедливо как для активных компонентов (гальванических элементов), так и для пассивных (лампочек, резисторов).

То есть это значит, что напряжение, измеренное на одном резисторе (оно называется падением напряжения), можно смело суммировать с напряжением, измеренным на другом резисторе, и в сумме получатся те же 3 В. А на каждом из сопротивлений оно окажется равным половине — то есть 1,5 В. И это справедливо. Два гальванических элемента вырабатывают свои напряжения, а две лампочки их потребляют. Потому что в источнике напряжения энергия химических процессов превращается в электроэнергию, принявшую вид напряжения, а в лампочках та же самая энергия из электрической превращается в тепловую и световую.

Вернемся к первой схеме, подключим в ней еще одну лампочку, но иначе.

Теперь напряжение в точках, соединяющих две ветки, то же, что и на гальваническом элементе — 1,5 В. Но так как сопротивление у обеих лампочек тоже такое, как и было, то и ток через каждую из них пойдет 1,5 А — ток «полного накала».

Гальванический элемент теперь питает их током одновременно, следовательно, из него вытекают сразу оба эти тока. То есть общий ток из источника напряжения будет равен 1,5 А + 1,5 А = 3,0 А.

В чем же отличие этой схемы от схемы, когда те же самые лампочки были включены последовательно? Только в накале лампочек, то есть только в токе.

Тогда ток был 0,75 А, а теперь он стал сразу 3 А.

Получается, если сравнить с первоначальной схемой, то при последовательном соединении лампочек (схема 2) току сопротивления оказывалось больше (отчего он уменьшался, и лампочки теряли светимость), а параллельное подключение оказывает МЕНЬШЕ сопротивления, хотя сопротивление лампочек осталось неизменным. В чем тут дело?

А дело в том, что мы забываем одну интересную истину, что всякая палка о двух концах.

Когда мы говорим, что резистор сопротивляется току, то как бы забываем, что он ток все-таки проводит. И теперь, когда подключили лампочки параллельно, увеличилось суммарное для них свойство проводить ток, а не сопротивляться ему. Ну и, соответственно, некую величину G , по аналогии с сопротивлением R и следовало бы назвать проводимостью. И должна она в параллельном соединении проводников суммироваться.

Ну и вот она

Закон Ома тогда будет выглядеть

I = U * G &

И в случае параллельного соединения ток I будет равен U*(G+G) = 2*U*G, что мы как раз и наблюдаем.

Замена элементов цепи общим эквивалентным элементом

Инженерам часто приходится узнавать токи и напряжения во всех частях схем. А реальные электрические схемы бывают достаточно сложными и разветвленными и могут содержать множество элементов, активно потребляющих электроэнергию и соединенных друг с другом в совершенно разных сочетаниях. Это называется расчет электрических схем. Он делается при проектировании энергоснабжения домов, квартир, организаций. При этом очень важно, какие токи и напряжения будут действовать в электрической цепи, хотя бы для того, чтобы выбрать подходящие им сечения проводов, нагрузки на всю сеть или ее части, и так далее. А уж насколько сложны бывают электронные схемы, содержащие тысячи, а то и миллионы элементов, думаю, понятно всякому.

Самое первое что, напрашивается — это воспользоваться знанием того, как ведут себя токи напряжения в таких простейших соединениях сети, как последовательное и параллельное. Делают так: вместо найденного в сети последовательного соединения двух или более активных устройств-потребителей (как наши лампочки) нарисовать один, но чтобы его сопротивление было таким же, как у обоих. Тогда картина токов и напряжений в остальной части схемы не изменится. Аналогично и с параллельным соединением: вместо них нарисовать такой элемент, ПРОВОДИМОСТЬ которого была бы такой же, как у обоих.

Теперь если схему перерисовать, заменив последовательные и параллельные соединения одним элементом, то получим схему, которая называется «схемой эквивалентного замещения».

Такую процедуру можно продолжать до тех пор, пока у нас не останется наипростейшая — которой мы в самом начале иллюстрировали закон Ома. Только вместо лампочки будет стоять одно сопротивление, которое и называют эквивалентным сопротивлением нагрузки.

Это первая задача. Она дает нам возможность по закону Ома рассчитать общий ток во всей сети, или общий ток нагрузки.

Вот это и есть полный расчет электрической сети.

Примеры

Пусть цепь содержит 9 активных сопротивлений. Это могут быть лампочки или что-то другое.

На ее входные клеммы подано напряжение в 60 В.

Значения сопротивлений для всех элементов следующие:

Найти все неизвестные токи и напряжения.

Надо пойти по пути поиска параллельных и последовательных участков сети, рассчитывать эквивалентные им сопротивления и постепенно упрощать схему. Видим, что R 3 , R 9 и R 6 соединены последовательно. Тогда им эквивалентное сопротивление R э 3, 6, 9 будет равно их сумме R э 3, 6, 9 = 1 + 4 + 1 Ом = 6 Ом.

Теперь заменяем параллельный кусочек из сопротивлений R 8 и R э 3, 6, 9, получая R э 8, 3, 6, 9 . Только при параллельном соединении проводников, складывать придется проводимости.

Проводимость измеряется в единицах, называемых сименсами, обратных омам.

Если перевернуть дробь, получим сопротивление R э 8, 3, 6, 9 = 2 Ом

Совершенно так же, как в первом случае, объединяем сопротивления R 2 , R э 8, 3, 6, 9 и R 5, включенные последовательно, получая R э 2, 8, 3, 6, 9, 5 = 1 + 2 + 1 = 4 Ом.

Осталось два шага: получить сопротивление, эквивалентное двум резисторам параллельного соединения проводников R 7 и R э 2, 8, 3, 6, 9, 5.

Оно равно R э 7, 2, 8, 3, 6, 9, 5 = 1/(1/4+1/4)=1/(2/4)=4/2 = 2 Ом

На последнем шаге просуммируем все последовательно включенные сопротивления R 1 , R э 7, 2, 8, 3, 6, 9, 5 и R 4 и получим сопротивление, эквивалентное сопротивлению всей цепи R э и равное сумме этих трех сопротивлений

R э = R 1 + R э 7, 2, 8, 3, 6, 9, 5 + R4 = 1 + 2 + 1 = 4 Ом

Ну и вспомним, в честь кого назвали единицу сопротивлений, написанную нами в последней из этих формул, и вычислим по его закону общий ток во всей цепи I

Теперь, двигаясь в обратном направлении, в сторону все большего усложнения сети, можно получать по закону Ома токи и напряжения во всех цепочках нашей достаточно простой схемы.

Так обычно и рассчитывают схемы электроснабжения квартир, которые состоят из параллельных и последовательных участков. Что, как правило, не годится в электронике, потому что там многое по-другому устроено, и все гораздо замысловатее. И вот такую, например, схему, когда не поймешь, параллельное это соединение проводников или последовательное, рассчитывают по законам Кирхгофа.

Резистор — это элемент электрической схемы, который обладает сопротивлением электрическому току. Классифицируют два типа резисторов: постоянные и переменные (подстроечные). При моделировании той или иной электрической схемы, а также при ремонте электронных изделий, возникает необходимость использовать резистор определенного номинала. Хотя и существует множество различных номиналов постоянных резисторов, в данный момент под рукой может не оказаться требуемого, либо резистора с таким номиналом не существует. Чтобы выйти из такой ситуации, можно использовать как последовательное так и параллельное соединение резисторов. О том, как правильно произвести расчет и подбор различных номиналов сопротивлений, будет рассказано в этой статье.

Последовательное соединение резисторов — это самая элементарная схема сборки радиодеталей, оно применяется для увеличения общего сопротивления цепи. При последовательном соединении, сопротивление используемых резисторов просто складывается, а вот при параллельном соединении необходимо производить расчет по нижеописанным формулам. Параллельное соединение необходимо для снижения результирующего сопротивления, а также для увеличения мощности, несколько параллельно подключенных резисторов имеют большую мощность, чем у одного.

На фотографии можно увидеть параллельное подключение резисторов.

Ниже представлена принципиальная схема параллельного соединения резисторов.

Общее номинальное сопротивление необходимо рассчитывать по следующей схеме:

R(общ)=1/(1/R1+1/R2+1/R3+1/R n).

R1, R2, R3 и Rn — параллельно подключенные резисторы.

Когда параллельное соединение резисторов состоит всего из двух элементов, в таком случае общее номинальное сопротивление можно высчитать по следующей формуле:

R(общ)=R1*R2/R1+R2.

R(общ) — общее сопротивление;

R1, R2 — параллельно подключенные резисторы.

В радиотехнике существует следующее правило: если параллельное подключение резисторов состоит из элементов одного номинала, то результирующее сопротивление можно высчитать, разделив номинал резистора на количество соединенных резисторов:

R(общ) — общее сопротивление;

R — номинал параллельно подключенного резистора;

N — количество соединенных элементов.

Важно учитывать, что при параллельном соединении результирующее сопротивление всегда будет ниже, чем сопротивление самого малого по номиналу резистора.

Приведем практический пример: возьмем три резистора, со следующими значениями номинального сопротивления: 100 Ом, 150 Ом и 30 Ом. Проведем расчет общего сопротивления, по первой формуле:

R(общ)=1/(1/100+1/150+1/30)=1/(0,01+0,007+0,03)=1/0,047=21,28Ом.

После расчета формулы мы видим, что параллельное соединение резисторов, состоящее из трех элементов, с наименьшим номиналом 30 Ом, в результате дает общее сопротивление в электрической цепи 21,28 Ом, что ниже наименьшего номинального сопротивления в цепи почти на 30 процентов.

Параллельное соединение резисторов чаще всего используют в тех случаях, когда необходимо получить сопротивление с большей мощностью. В таком случае необходимо взять резисторы одинаковой мощности и с одинаковым сопротивлением. Результирующая мощность в таком случае рассчитывается путем умножения мощности одного элемента сопротивления на общее количество параллельно подключенных резисторов в цепи.

Например: пять резисторов с номиналом в 100 Ом и с мощностью 1 Вт в каждом, подключенные параллельно, имеют общее сопротивление 20 Ом и мощность 5 Вт.

При последовательном подключении тех же резисторов (мощность так же складывается), получим результирующую мощность 5 Вт, общее сопротивление составит 500 Ом.

последовательное, параллельное, смешанное соединение. Расчет сопротивления

Резисторы между собой могут быть соединены двумя основными способами: последовательно и параллельно. Смешанное соединение резисторов является их комбинацией.

Сочетания любых соединений резисторов можно привести к одному резистору, расчетом сопротивления которого (R) мы сейчас займемся.

ПАРАЛЛЕЛЬНОЕ СОЕДИНЕНИЕ РЕЗИСТОРОВ

Давайте рассчитаем общее сопротивление такой цепи (рисунок 1). Для этого нам понадобится закон Ома — I=U/R и закон Кирхгофа — I=I1+I2+..In

С учетом этого имеем:

  • I=U/R
  • I1=U/R1
  • I2=U/R2
  • In=U/Rn
  • U/R=U/R1+U/R2+…U/Rn
  • 1/R=1/R1+1/R2+…1/Rn

Последняя формула является основной для расчета сопротивления цепи параллельно соединенных резисторов. Для двух резисторов ее можно записать более удобно: R=(R1*R2)/(R1+R2).

Отсюда следует, что в случае параллельного соединения двух одинаковых по номиналу резисторов (R1=R2) их общее сопротивление будет вдвое меньше любого из них. Это полезно помнить.

ПОСЛЕДОВАТЕЛЬНОЕ СОЕДИНЕНИЕ РЕЗИСТОРОВ

Используя уже упомянутые законы для цепи последовательно соединенных резисторов (рисунок 2) можем записать:

  • U=I*R
  • I=I1=I2=…In
  • U=U1+U2+…Un
  • I*R=I*R1+I*R2+…I*Rn
  • R=R1+R2+…Rn

То есть общее сопротивление резисторов при последовательном соединении равно сумме их сопротивлений.

СМЕШАННОЕ СОЕДИНЕНИЕ РЕЗИСТОРОВ

Такое соединение всегда можно представить как комбинацию последовательного и параллельного соединений (рис.3).

Расчет общего сопротивления цепи при этом производится поэтапно. В приведенном примере рассчитываем:

  1. последовательное сопротивление резисторов Rпосл=R1+R2
  2. параллельное соединение R=(Rпосл*R3)/(Rпосл+R3)

Безусловно, могут встретиться более сложные варианты, но методика расчета их сопротивления та же.

Несколько слов про то, когда возникает необходимость соединять резисторы тем или иным способом:

  1. Отсутствие «под рукой» резистора нужного номинала. При этом следует помнить, что погрешности резисторов будут суммироваться.

    Например, для рисунка 3.a, если фактическая погрешность R1 составляет +10%, а R2 имеет +15%, то для Rпосл она будет +25%.

    Здесь следует обращать внимание на знак, то есть для -10% и +15% в результате получим +5%.

  2. Необходимость получить большую мощность.

    Здесь надо учесть, что при одинаковых номиналах сопротивлений и мощностей соединяемых резисторов, как при последовательном, так и при параллельном их соединении итоговая мощность будет равна сумме мощностей.

    В противном случае следует ее рассчитать, используя закон Ома и формулу для определения рассеиваемой мощности P=I*U.

Про мощность и номиналы резисторов можно почитать здесь.

© 2012-2020 г. Все права защищены.

Представленные на сайте материалы имеют информационный характер и не могут быть использованы в качестве руководящих и нормативных документов


При последовательном соединении сопротивление равно. Параллельное соединение сопротивлениий (резисторов)

Последовательное, параллельное и смешанное соединения резисторов. Значительное число приемников, включенных в электрическую цепь (электрические лампы, электронагревательные приборы и др.), можно рассматривать как некоторые элементы, имеющие определенное сопротивление. Это обстоятельство дает нам возможность при составлении и изучении электрических схем заменять конкретные приемники резисторами с определенными сопротивлениями. Различают следующие способы соединения резисторов (приемников электрической энергии): последовательное, параллельное и смешанное.

Последовательное соединение резисторов . При последовательном соединении нескольких резисторов конец первого резистора соединяют с началом второго, конец второго — с началом третьего и т. д. При таком соединении по всем элементам последовательной цепи проходит
один и тот же ток I.
Последовательное соединение приемников поясняет рис. 25, а.
.Заменяя лампы резисторами с сопротивлениями R1, R2 и R3, получим схему, показанную на рис. 25, б.
Если принять, что в источнике Ro = 0, то для трех последовательно соединенных резисторов согласно второму закону Кирхгофа можно написать:

E = IR 1 + IR 2 + IR 3 = I(R 1 + R 2 + R 3) = IR эк (19)

где R эк = R 1 + R 2 + R 3 .
Следовательно, эквивалентное сопротивление последовательной цепи равно сумме сопротивлений всех последовательно соединенных резисторов.Так как напряжения на отдельных участках цепи согласно закону Ома: U 1 =IR 1 ; U 2 = IR 2 , U 3 = IR з и в данном случае E = U, то длярассматриваемой цепи

U = U 1 + U 2 +U 3 (20)

Следовательно, напряжение U на зажимах источника равно сумме напряжений на каждом из последовательно включенных резисторов.
Из указанных формул следует также, что напряжения распределяются между последовательно соединенными резисторами пропорционально их сопротивлениям:

U 1: U 2: U 3 = R 1: R 2: R 3 (21)

т. е. чем больше сопротивление какого-либо приемника в последовательной цепи, тем больше приложенное к нему напряжение.

В случае если последовательно соединяются несколько, например п, резисторов с одинаковым сопротивлением R1, эквивалентное сопротивление цепи Rэк будет в п раз больше сопротивления R1, т. е. Rэк = nR1. Напряжение U1 на каждом резисторе в этом случае в п раз меньше общего напряжения U:

При последовательном соединении приемников изменение сопротивления одного из них тотчас же влечет за собой изменение напряжения на других связанных с ним приемниках. При выключении или обрыве электрической цепи в одном из приемников и в остальных приемниках прекращается ток. Поэтому последовательное соединение приемников применяют редко — только в том случае, когда напряжение источника электрической энергии больше номинального напряжения, на которое рассчитан потребитель. Например, напряжение в электрической сети, от которой питаются вагоны метрополитена, составляет 825 В, номинальное же напряжение электрических ламп, применяемых в этих вагонах, 55 В. Поэтому в вагонах метрополитена электрические лампы включают последовательно по 15 ламп в каждой цепи.
Параллельное соединение резисторов . При параллельном соединении нескольких приемников они включаются между двумя точками электрической цепи, образуя параллельные ветви (рис. 26, а). Заменяя

лампы резисторами с сопротивлениями R1, R2, R3, получим схему, показанную на рис. 26, б.
При параллельном соединении ко всем резисторам приложено одинаковое напряжение U. Поэтому согласно закону Ома:

I 1 =U/R 1 ; I 2 =U/R 2 ; I 3 =U/R 3 .

Ток в неразветвленной части цепи согласно первому закону Кирхгофа I = I 1 +I 2 +I 3 , или

I = U / R 1 + U / R 2 + U / R 3 = U (1/R 1 + 1/R 2 + 1/R 3) = U / R эк (23)

Следовательно, эквивалентное сопротивление рассматриваемой цепи при параллельном соединении трех резисторов определяется формулой

1/R эк = 1/R 1 + 1/R 2 + 1/R 3 (24)

Вводя в формулу (24) вместо значений 1/R эк, 1/R 1 , 1/R 2 и 1/R 3 соответствующие проводимости G эк, G 1 , G 2 и G 3 , получим: эквивалентная проводимость параллельной цепи равна сумме проводимостей параллельно соединенных резисторов :

G эк = G 1 + G 2 +G 3 (25)

Таким образом, при увеличении числа параллельно включаемых резисторов результирующая проводимость электрической цепи увеличивается, а результирующее сопротивление уменьшается.
Из приведенных формул следует, что токи распределяются между параллельными ветвями обратно пропорционально их электрическим сопротивлениям или прямо пропорционально их проводимостям. Например, при трех ветвях

I 1: I 2: I 3 = 1/R 1: 1/R 2: 1/R 3 = G 1 + G 2 + G 3 (26)

В этом отношении имеет место полная аналогия между распределением токов по отдельным ветвям и распределением потоков воды по трубам.
Приведенные формулы дают возможность определить эквивалентное сопротивление цепи для различных конкретных случаев. Например, при двух параллельно включенных резисторах результирующее сопротивление цепи

R эк =R 1 R 2 /(R 1 +R 2)

при трех параллельно включенных резисторах

R эк =R 1 R 2 R 3 /(R 1 R 2 +R 2 R 3 +R 1 R 3)

При параллельном соединении нескольких, например n, резисторов с одинаковым сопротивлением R1 результирующее сопротивление цепи Rэк будет в n раз меньше сопротивления R1, т.е.

R эк = R1 / n (27)

Проходящий по каждой ветви ток I1, в этом случае будет в п раз меньше общего тока:

I1 = I / n (28)

При параллельном соединении приемников, все они находятся под одним и тем же напряжением, и режим работы каждого из них не зависит от остальных. Это означает, что ток, проходящий по какому-либо из приемников, не будет оказывать существенного влияния на другие приемники. При всяком выключении или выходе из строя любого приемника остальные приемники остаются вклю-

ченными. Поэтому параллельное соединение имеет существенные преимущества перед последовательным, вследствие чего оно получило наиболее широкое распространение. В частности, электрические лампы и двигатели, предназначенные для работы при определенном (номинальном) напряжении, всегда включают параллельно.
На электровозах постоянного тока и некоторых тепловозах тяговые двигатели в процессе регулирования скорости движения нужно включать под различные напряжения, поэтому они в процессе разгона переключаются с последовательного соединения на параллельное.

Смешанное соединение резисторов . Смешанным соединением называется такое соединение, при котором часть резисторов включается последовательно, а часть — параллельно. Например, в схеме рис. 27, а имеются два последовательно включенных резистора сопротивлениями R1 и R2, параллельно им включен резистор сопротивлением Rз, а резистор сопротивлением R4 включен последовательно с группой резисторов сопротивлениями R1, R2 и R3.
Эквивалентное сопротивление цепи при смешанном соединении обычно определяют методом преобразования, при котором сложную цепь последовательными этапами преобразовывают в простейшую. Например, для схемы рис. 27, а вначале определяют эквивалентное сопротивление R12 последовательно включенных резисторов с сопротивлениями R1 и R2: R12 = R1 + R2. При этом схема рис. 27, а заменяется эквивалентной схемой рис. 27, б. Затем определяют эквивалентное сопротивление R123 параллельно включенных сопротивлений и R3 по формуле

R 123 =R 12 R 3 /(R 12 +R 3)=(R 1 +R 2)R 3 /(R 1 +R 2 +R 3).

При этом схема рис. 27, б заменяется эквивалентной схемой рис. 27, в. После этого находят эквивалентное сопротивление всей цепи суммированием сопротивления R123 и последовательно включенного с ним сопротивления R4:

R эк = R 123 + R 4 = (R 1 + R 2) R 3 / (R 1 + R 2 + R 3) + R 4

Последовательное, параллельное и смешанное соединения широко применяют для изменения сопротивления пусковых реостатов при пуске э. п. с. постоянного тока.

1. При последовательном соединении проводников

1. Сила тока во всех проводниках одинакова :

I 1 = I 2 = I

2. Общее напряжение U на обоих проводниках равно сумме напряжений U 1 и U 2 на каждом проводнике :

U = U 1 + U 2

3. По закону Ома, напряжения U 1 и U 2 на проводниках равны U 1 = IR 1 , U 2 = IR 2 а общее напряжение U = IR где R – электрическое сопротивление всей цепи, тогда IR = IR 1 + I R 2. Отсюда следует

R = R 1 + R 2

При последовательном соединении полное сопротивление цепи равно сумме сопротивлений отдельных проводников.

Этот результат справедлив для любого числа последовательно соединенных проводников.

2. При параллельном соединении проводников

1. Напряжения U 1 и U 2 на обоих проводниках одинаковы

U 1 = U 2 = U

2. Сумма токов I 1 + I 2 , протекающих по обоим проводникам, равна току в неразветвленной цепи :

I = I 1 + I 2

Этот результат следует из того, что в точках разветвления токов (узлы A и B ) в цепи постоянного тока не могут накапливаться заряды. Например, к узлу A за время Δt подтекает заряд I Δt , а утекает от узла за то же время заряд I 1 Δt + I 2 Δt . Следовательно, I = I 1 + I 2 .

3. Записывая на основании закона Ома

где R – электрическое сопротивление всей цепи, получим

При параллельном соединении проводников величина, обратная общему сопротивлению цепи, равна сумме величин, обратных сопротивлениям параллельно включенных проводников.

Этот результат справедлив для любого числа параллельно включенных проводников.

Формулы для последовательного и параллельного соединения проводников позволяют во многих случаях рассчитывать сопротивление сложной цепи, состоящей из многих резисторов. На рисунке приведен пример такой сложной цепи и указана последовательность вычислений. Сопротивления всех проводников указаны в омах (Ом).


На пракутике одного источника тока в цепи бывает недостаточно, и тогда источники тока тоже соединяют между собой для питания цепи. Соединение источников в батарею может быть последовательным и параллельным.

При последовательном соединении два соседних источника соединяются разноименными полюсами.

Т.е., для последовательного соединения аккумуляторов, к ″плюсу″ электрической схемы подключают положительную клемму первого аккумулятора. К его отрицательной клемме подключают положительную клемму второго аккумулятора и т.д. Отрицательную клемму последнего аккумулятора подключают к ″минусу″ электрической схемы.

Получившаяся при последовательном соединении аккумуляторная батарея имеет ту же емкость, что и у одиночного аккумулятора, а напряжение такой аккумуляторной батареи равно сумме напряжений входящих в нее аккумуляторов. Т.е. если аккумуляторы имеют одинаковые напряжения, то напряжение батареи равно напряжению одного аккумулятора, умноженному на количество аккумуляторов в аккумуляторной батарее.


1. ЭДС батареи равна сумме ЭДС отдельных источников ε= ε 1 + ε 2 + ε 3

2 . Общее сопротивление батареи источников равно сумме внутренних сопротивлений отдельных источников r батареи = r 1 + r 2 + r 3

Если в батарею соединены n одинаковых источников, то ЭДС батареи ε= nε 1, а сопротивление r батареи = nr 1

3.

При параллельном соединении соединяют между собой все положительные и все отрицательные полюсы двух или n источников.

Т.е., при параллельном соединении, аккумуляторы соединяют так, чтобы положительные клеммы всех аккумуляторов были подключены к одной точке электрической схемы (″плюсу″), а отрицательные клеммы всех аккумуляторов были подключены к другой точке схемы (″минусу″).

Параллельно соединяют только источники с одинаковой ЭДС . Получившаяся при параллельном соединении аккумуляторная батарея имеет то же напряжение, что и у одиночного аккумулятора, а емкость такой аккумуляторной батареи равна сумме емкостей входящих в нее аккумуляторов. Т.е. если аккумуляторы имеют одинаковые емкости, то емкость аккумуляторной батареи равна емкости одного аккумулятора, умноженной на количество аккумуляторов в батарее.


1. ЭДС батареи одинаковых источников равна ЭДС одного источника. ε= ε 1 = ε 2 = ε 3

2. Сопротивление батареи меньше, чем сопротивление одного источника r батареи = r 1 /n
3. Сила тока в такой цепи по закону Ома

Электрическая энергия, накопленная в аккумуляторной батарее равна сумме энергий отдельных аккумуляторов (произведению энергий отдельных аккумуляторов, если аккумуляторы одинаковые), независимо от того, как соединены аккумуляторы — параллельно или последовательно.

Внутреннее сопротивление аккумуляторов, изготовленных по одной технологии, примерно обратно пропорционально емкости аккумулятора. Поэтому т.к.при параллельном соединении емкость аккумуляторной батареи равна сумме емкостей входящих в нее аккумуляторов, т.е увеличивается, то внутреннее сопротивление уменьшается.

Параллельным соединением сопротивлений называется такое соединение, когда начала сопротивлений соединены в одну общую точку, а концы — в другую.

Для параллельного соединения сопротивлений характерны следующие свойства:

Напряжения на зажимах всех сопротивлений одинаковы:

U 1 = U 2 =U 3 =U ;

Проводимость всех параллельно соединённых сопротивлений равна сумме проводимостей отдельных сопротивлений:

1/R = 1/R 1 + 1/R 2 + 1/R 3 = R 1 R 2 + R 1 R 3 + R 2 R 3 /R 1 R 2 R 3 ,

где R — эквивалентное (равнодействующее) сопротивление трёх сопротивлений (в данном случае R 1 , R 2 и R 3 ) .

Чтобы получить сопротивление такой цепи, надо перевернуть дробь, определяющую величину её проводимости. Следовательно, сопротивление параллельного разветвления из трёх резисторов:

R = R 1 R 2 R 3 /R 1 R 2 + R 2 R 3 + R 1 R 3 .

Эквивалентным сопротивлением называется такое сопротивление, которым можно заменить несколько сопротивлений (включенных параллельно или последовательно), не изменяя величины тока в цепи.

Чтобы найти эквивалентное сопротивление при параллельном соединении, необходимо сложить проводимости всех отдельных участков, т.е. найти общую проводимость. Величина, обратная общей проводимости, и является общим сопротивлением.

При параллельном соединении эквивалентная проводимость равна сумме проводимостей отдельных ветвей, следовательно, эквивалентное сопротивление в этом случае всегда меньше наименьшего из параллельно включенных сопротивлений.

На практике могут быть случаи, когда цепь состоит из более, чем трёх параллельных ветвей. Все полученные соотношения остаются справедливыми и для цепей, состоящих из любого числа параллельно соединённых резисторов.

Найдём эквивалентное сопротивление двух параллельно включенных сопротивлений R 1 и R 2 (см. рис.). Проводимость первой ветви равна 1/R 1 , проводимость второй ветви — 1/R 2 . Общая проводимость:

1/R = 1/R 1 + 1/R 2 .

Приведём к общему знаменателю:

1/R = R 2 + R 1 /R 1 R 2 ,

отсюда эквивалентное сопротивление

R = R 1 R 2 /R 1 + R 2 .

Эта формула и служит для расчётов общего сопротивления цепи, состоящей из двух параллельно включенных сопротивлений.

Таким образом, эквивалентное сопротивление двух параллельно включенных сопротивлений равно произведению этих сопротивлений, делённому на их сумму.

При параллельном соединении n равных сопротивлений R 1 эквивалентное сопротивление их будет в n раз меньше, т.е.

R = R 1 /n .

На схеме, изображённой на последнем рисунке, включено пять сопротивлений R 1 по 30 Ом каждое. Следовательно, общее сопротивление R будет

R = R 1 /5 = 30/5 = 6 Ом.

Можно сказать, что сумма токов, подходящих к узловой точке А (на первом рисунке), равна сумме токов, от неё отходящих:

I = I 1 + I 2 + I 3 .

Рассмотрим, как происходит разветвление тока в цепях с сопротивлениями R 1 и R 2 (второй рисунок). Так как напряжение на зажимах этих сопротивлений одинаково, то

U = I 1 R 1 и U = I 2 R 2 .

Левые части этих равенств одинаковы, следовательно, равны и правые части:

I 1 R 1 = I 2 R 2 ,

или

I 1 /I 2 = R 2 /R 1 ,

Т.е. ток при параллельном соединении сопротивлений разветвляется обратно пропорционально сопротивлениям ветвей (или прямо пропорционально их проводимостям). Чем больше сопротивление ветви, тем меньше ток в ней, и наоборот.

Таким образом, из нескольких одинаковых резисторов можно получить общий резистор с бОльшей мощностью рассеивания.

При параллельном соединении неодинаковых резисторов в наиболее высокоомном резисторе выделяется наибольшая мощность.

Пример 1. Имеются два сопротивления, включенных параллельно. Сопротивление R 1 = 25 Ом, а R 2 = 50 Ом. Определить общее сопротивление цепи R общ .

Решение. R общ = R 1 R 2 /R 1 + R 2 = 25 . 50 / 25 + 50 ≈ 16, 6 Ом.

Пример 2. В ламповом усилителе имеются три лампы, нити накала которых включены параллельно. Ток накала первой лампы I 1 = 1 ампер, второй I 2 = 1, 5 ампера и третьей I 3 = 2, 5 ампера. Определить общий ток цепи накала ламп усилителя I общ .

Решение. I общ = I 1 + I 2 + I 3 = 1 + 1, 5 + 2, 5 = 5 ампер.

Параллельное соединение резисторов часто встречается в радиотехнической аппаратуре. Два или более резисторов включается параллельно в тех случаях, когда ток в цепи слишком большой и может вызвать чрезмерный нагрев резистора.

Примером параллельного соединения потребителей электрической энергии может служить включение электрических ламп обычной осветительной сети, которые соединяются параллельно. Достоинство параллельного соединения потребителей заключается в том, что выключение одного из них не влияет на работу других.

формула расчета общего сопротивления. Примеры параллельного соединения проводников

Резистор — это элемент электрической схемы, который обладает сопротивлением электрическому току. Классифицируют два типа резисторов: постоянные и переменные (подстроечные). При моделировании той или иной электрической схемы, а также при ремонте электронных изделий, возникает необходимость использовать резистор определенного номинала. Хотя и существует множество различных номиналов постоянных резисторов, в данный момент под рукой может не оказаться требуемого, либо резистора с таким номиналом не существует. Чтобы выйти из такой ситуации, можно использовать как последовательное так и параллельное соединение резисторов. О том, как правильно произвести расчет и подбор различных номиналов сопротивлений, будет рассказано в этой статье.

Последовательное соединение резисторов — это самая элементарная схема сборки радиодеталей, оно применяется для увеличения общего сопротивления цепи. При последовательном соединении, сопротивление используемых резисторов просто складывается, а вот при параллельном соединении необходимо производить расчет по нижеописанным формулам. Параллельное соединение необходимо для снижения результирующего сопротивления, а также для увеличения мощности, несколько параллельно подключенных резисторов имеют большую мощность, чем у одного.

На фотографии можно увидеть параллельное подключение резисторов.

Ниже представлена принципиальная схема параллельного соединения резисторов.

Общее номинальное сопротивление необходимо рассчитывать по следующей схеме:

R(общ)=1/(1/R1+1/R2+1/R3+1/R n).

R1, R2, R3 и Rn — параллельно подключенные резисторы.

Когда параллельное соединение резисторов состоит всего из двух элементов, в таком случае общее номинальное сопротивление можно высчитать по следующей формуле:

R(общ)=R1*R2/R1+R2.

R(общ) — общее сопротивление;

R1, R2 — параллельно подключенные резисторы.

В радиотехнике существует следующее правило: если параллельное подключение резисторов состоит из элементов одного номинала, то результирующее сопротивление можно высчитать, разделив номинал резистора на количество соединенных резисторов:

R(общ) — общее сопротивление;

R — номинал параллельно подключенного резистора;

N — количество соединенных элементов.

Важно учитывать, что при параллельном соединении результирующее сопротивление всегда будет ниже, чем сопротивление самого малого по номиналу резистора.

Приведем практический пример: возьмем три резистора, со следующими значениями номинального сопротивления: 100 Ом, 150 Ом и 30 Ом. Проведем расчет общего сопротивления, по первой формуле:

R(общ)=1/(1/100+1/150+1/30)=1/(0,01+0,007+0,03)=1/0,047=21,28Ом.

После расчета формулы мы видим, что параллельное соединение резисторов, состоящее из трех элементов, с наименьшим номиналом 30 Ом, в результате дает общее сопротивление в электрической цепи 21,28 Ом, что ниже наименьшего номинального сопротивления в цепи почти на 30 процентов.

Параллельное соединение резисторов чаще всего используют в тех случаях, когда необходимо получить сопротивление с большей мощностью. В таком случае необходимо взять резисторы одинаковой мощности и с одинаковым сопротивлением. Результирующая мощность в таком случае рассчитывается путем умножения мощности одного элемента сопротивления на общее количество параллельно подключенных резисторов в цепи.

Например: пять резисторов с номиналом в 100 Ом и с мощностью 1 Вт в каждом, подключенные параллельно, имеют общее сопротивление 20 Ом и мощность 5 Вт.

При последовательном подключении тех же резисторов (мощность так же складывается), получим результирующую мощность 5 Вт, общее сопротивление составит 500 Ом.

Проверим справедливость показанных здесь формул на простом эксперименте.

Возьмём два резистора МЛТ-2 на 3 и 47 Ом и соединим их последовательно. Затем измерим общее сопротивление получившейся цепи цифровым мультиметром. Как видим оно равно сумме сопротивлений резисторов, входящих в эту цепочку.


Замер общего сопротивления при последовательном соединении

Теперь соединим наши резисторы параллельно и замерим их общее сопротивление.


Измерение сопротивления при параллельном соединении

Как видим, результирующее сопротивление (2,9 Ом) меньше самого меньшего (3 Ом), входящего в цепочку. Отсюда вытекает ещё одно известное правило, которое можно применять на практике:

При параллельном соединении резисторов общее сопротивление цепи будет меньше наименьшего сопротивления, входящего в эту цепь.

Что ещё нужно учитывать при соединении резисторов?

Во-первых, обязательно учитывается их номинальная мощность. Например, нам нужно подобрать замену резистору на 100 Ом и мощностью 1 Вт . Возьмём два резистора по 50 Ом каждый и соединим их последовательно. На какую мощность рассеяния должны быть рассчитаны эти два резистора?

Поскольку через последовательно соединённые резисторы течёт один и тот же постоянный ток (допустим 0,1 А ), а сопротивление каждого из них равно 50 Ом , тогда мощность рассеивания каждого из них должна быть не менее 0,5 Вт . В результате на каждом из них выделится по 0,5 Вт мощности. В сумме это и будет тот самый 1 Вт .

Данный пример достаточно грубоват. Поэтому, если есть сомнения, стоит брать резисторы с запасом по мощности.

Подробнее о мощности рассеивания резистора читайте .

Во-вторых, при соединении стоит использовать однотипные резисторы, например, серии МЛТ. Конечно, нет ничего плохого в том, чтобы брать разные. Это лишь рекомендация.

На практике нередко встречается задача нахождения сопротивления проводников и резисторов при различных способах соединения. В статье рассмотрено, как рассчитывается сопротивление при параллельном соединении проводников и некоторые другие технические вопросы.

Сопротивление проводника

Все проводники имеют свойство препятствовать течению электрического тока, его принято называть электрическим сопротивлением R, оно измеряется в омах. Это основное свойство проводниковых материалов.

Для ведения электротехнических расчётов применяется удельное сопротивление — ρ Ом·м/мм 2 . Все металлы — хорошие проводники, наибольшее применение получили медь и алюминий, гораздо реже применяется железо. Лучший проводник — серебро, оно применяется в электротехнической и электронной промышленности. Широко распространены сплавы с высоким

При расчёте сопротивления используется известная из школьного курса физики формула:

R = ρ · l/S, S — площадь сечения; l — длина.

Если взять два проводника, то их сопротивление при параллельном соединении станет меньше из-за увеличения общего сечения.

и нагрев проводника

Для практических расчётов режимов работы проводников применяется понятие плотности тока — δ А/мм 2 , она вычисляется по формуле:

δ = I/S, I — ток, S — сечение.

Ток, проходя по проводнику, нагревает его. Чем больше δ, тем сильнее нагревается проводник. Для проводов и кабелей разработаны нормы допустимой плотности, которые приводятся в Для проводников нагревательных устройств существуют свои нормы плотности тока.

Если плотность δ выше допустимой, может произойти разрушение проводника, например, при перегреве кабеля у него разрушается изоляция.

Правилами регламентируется производить расчёт проводников на нагрев.

Способы соединения проводников

Любой проводник гораздо удобнее изображать на схемах как электрическое сопротивление R, тогда их легко читать и анализировать. Существует всего три способа соединения сопротивлений. Первый способ самый простой — последовательное соединение.

На фото видно, что полное сопротивление равно: R = R 1 + R 2 + R 3 .

Второй способ более сложный — параллельное соединение. Расчёт сопротивления при параллельном соединении выполняется поэтапно. Рассчитывается полная проводимость G = 1/R, а затем полное сопротивление R = 1/G.

Можно поступить и по-другому, прежде рассчитать общее сопротивление при R1 и R2, после этого повторить операцию и найти R.

Третий способ соединения наиболее сложный — смешанное соединение, то есть присутствуют все рассмотренные варианты. Схема приведена на фото.

Для расчёта этой схемы её следует упростить, для этого заменяют резисторы R2 и R3 одним R2,3. Получается несложная схема.

R2,3,4 = R2,3 · R4/(R2,3 + R4).

Схема становится ещё проще, в ней остаются резисторы, имеющие последовательное соединение. В более сложных ситуациях используется этот же метод преобразования.

Виды проводников

В электронной технике, при производстве проводники представляют собою тонкие полоски медной фольги. Ввиду малой длины сопротивление у них незначительно, им во многих случаях можно пренебречь. Для этих проводников сопротивление при параллельном соединении уменьшается вследствие увеличения сечения.

Большой раздел проводников представляют обмоточные провода. Они выпускаются разных диаметров — от 0,02 до 5,6 миллиметра. Для мощных трансформаторов и электродвигателей выпускаются медные шинки прямоугольного сечения. Иногда при ремонте заменяют провод большого диаметра на несколько параллельно соединённых меньшего размера.

Особый раздел проводников представляют провода и кабели, промышленность предоставляет широчайший выбор марок для самых различных нужд. Нередко приходится заменять один кабель на несколько, меньшего сечения. Причины этого бывают самые различные, например, кабель сечением 240 мм 2 очень трудно прокладывать по трассе с крутыми изгибами. Его заменяют на 2×120 мм 2 , и проблема решена.

Расчёт проводов на нагрев

Проводник нагревается протекающим током, если его температура превысит допустимую, наступает разрушение изоляции. ПУЭ предусматривает расчёт проводников на нагрев, исходными данными для него являются сила тока и условия внешней среды, в которой проложен проводник. По этим данным из таблиц в ПУЭ выбирается рекомендуемое проводника сечение (провода или кабеля).

На практике встречаются ситуации, когда нагрузка на действующий кабель сильно возросла. Существует два выхода ‒ заменить кабель на другой, это бывает дорого, или параллельно ему проложить ещё один, чтобы разгрузить основной кабель. В этом случае сопротивление проводника при параллельном соединении уменьшается, следовательно падает выделение тепла.

Чтобы правильно выбрать сечение второго кабеля, пользуются таблицами ПУЭ, важно при этом не ошибиться с определением его рабочего тока. В этой ситуации охлаждение кабелей будет даже лучше, чем у одного. Рекомендуется рассчитать сопротивление при двух кабелей, чтобы точнее определить их тепловыделение.

Расчёт проводников на потерю напряжения

При расположении потребителя R н на большом расстоянии L от источника энергии U 1 возникает довольно большое падение напряжения на проводах линии. К потребителю R н поступает напряжение U 2 значительно ниже начального U 1 . Практически в качестве нагрузки выступает различное электрооборудование, подключаемое к линии параллельно.

Для решения проблемы производят расчет сопротивления при параллельном соединении всего оборудования, так находится сопротивление нагрузки R н. Далее следует определить сопротивление проводов линии.

R л = ρ · 2L/S,

Здесь S — сечение провода линии, мм 2 .

Параллельное соединение резисторов. При параллельном соединении резисторов нескольких приемников они включаются между двумя точками электрической цепи, образуя параллельные ветви (рис. 26, а). Заменяя

лампы резисторами с сопротивлениями R1, R2, R3, получим схему, показанную на рис. 26, б.
При параллельном соединении ко всем резисторам приложено одинаковое напряжение U. Поэтому согласно закону Ома:

I 1 =U/R 1 ; I 2 =U/R 2 ; I 3 =U/R 3 .

Ток в неразветвленной части цепи согласно первому закону Кирхгофа I = I 1 +I 2 +I 3 , или

I = U / R 1 + U / R 2 + U / R 3 = U (1/R 1 + 1/R 2 + 1/R 3) = U / R эк (23)

Следовательно, эквивалентное сопротивление рассматриваемой цепи при параллельном соединении трех резисторов определяется формулой

1/R эк = 1/R 1 + 1/R 2 + 1/R 3 (24)

Вводя в формулу (24) вместо значений 1/R эк, 1/R 1 , 1/R 2 и 1/R 3 соответствующие проводимости G эк, G 1 , G 2 и G 3 , получим: эквивалентная проводимость параллельной цепи равна сумме проводимостей параллельно соединенных резисторов :

G эк = G 1 + G 2 +G 3 (25)

Таким образом, при увеличении числа параллельно включаемых резисторов результирующая проводимость электрической цепи увеличивается, а результирующее сопротивление уменьшается.
Из приведенных формул следует, что токи распределяются между параллельными ветвями обратно пропорционально их электрическим сопротивлениям или прямо пропорционально их проводимостям. Например, при трех ветвях

I 1: I 2: I 3 = 1/R 1: 1/R 2: 1/R 3 = G 1 + G 2 + G 3 (26)

В этом отношении имеет место полная аналогия между распределением токов по отдельным ветвям и распределением потоков воды по трубам.
Приведенные формулы дают возможность определить эквивалентное сопротивление цепи для различных конкретных случаев. Например, при двух параллельно включенных резисторах результирующее сопротивление цепи

R эк =R 1 R 2 /(R 1 +R 2)

при трех параллельно включенных резисторах

R эк =R 1 R 2 R 3 /(R 1 R 2 +R 2 R 3 +R 1 R 3)

При параллельном соединении нескольких, например n, резисторов с одинаковым сопротивлением R1 результирующее сопротивление цепи Rэк будет в n раз меньше сопротивления R1, т.е.

R эк = R1 / n (27)

Проходящий по каждой ветви ток I1, в этом случае будет в п раз меньше общего тока:

I1 = I / n (28)

При параллельном соединении приемников, все они находятся под одним и тем же напряжением, и режим работы каждого из них не зависит от остальных. Это означает, что ток, проходящий по какому-либо из приемников, не будет оказывать существенного влияния на другие приемники. При всяком выключении или выходе из строя любого приемника остальные приемники остаются включенными. Поэтому параллельное соединение имеет существенные преимущества перед последовательным, вследствие чего оно получило наиболее широкое распространение. В частности, электрические лампы и двигатели, предназначенные для работы при определенном (номинальном) напряжении, всегда включают параллельно.
На электровозах постоянного тока и некоторых тепловозах тяговые двигатели в процессе регулирования скорости движения нужно включать под различные напряжения, поэтому они в процессе разгона переключаются с последовательного соединения на параллельное.

Возьмем три постоянных сопротивления R1, R2 и R3 и включим их в цепь так, чтобы конец первого сопротивления R1 был соединен с началом второго сопротивления R 2, конец второго — с началом третьего R 3, а к началу первого сопротивления и к концу третьего подведем проводники от источника тока (рис. 1 ).

Такое соединение сопротивлений называется последовательным. Очевидно, что ток в такой цепи будет во всех ее точках один и тот же.

Рис 1

Как определить общее сопротивление цепи, если все включенные в нее последовательно сопротивления мы уже знаем? Используя положение, что напряжение U на зажимах источника тока равно сумме падений напряжений на участках цепи, мы можем написать:

U = U1 + U2 + U3

где

U1 = IR1 U2 = IR2 и U3 = IR3

или

IR = IR1 + IR2 + IR3

Вынеся в правой части равенства I за скобки, получим IR = I(R1 + R2 + R3) .

Поделив теперь обе части равенства на I , будем окончательно иметь R = R1 + R2 + R3

Таким образом, мы пришли к выводу, что при последовательном соединении сопротивлений общее сопротивление всей цепи равно сумме сопротивлений отдельных участков.

Проверим этот вывод на следующем примере. Возьмем три постоянных сопротивления, величины которых известны (например, R1 == 10 Ом, R 2 = 20 Ом и R 3 = 50 Ом). Соединим их последовательно (рис. 2 ) и подключим к источнику тока, ЭДС которого равна 60 В ( пренебрегаем).


Рис. 2. Пример последовательного соединения трех сопротивлений

Подсчитаем, какие показания должны дать приборы, включенные, как показано на схеме, если замкнуть цепь. Определим внешнее сопротивление цепи: R = 10 + 20 + 50 = 80 Ом.

Найдем ток в цепи : 60 / 80 = 0 ,75 А

Зная ток в цепи и сопротивления ее участков, определим падение напряжения на каждое участке цепи U 1 = 0,75х 10 = 7,5 В, U 2 = 0,75 х 20=15 В, U3 = 0,75 х 50 = 37,5 В.

Зная падение напряжений на участках, определим общее падение напряжения во внешней цепи, т. е. напряжение на зажимах источника тока U = 7,5+15 + 37,5 = 60 В.

Мы получили таким образом, что U = 60 В, т. е. несуществующее равенство ЭДС источника тока и его напряжения. Объясняется это тем, что мы пренебрегли внутренним сопротивлением источника тока.

Замкнув теперь ключ выключатель К, можно убедиться по приборам, что наши подсчеты примерно верны.

Возьмем два постоянных сопротивления R1 и R2 и соединим их так, чтобы начала этих сопротивлений были включены в одну общую точку а, а концы — в другую общую точку б. Соединив затем точки а и б с источником тока, получим замкнутую электрическую цепь. Такое соединение сопротивлений называется параллельным соединением.


Рис 3. Параллельное соединение сопротивлений

Проследим течение тока в этой цепи. От положительного полюса источника тока по соединительному проводнику ток дойдет до точки а. В точке а он разветвится, так как здесь сама цепь разветвляется на две отдельные ветви: первую ветвь с сопротивлением R1 и вторую — с сопротивлением R2. Обозначим токи в этих ветвях соответственно через I1 и I 2. Каждый из этих токов пойдет по своей ветви до точки б. В этой точке произойдет слияние токов в один общий ток, который и придет к отрицательному полюсу источника тока.

Таким образом, при параллельном соединении сопротивлений получается разветвленная цепь. Посмотрим, какое же будет соотношение между токами в составленной нами цепи.

Включим амперметр между положительным полюсом источника тока (+) и точкой а и заметим его показания. Включив затем амперметр (показанный «а рисунке пунктиром) в провод, соединяющий точку б с отрицательным полюсом источника тока (-), заметим, что прибор покажет ту же величину силы тока.

Значит, до ее разветвления (до точки а) равна силе тока после разветвления цепи (после точки б).

Будем теперь включать амперметр поочередно в каждую ветвь цепи, запоминая показания прибора. Пусть в первой ветви амперметр покажет силу тока I1 , а во второй — I 2. Сложив эти два показания амперметра, мы получим суммарный ток, по величине равный току I до разветвления (до точки а).

Следовательно, сила тока, протекающего до точки разветвления, равна сумме сил токов, утекающих от этой точки. I = I1 + I2 Выражая это формулой, получим

Это соотношение, имеющее большое практическое значение, носит название закона разветвленной цепи .

Рассмотрим теперь, каково будет соотношение между токами в ветвях.

Включим между точками а и б вольтметр и посмотрим, что он нам покажет. Во-первых, вольтметр покажет напряжение источника тока, так как он подключен, как это видно из рис. 3 , непосредственно к зажимам источника тока. Во-вторых, вольтметр покажет падения напряжений U1 и U2 на сопротивлениях R1 и R2, так как он соединен с началом и концом каждого сопротивления.

Следовательно, при параллельном соединении сопротивлений напряжение на зажимах источника тока равно падению напряжения на каждом сопротивлении.

Это дает нам право написать, что U = U1 = U2 ,

где U — напряжение на зажимах источника тока; U1 — падение напряжения на сопротивлении R1 , U2 — падение напряжения на сопротивлении R2. Вспомним, что падение напряжения на участке цепи численно равно произведению силы тока, протекающего через этот участок, на сопротивление участка U = IR .

Поэтому для каждой ветви можно написать: U1 = I1R1 и U2 = I2R2 , но так как U1 = U2, то и I1R1 = I2R2 .

Применяя к этому выражению правило пропорции, получим I1/ I2 = U2 / U1 т. е. ток в первой ветви будет во столько раз больше (или меньше) тока во второй ветви, во сколько раз сопротивление первой ветви меньше (или больше) сопротивления второй ветви.

Итак, мы пришли к важному выводу, заключающемуся в том, что при параллельном соединении сопротивлений общий ток цепи разветвляется на токи, обратно пропорциональные величинам сопротивлении параллельных ветвей. Иначе говоря, чем больше сопротивление ветви, тем меньший ток потечет через нее, и, наоборот, чем меньше сопротивление ветви, тем больший ток потечет через эту ветвь.

Убедимся в правильности этой зависимости на следующем примере. Соберем схему, состоящую из двух параллельно соединенных сопротивлений R1 и R 2, подключенных к источнику тока. Пусть R1 = 10 Ом, R2 = 20 Ом и U = 3 В.

Подсчитаем сначала, что покажет нам амперметр, включенный в каждую ветвь:

I1 = U / R1 = 3 / 10 = 0 ,3 А = 300 мА

I 2 = U / R 2 = 3 / 20 = 0,15 А = 150 мА

Общий ток в цепи I = I1 +I2 = 300 + 150 = 450 мА

Проделанный нами расчет подтверждает, что при параллельном соединении сопротивлений ток в цепи разветвляется обратно пропорционально сопротивлениям.

Действительно, R1 == 10 Ом вдвое меньше R 2 = 20 Ом, при этом I1 = 300 мА вдвое больше I2 = 150 мА. Общий ток в цепи I = 450 мА разветвился на две части так, что большая его часть (I1 = 300 мА) пошла через меньшее сопротивление (R1 = 10 Ом), а меньшая часть (R2 = 150 мА) -через большее сопротивление (R 2 = 20 Ом).

Такое разветвление тока в параллельных ветвях сходно с течением жидкости по трубам. Представьте себе трубу А, которая в каком-то месте разветвляется на две трубы Б и В различного диаметра (рис. 4). Так как диаметр трубы Б больше диаметра трубок В, то через трубу Б в одно и то же время пройдет больше воды, чем через трубу В, которая оказывает потоку воды большее сопротивление.

Рис. 4

Рассмотрим теперь, чему будет равно общее сопротивление внешней цепи, состоящей из двух параллельно соединенных сопротивлений.

Под этим общим сопротивлением внешней цепи надо понимать такое сопротивление, которым можно было бы заменить при данном напряжении цепи оба параллельно включенных сопротивления, не изменяя при этом тока до разветвления. Такое сопротивление называется эквивалентным сопротивлением.

Вернемся к цепи, показанной на рис. 3, и посмотрим, чему будет равно эквивалентное сопротивление двух параллельно соединенных сопротивлений. Применяя к этой цепи закон Ома, мы можем написать: I = U/R , где I — ток во внешней цепи (до точки разветвления), U — напряжение внешней цепи, R — сопротивление внешней цепи, т. е. эквивалентное сопротивление.

Точно так же для каждой ветви I1 = U1 / R1 , I2 = U2 / R2 , где I1 и I 2 — токи в ветвях; U1 и U2 — напряжение на ветвях; R1 и R2 — сопротивления ветвей.

По закону разветвленной цепи: I = I1 + I2

Подставляя значения токов, получим U / R = U1 / R1 + U2 / R2

Так как при параллельном соединении U = U1 = U2 , то можем написать U / R = U / R1 + U / R2

Вынеся U в правой части равенства за скобки, получим U / R = U (1 / R1 + 1 / R2 )

Разделив теперь обе части равенства на U , будем окончательно иметь 1 / R = 1 / R1 + 1 / R2

Помня, что проводимостью называется величина, обратная сопротивлению , мы можем сказать, что в полученной формуле 1 / R — проводимость внешней цепи; 1 / R1 проводимость первой ветви; 1 / R2- проводимость второй ветви.

На основании этой формулы делаем вывод: при параллельном соединении проводимость внешней цепи равна сумме проводимостей отдельных ветвей.

Следовательно, чтобы определить эквивалентное сопротивление включенных параллельно сопротивлений, надо определить проводимость цепи и взять величину, ей обратную.

Из формулы также следует, что проводимость цепи больше проводимости каждой ветви, а это значит, что эквивалентное сопротивление внешней цепи меньше наименьшего из включенных параллельно сопротивлений.

Рассматривая случай параллельного соединения сопротивлений, мы взяли наиболее простую цепь, состоящую из двух ветвей. Однако на практике могут встретиться случаи, когда цепь состоит из трех и более параллельных ветвей. Как же поступать в этих случаях?

Оказывается, все полученные нами соотношения остаются справедливыми и для цепи, состоящей из любого числа параллельно соединенных сопротивлений.

Чтобы убедиться в этом, рассмотрим следующий пример.

Возьмем три сопротивления R1 = 10 Ом, R2 = 20 Ом и R3 = 60 Ом и соединим их параллельно. Определим эквивалентное сопротивление цепи (рис. 5 ).


Рис. 5. Цепь с тремя параллельно соединенными сопротивлениями

Применяя для этой цепи формулу 1 / R = 1 / R1 + 1 / R2 , можем написать 1 / R = 1 / R1 + 1 / R2 + 1 / R3 и, подставляя известные величины, получим 1 / R = 1 / 10 + 1 / 20 + 1 / 60

Сложим эта дроби: 1/R = 10 / 60 = 1 / 6, т. е.. проводимость цепи 1 / R = 1 / 6 Следовательно, эквивалентное сопротивление R = 6 Ом.

Таким образом, эквивалентное сопротивление меньше наименьшего из включенных параллельно в цепь сопротивлений , т. е. меньше сопротивления R1.

Посмотрим теперь, действительно ли это сопротивление является эквивалентным, т. е. таким, которое могло бы заменить включенные параллельно сопротивления в 10, 20 и 60 Ом, не изменяя при этом силы тока до разветвления цепи.

Допустим, что напряжение внешней цепи, а следовательно, и напряжение на сопротивлениях R1, R2, R3 равно 12 В. Тогда сила токов в ветвях будет: I1 = U/R1 = 12 / 10 = 1 ,2 А I 2 = U/R 2 = 12 / 20 = 1 ,6 А I 3 = U/R1 = 12 / 60 = 0,2 А

Общий ток в цепи получим, пользуясь формулой I = I1 + I2 + I3 =1,2 + 0,6 + 0,2 = 2 А.

Проверим по формуле закона Ома, получится ли в цепи ток силой 2 А, если вместо трех параллельно включенных известных нам сопротивлений включено одно эквивалентное им сопротивление 6 Ом.

I = U / R = 12 / 6 = 2 А

Как видим, найденное нами сопротивление R = 6 Ом действительно является для данной цепи эквивалентным.

В этом можно убедиться и на измерительных приборах, если собрать схему с взятыми нами сопротивлениями, измерить ток во внешней цепи (до разветвления), затем заменить параллельно включенные сопротивления одним сопротивлением 6 Ом и снова измерить ток. Показания амперметра и в том и в другом случае будут примерно одинаковыми.

На практике могут встретиться также параллельные соединения, для которых рассчитать эквивалентное сопротивление можно проще, т. е. не определяя предварительно проводимостей, сразу найти сопротивление.

Например, если соединены параллельно два сопротивления R1 и R2 , то формулу 1 / R = 1 / R1 + 1 / R2 можно преобразовать так: 1/R = (R2 + R1) / R1 R2 и, решая равенство относительно R, получить R = R1 х R2 / (R1 + R2 ), т. е. при параллельном соединении двух сопротивлений эквивалентное сопротивление цепи равно произведению включенных параллельно сопротивлений, деленному на их сумму.

Последовательные и параллельные резисторы — College Physics

Большинство схем имеет более одного компонента, называемого резистором, который ограничивает поток заряда в цепи. Мера этого предела для потока заряда называется сопротивлением. Простейшие комбинации резисторов — это последовательное и параллельное соединение, показанное на (Рисунок). Общее сопротивление комбинации резисторов зависит как от их индивидуальных значений, так и от способа их подключения.

Резисторы серии

Когда резисторы включены последовательно? Резисторы включены последовательно всякий раз, когда поток заряда, называемый током, должен проходить через устройства последовательно.Например, если ток течет через человека, держащего отвертку, в землю, то на (Рисунок) (a) может быть сопротивление вала отвертки, сопротивление ее ручки, сопротивление тела человека и сопротивление ее туфли.

(рисунок) показывает резисторы, последовательно подключенные к источнику напряжения. Кажется разумным, что полное сопротивление является суммой отдельных сопротивлений, учитывая, что ток должен проходить через каждый резистор последовательно.(Этот факт был бы преимуществом для человека, желающего избежать поражения электрическим током, который мог бы уменьшить ток, надев обувь с высоким сопротивлением на резиновой подошве. Это могло бы стать недостатком, если бы одним из сопротивлений был неисправный шнур с высоким сопротивлением. прибор, уменьшающий рабочий ток.) ​​

Три резистора, подключенных последовательно к батарее (слева) и эквивалентному одиночному или последовательному сопротивлению (справа).

Чтобы убедиться, что последовательно включенные сопротивления действительно складываются, давайте рассмотрим потерю электроэнергии, называемую падением напряжения, в каждом резисторе на (Рисунок).

Согласно закону Ома падение напряжения на резисторе при протекании через него тока рассчитывается по формуле, где равно току в амперах (А), а — сопротивление в омах. Другой способ думать об этом — это напряжение, необходимое для протекания тока через сопротивление.

Таким образом, падение напряжения на нем равно, а на выходе есть. Сумма этих напряжений равна выходному напряжению источника; то есть

Это уравнение основано на сохранении энергии и сохранении заряда.Электрическая потенциальная энергия может быть описана уравнением, где — электрический заряд, а — напряжение. Таким образом, энергия, подаваемая источником, равна, а энергия, рассеиваемая резисторами, равна

.

Связи: законы сохранения

Вывод выражений для последовательного и параллельного сопротивления основан на законах сохранения энергии и сохранения заряда, которые утверждают, что общий заряд и полная энергия постоянны в любом процессе. Эти два закона непосредственно участвуют во всех электрических явлениях и будут многократно использоваться для объяснения как конкретных эффектов, так и общего поведения электричества.

Эти энергии должны быть равны, потому что в цепи нет другого источника и другого назначения для энергии. Таким образом, . Плата отменяется, уступая, как указано. (Обратите внимание, что одинаковое количество заряда проходит через батарею и каждый резистор за заданный промежуток времени, поскольку нет емкости для хранения заряда, нет места для утечки заряда и заряд сохраняется.)

Теперь подстановка значений отдельных напряжений дает

Обратите внимание, что для эквивалентного одиночного последовательного сопротивления мы имеем

Это означает, что полное или эквивалентное последовательное сопротивление трех резисторов составляет.

Эта логика действительна в общем для любого количества резисторов, включенных последовательно; таким образом, полное сопротивление последовательного соединения составляет

, как предлагается. Поскольку весь ток должен проходить через каждый резистор, он испытывает сопротивление каждого, а последовательно соединенные сопротивления просто складываются.

Расчет сопротивления, тока, падения напряжения и рассеиваемой мощности: анализ последовательной цепи

Предположим, что выходное напряжение батареи на (Рисунок) равно, а сопротивления равны, и.а) Каково полное сопротивление? (б) Найдите ток. (c) Рассчитайте падение напряжения на каждом резисторе и покажите, как они складываются, чтобы равняться выходному напряжению источника. (d) Рассчитайте мощность, рассеиваемую каждым резистором. (e) Найдите выходную мощность источника и покажите, что она равна общей мощности, рассеиваемой резисторами.

Стратегия и решение для (а)

Общее сопротивление — это просто сумма отдельных сопротивлений, определяемая следующим уравнением:

Стратегия и решение для (b)

Ток определяется по закону Ома.Ввод значения приложенного напряжения и общего сопротивления дает ток для цепи:

Стратегия и решение для (c)

Напряжение — или падение — на резисторе определяется законом Ома. Ввод значения тока и значения первого сопротивления дает

.

Аналогично

и

Обсуждение для (c)

Три капли добавляют к, как и прогнозировалось:

Стратегия и решение для (d)

Самый простой способ рассчитать мощность в ваттах (Вт), рассеиваемую резистором в цепи постоянного тока, — это использовать закон Джоуля, где — электрическая мощность.В этом случае через каждый резистор протекает одинаковый полный ток. Подставляя закон Ома в закон Джоуля, мы получаем мощность, рассеиваемую первым резистором, как

Аналогично

и

Обсуждение для (d)

Мощность также можно рассчитать с помощью или, где — падение напряжения на резисторе (а не полное напряжение источника). Будут получены те же значения.

Стратегия и решение для (e)

Самый простой способ рассчитать выходную мощность источника — использовать, где — напряжение источника.Это дает

Обсуждение для (e)

По совпадению обратите внимание, что общая мощность, рассеиваемая резисторами, также составляет 7,20 Вт, что соответствует мощности, выдаваемой источником. То есть

Мощность — это энергия в единицу времени (ватты), поэтому для сохранения энергии требуется, чтобы выходная мощность источника была равна общей мощности, рассеиваемой резисторами.

Основные характеристики резисторов серии

  1. Сопротивления серии добавить:
  2. Одинаковый ток протекает последовательно через каждый резистор.
  3. Отдельные последовательно включенные резисторы не получают полное напряжение источника, а делят его.

Параллельные резисторы

(рисунок) показывает резисторы, включенные параллельно, подключенные к источнику напряжения. Резисторы включены параллельно, когда каждый резистор подключен непосредственно к источнику напряжения с помощью соединительных проводов с незначительным сопротивлением. Таким образом, к каждому резистору приложено полное напряжение источника.

Каждый резистор потребляет такой же ток, как если бы он один был подключен к источнику напряжения (при условии, что источник напряжения не перегружен).Например, автомобильные фары, радио и т. Д. Подключены параллельно, так что они используют полное напряжение источника и могут работать полностью независимо. То же самое и в вашем доме, или в любом другом здании. (См. (Рисунок) (b).)

(a) Три резистора, подключенных параллельно батарее, и эквивалентное одиночное или параллельное сопротивление. (б) Электроснабжение в доме. (Источник: Dmitry G, Wikimedia Commons)

Чтобы найти выражение для эквивалентного параллельного сопротивления, давайте рассмотрим протекающие токи и их связь с сопротивлением.Поскольку каждый резистор в цепи имеет полное напряжение, токи, протекающие через отдельные резисторы, равны, и. Сохранение заряда подразумевает, что полный ток, производимый источником, является суммой этих токов:

Подстановка выражений для отдельных токов дает

Обратите внимание, что закон Ома для эквивалентного одиночного сопротивления дает

Члены в скобках в последних двух уравнениях должны быть равны. Обобщая для любого количества резисторов, общее сопротивление параллельного соединения связано с отдельными сопротивлениями на

.

Это соотношение приводит к общему сопротивлению, которое меньше наименьшего из отдельных сопротивлений.(Это видно в следующем примере.) При параллельном подключении резисторов от источника течет больше тока, чем протекает по любому из них по отдельности, поэтому общее сопротивление ниже.

Расчет сопротивления, тока, рассеиваемой мощности и выходной мощности: анализ параллельной цепи

Пусть выходное напряжение батареи и сопротивления при параллельном соединении на (Рисунок) будут такими же, как и в ранее рассмотренном последовательном соединении:,, и. а) Каково полное сопротивление? (б) Найдите полный ток.(c) Рассчитайте токи в каждом резисторе и покажите, как они складываются, чтобы равняться общему выходному току источника. (d) Рассчитайте мощность, рассеиваемую каждым резистором. (e) Найдите выходную мощность источника и покажите, что она равна общей мощности, рассеиваемой резисторами.

Стратегия и решение для (а)

Общее сопротивление для параллельной комбинации резисторов находится с помощью следующего уравнения. Ввод известных значений дает

Таким образом,

(Обратите внимание, что в этих расчетах каждый промежуточный ответ отображается с дополнительной цифрой.)

Мы должны инвертировать это, чтобы найти полное сопротивление. Это дает

Суммарное сопротивление с правильным количеством значащих цифр —

.

Обсуждение для (а)

, как и предполагалось, меньше наименьшего индивидуального сопротивления.

Стратегия и решение для (b)

Полный ток можно найти из закона Ома, заменив полное сопротивление. Это дает

Обсуждение для (б)

Ток для каждого устройства намного больше, чем для тех же устройств, подключенных последовательно (см. Предыдущий пример).Схема с параллельным соединением имеет меньшее общее сопротивление, чем резисторы, включенные последовательно.

Стратегия и решение для (c)

Отдельные токи легко вычислить по закону Ома, поскольку каждый резистор получает полное напряжение. Таким образом,

Аналогично

и

Обсуждение для (c)

Общий ток складывается из отдельных токов:

Это соответствует сохранению заряда.

Стратегия и решение для (d)

Мощность, рассеиваемую каждым резистором, можно найти с помощью любого из уравнений, связывающих мощность с током, напряжением и сопротивлением, поскольку все три известны.Давайте использовать, так как каждый резистор получает полное напряжение. Таким образом,

Аналогично

и

Обсуждение для (d)

Мощность, рассеиваемая каждым резистором параллельно, значительно выше, чем при последовательном подключении к тому же источнику напряжения.

Стратегия и решение для (e)

Общую мощность также можно рассчитать несколькими способами. Выбор и ввод полного тока дает

Обсуждение для (e)

Суммарная мощность, рассеиваемая резисторами, также 179 Вт:

Это соответствует закону сохранения энергии.

Общее обсуждение

Обратите внимание, что как токи, так и мощность при параллельном подключении больше, чем для тех же устройств, подключенных последовательно.

Основные характеристики параллельных резисторов

  1. Параллельное сопротивление получается из любого отдельного сопротивления в комбинации, и оно меньше.
  2. Каждый резистор, включенный параллельно, имеет одинаковое полное напряжение источника. (В системах распределения электроэнергии чаще всего используются параллельные соединения для питания бесчисленных устройств, обслуживаемых одним и тем же напряжением, и для того, чтобы они могли работать независимо.)
  3. Не каждый параллельный резистор получает полный ток; они делят это.

Сочетания последовательного и параллельного

Более сложные соединения резисторов иногда представляют собой просто комбинации последовательного и параллельного. Они часто встречаются, особенно если учитывать сопротивление провода. В этом случае сопротивление провода включено последовательно с другими сопротивлениями, включенными параллельно.

Комбинации последовательного и параллельного соединения можно уменьшить до одного эквивалентного сопротивления, используя метод, показанный на (Рисунок).Различные части идентифицируются как последовательные или параллельные, уменьшаются до их эквивалентов и далее уменьшаются до тех пор, пока не останется единственное сопротивление. Процесс более трудоемкий, чем трудный.

Эта комбинация из семи резисторов имеет как последовательные, так и параллельные части. Каждое из них идентифицируется и приводится к эквивалентному сопротивлению, а затем уменьшается до достижения единого эквивалентного сопротивления.

Простейшая комбинация последовательного и параллельного сопротивления, показанная на (Рисунок), также является наиболее поучительной, поскольку она используется во многих приложениях.Например, это может быть сопротивление проводов от автомобильного аккумулятора к его электрическим устройствам, которые включены параллельно. и мог быть стартером и светом салона. Ранее мы предполагали, что сопротивление провода незначительно, но, когда это не так, оно имеет важные последствия, как показывает следующий пример.

Расчет сопротивления, падения, тока и рассеиваемой мощности: объединение последовательных и параллельных цепей

(рисунок) показывает резисторы из двух предыдущих примеров, подключенные другим способом — комбинацией последовательного и параллельного.Мы можем считать сопротивление проводов, ведущих к и. (а) Найдите полное сопротивление. б) Что такое падение? (c) Найдите сквозной ток. (г) Какая мощность рассеивается?

Эти три резистора подключены к источнику напряжения так, что и включены параллельно друг другу, а эта комбинация — последовательно с.

Стратегия и решение для (а)

Чтобы найти полное сопротивление, отметим, что и находятся параллельно, а их комбинация — последовательно с.Таким образом, полное (эквивалентное) сопротивление этой комбинации составляет

.

Сначала мы находим, используя уравнение для параллельных резисторов и вводя известные значения:

Инвертирование дает

Таким образом, общее сопротивление равно

.

Обсуждение для (а)

Общее сопротивление этой комбинации является промежуточным между значениями чисто последовательного и чистого параллельного (и, соответственно), найденными для тех же резисторов в двух предыдущих примерах.

Стратегия и решение для (b)

Чтобы найти падение, отметим, что через него протекает полный ток.Таким образом, его падение составляет

.

Мы должны найти, прежде чем сможем вычислить. Полный ток определяется по закону Ома для цепи. То есть

Вводя это в выражение выше, мы получаем

Обсуждение для (б)

Напряжение, приложенное к общему напряжению, и меньше его на величину. Когда сопротивление провода велико, это может существенно повлиять на работу устройств, представленных и.

Стратегия и решение для (c)

Чтобы найти сквозной ток, мы должны сначала найти приложенное к нему напряжение.Мы называем это напряжением, потому что оно приложено к параллельной комбинации резисторов. Напряжение, приложенное к обоим и уменьшается на величину, так что оно составляет

.

Теперь ток через сопротивление определяется по закону Ома:

Обсуждение для (c)

Ток меньше, чем 2,00 А, которые протекали, когда он был подключен параллельно к батарее в предыдущем примере параллельной цепи.

Стратегия и решение для (d)

Рассеиваемая мощность равна

Обсуждение для (d)

Мощность меньше 24.0 Вт этот резистор рассеивает при параллельном подключении к источнику 12,0 В.

Практическое применение

Одним из следствий этого последнего примера является то, что сопротивление в проводах снижает ток и мощность, подаваемую на резистор. Если сопротивление провода относительно велико, как в изношенном (или очень длинном) удлинителе, то эти потери могут быть значительными. Если протекает большой ток, провал в проводах также может быть значительным.

Например, когда вы роетесь в холодильнике и включается мотор, свет холодильника на мгновение гаснет.Точно так же вы можете увидеть тусклый свет в салоне, когда вы запускаете двигатель вашего автомобиля (хотя это может быть связано с сопротивлением внутри самой батареи).

Что происходит в этих сильноточных ситуациях, показано на (Рисунок). Устройство, представленное значком, имеет очень низкое сопротивление, поэтому при его включении протекает большой ток. Этот увеличенный ток вызывает большее падение в проводах, представленных значком, уменьшая напряжение на лампочке (которая есть), которое затем заметно гаснет.

Почему гаснет свет при включении большого прибора? Ответ заключается в том, что большой ток, потребляемый двигателем прибора, вызывает значительное падение напряжения в проводах и снижает напряжение на свету.

Учебное пособие по физике: два типа соединений

Когда в цепи с источником энергии присутствуют два или более электрических устройства, существует несколько основных способов их соединения. Их можно соединить последовательно или соединить параллельно .Предположим, что в одну цепь включены три лампочки. При последовательном соединении они соединяются таким образом, чтобы отдельный заряд проходил через каждую из лампочек последовательно. При последовательном соединении заряд проходит через каждую лампочку. При параллельном подключении один заряд, проходящий через внешнюю цепь, будет проходить только через одну из лампочек. Лампочки помещаются в отдельную ветвь, и заряд, проходящий через внешнюю цепь, проходит только через одну из ветвей на обратном пути к клемме с низким потенциалом.Способы подключения резисторов будут иметь большое влияние на общее сопротивление цепи, общий ток в цепи и ток в каждом резисторе. В Уроке 4 мы исследуем влияние типа подключения на общий ток и сопротивление цепи.

Обычная физическая лаборатория состоит в построении обоих типов цепей с лампочками, соединенными последовательно, и лампочками, соединенными параллельно. Эти две схемы сравниваются и противопоставляются.

Основные вопросы, вызывающие беспокойство при такой лабораторной деятельности, как правило, следующие:

  • Что происходит с общим током в цепи по мере увеличения количества резисторов (лампочек)?
  • Что происходит с общим сопротивлением в цепи при увеличении количества резисторов (лампочек)?
  • Если один из резисторов выключится (т.е. лампочка погаснет, ), что произойдет с другими резисторами (лампочками) в цепи? Они остаются на (т.е., лит)?

Исследование последовательных подключений

При проведении лабораторных работ для двух типов цепей производятся совершенно разные наблюдения. Последовательная цепь может быть построена путем соединения лампочек таким образом, чтобы оставался единственный путь для потока заряда; луковицы добавляются к той же линии без точки ветвления. По мере того, как добавляется все больше и больше лампочек, яркость каждой лампочки постепенно уменьшается.Это наблюдение является индикатором того, что ток в цепи уменьшается.

Итак, для последовательных цепей по мере добавления резисторов общий ток в цепи уменьшается. Это уменьшение тока согласуется с выводом о том, что общее сопротивление увеличивается.

Последнее наблюдение, которое является уникальным для последовательных цепей, — это эффект вынимания лампы из розетки. Если одна из трех лампочек в последовательной цепи вывинчивается из патрона, то наблюдается, что остальные лампочки сразу же гаснут.Чтобы устройства в последовательной цепи работали, каждое устройство должно работать. Если один погаснет, погаснут все. Предположим, что вся бытовая техника на домашней кухне подключена последовательно. Чтобы холодильник работал на этой кухне, должны быть включены тостер, посудомоечная машина, мусоропровод и верхний свет. Чтобы одно устройство, включенное последовательно, работало, все они должны работать. Если ток равен , отрежьте от любого из них, он отключается от всех из них. Совершенно очевидно, что приборы на кухне не подключены последовательно.

Исследование параллельных подключений

Используя тот же набор проводов, D-элементов и лампочек, можно таким же образом исследовать параллельные цепи. Можно исследовать влияние количества резисторов на общий ток и общее сопротивление. На схемах ниже изображены обычные способы построения схемы с параллельным подключением лампочек. Следует отметить, что исследование общего тока для параллельных соединений требует добавления индикаторной лампы .Лампа индикатора размещена вне ответвлений и позволяет наблюдать влияние дополнительных резисторов на общий ток. Лампочки, размещенные в параллельных ветвях, показывают только ток, протекающий через эту конкретную ветвь. Поэтому, исследуя влияние количества резисторов на общий ток и сопротивление, нужно внимательно следить за лампочкой индикатора, а не за лампочками, помещенными в ответвления. На диаграмме ниже показаны типичные наблюдения.

Из показаний лампочек индикаторов на приведенных выше схемах видно, что добавление большего количества резисторов приводит к тому, что лампочка индикатора становится ярче. Для параллельных цепей с увеличением количества резисторов общий ток также увеличивается. Это увеличение тока согласуется с уменьшением общего сопротивления. Добавление резисторов в отдельную ветвь приводит к неожиданному результату уменьшения общего сопротивления!

Если отдельная лампочка в параллельной ветви вывинчивается из патрона, то ток в общей цепи и в других ветвях все равно остается.Удаление третьей лампочки из патрона приводит к преобразованию схемы из параллельной цепи с тремя лампами в параллельную цепь с двумя лампами. Если бы приборы на домашней кухне были подключены параллельно, то холодильник мог бы работать без включения посудомоечной машины, тостера, мусоропровода и верхнего освещения. Одно устройство может работать без включения других. Поскольку каждое устройство находится в своей отдельной ветви, отключение этого устройства просто прекращает подачу заряда в эту ветвь.По другим ответвлениям к другим приборам по-прежнему будет поступать заряд. Совершенно очевидно, что бытовая техника в доме подключена параллельно.

Аналогия с платной станции

Эффект добавления резисторов совершенно иной, если они добавляются параллельно, по сравнению с их последовательным соединением. Последовательное добавление большего количества резисторов означает увеличение общего сопротивления; однако добавление большего количества резисторов параллельно означает уменьшение общего сопротивления.Тот факт, что можно добавить больше резисторов параллельно и добиться меньшего сопротивления, многих очень беспокоит. Аналогия может помочь прояснить причину этой изначально надоедливой правды.

Поток заряда по проводам цепи можно сравнить с потоком автомобилей по платной дороге в очень многолюдном мегаполисе. Основными источниками сопротивления на платных дорогах являются посты. Остановка автомобилей и принуждение их к уплате дорожных сборов не только замедляет движение автомобилей, но и в зоне с интенсивным движением транспорта также может вызвать узкое место с резервной копией на многие мили.Скорость, с которой автомобили проезжают через точку на этой платной системе, значительно снижается из-за наличия платы за проезд. Понятно, что пункты пропуска дороги — это главный фактор, препятствующий потоку автомобилей.

Теперь предположим, что в попытке увеличить скорость потока Управление взимания платы за проезд решает добавить еще две точки взимания платы за проезд на определенной станции взимания платы, где узкое место создает проблемы для путешественников. Они рассматривают два возможных способа подключения своих платных постов — последовательно или параллельно. При добавлении платных постов (т.е., резисторы) последовательно, они добавляли бы их таким образом, чтобы каждая машина, движущаяся по шоссе, должна была бы последовательно останавливаться на каждой плате за проезд. При наличии только одного пути через пункты взимания платы за проезд каждая машина должна будет останавливаться и платить за проезд в каждой будке. Вместо того, чтобы платить 60 центов один раз в одной будке, теперь им придется платить по 20 центов трижды в каждой из трех платных. Совершенно очевидно, что добавление платных постов последовательно имело бы общий эффект увеличения общего сопротивления и уменьшения общей скорости потока автомобилей (т.э., ток).

Другим способом добавления двух дополнительных пунктов взимания платы за проезд на этом конкретном пункте сбора платы за проезд является добавление пунктов взимания платы за проезд параллельно. Каждую будку можно разместить в отдельном филиале. Автомобили, движущиеся по платной дороге, останавливались только у одной из трех будок. У автомобилей будет три возможных пути, по которым они будут проезжать через станцию ​​сбора платы за проезд, и каждая машина выберет только один из путей. Совершенно очевидно, что параллельное добавление платных постов будет иметь общий эффект уменьшения общего сопротивления и увеличения общей скорости потока автомобилей (т.е., ток) по платной дороге. Как и в случае добавления дополнительных электрических резисторов параллельно, добавление дополнительных плат в параллельных ветвях создает меньшее общее сопротивление. Обеспечивая большее количество путей (то есть ответвлений), по которым заряд и автомобили могут проходить через узкие места, скорость потока может быть увеличена.

Мы хотели бы предложить … Зачем просто читать об этом и когда можно с этим взаимодействовать? Взаимодействие — это именно то, что вы делаете, когда используете одну из интерактивных функций The Physics Classroom.Мы хотели бы предложить вам совместить чтение этой страницы с использованием нашего интерактивного средства построения цепей постоянного тока. Вы можете найти его в разделе Physics Interactives на нашем сайте. Построитель цепей постоянного тока предоставляет учащемуся набор для построения виртуальных цепей. Вы можете легко перетащить источники напряжения, резисторы и провода на рабочее место, расположить и подключить их так, как вам нужно. Вольтметры и амперметры позволяют измерять ток и падение напряжения. Нажатие на резистор или источник напряжения позволяет изменять сопротивление или входное напряжение.Это просто. Это весело. И это безопасно (если вы не используете его в ванне).


Проверьте свое понимание

1. Обратите внимание на электрическую проводку, указанную ниже. Укажите, являются ли соединения последовательными или параллельными. Объясните каждый выбор.

2. Ниже показаны две электрические цепи. Для каждой цепи укажите, какие два устройства подключены последовательно, а какие — параллельно.

Последовательно? ___________________

Параллельно? _________________

Последовательно? ___________________

Параллельно? _________________

Последовательные и параллельные резисторы

Цели обучения

К концу этого раздела вы сможете:

  • Нарисуйте цепь с резисторами, включенными параллельно и последовательно.
  • Рассчитайте падение напряжения тока на резисторе, используя закон Ома.
  • Сравните способ расчета общего сопротивления для резисторов, включенных последовательно и параллельно.
  • Объясните, почему полное сопротивление параллельной цепи меньше наименьшего сопротивления любого из резисторов в этой цепи.
  • Вычислите общее сопротивление цепи, которая содержит смесь резисторов, включенных последовательно и параллельно.

Большинство схем имеет более одного компонента, называемого резистором , который ограничивает поток заряда в цепи.Мера этого предела расхода заряда называется сопротивлением . Простейшие комбинации резисторов — это последовательное и параллельное соединение, показанное на рисунке 1. Общее сопротивление комбинации резисторов зависит как от их индивидуальных значений, так и от способа их подключения.

Рис. 1. (a) Последовательное соединение резисторов. (б) Параллельное соединение резисторов.

Когда резисторы в серии ? Резисторы включены последовательно всякий раз, когда поток заряда, называемый током , должен проходить через устройства последовательно.Например, если ток течет через человека, держащего отвертку, в землю, тогда R 1 на Рисунке 1 (a) может быть сопротивлением вала отвертки, R 2 сопротивлением ее ручки , R 3 сопротивление тела человека и R 4 сопротивление его обуви. На рисунке 2 показаны резисторы, последовательно подключенные к источнику напряжения . Кажется разумным, что полное сопротивление является суммой отдельных сопротивлений, учитывая, что ток должен проходить через каждый резистор последовательно.(Этот факт был бы преимуществом для человека, желающего избежать поражения электрическим током, который мог бы уменьшить ток, надев обувь с высоким сопротивлением на резиновой подошве. Это могло бы стать недостатком, если бы одним из сопротивлений был неисправный шнур с высоким сопротивлением. прибор, уменьшающий рабочий ток.) ​​

Рис. 2. Три резистора, подключенных последовательно к батарее (слева), и эквивалентное одиночное или последовательное сопротивление (справа).

Чтобы убедиться, что последовательно включенные сопротивления действительно складываются, давайте рассмотрим потерю электроэнергии, называемую падением напряжения , в каждом резисторе на Рисунке 2.Согласно закону Ома , падение напряжения В на резисторе при протекании через него тока рассчитывается по формуле В = IR , где I равно току в амперах (A) и R — сопротивление в Ом (Ом). Другой способ представить это: В, — это напряжение, необходимое для протекания тока I через сопротивление R . Таким образом, падение напряжения на R 1 составляет В 1 = IR 1 , что на R 2 составляет В 2 = IR 2 и 2 что для R 3 составляет V 3 = IR 3 .Сумма этих напряжений равна выходному напряжению источника; то есть

В = В 1 + В 2 + В 3 .

Это уравнение основано на сохранении энергии и сохранении заряда. Электрическая потенциальная энергия может быть описана уравнением PE = qV , где q — электрический заряд, а V — напряжение. Таким образом, энергия, подаваемая источником, составляет кв.кв. , а энергия, рассеиваемая резисторами, составляет

.

qV 1 + qV 2 + qV 3 .

Установление связей: законы сохранения

Вывод выражений для последовательного и параллельного сопротивления основан на законах сохранения энергии и сохранения заряда, которые утверждают, что общий заряд и полная энергия постоянны в любом процессе. Эти два закона непосредственно участвуют во всех электрических явлениях и будут многократно использоваться для объяснения как конкретных эффектов, так и общего поведения электричества.

Эти энергии должны быть равны, потому что в цепи нет другого источника и другого назначения для энергии.Таким образом, qV = qV 1 + qV 2 + qV 3 . Заряд q отменяется, давая V = V 1 + V 2 + V 3 , как указано. (Обратите внимание, что одинаковое количество заряда проходит через батарею и каждый резистор за заданный промежуток времени, поскольку нет емкости для хранения заряда, нет места для утечки заряда и заряд сохраняется.) Теперь подстановка значений для отдельных напряжений дает

V = IR 1 + IR 2 + IR 3 = I ( R 1 + R 3 902 902 902 ).

Обратите внимание, что для эквивалентного сопротивления одной серии R с , мы имеем

В = ИК с .

Это означает, что полное или эквивалентное последовательное сопротивление R с трех резисторов составляет R с = R 1 + R 2 + R 3 .Эта логика действительна в общем для любого количества резисторов, включенных последовательно; таким образом, полное сопротивление R с последовательного соединения равно

R с = R 1 + R 2 + R 3 +…,

, как предлагается. Поскольку весь ток должен проходить через каждый резистор, он испытывает сопротивление каждого, а последовательно соединенные сопротивления просто складываются.

Пример 1. Расчет сопротивления, тока, падения напряжения и рассеиваемой мощности: анализ последовательной цепи

Предположим, что выходное напряжение батареи на рисунке 2 равно 12.0 В, а сопротивления равны R 1 = 1,00 Ом, R 2 = 6,00 Ом и R 3 = 13,0 Ом. а) Каково полное сопротивление? (б) Найдите ток. (c) Рассчитайте падение напряжения на каждом резисторе и покажите, как они складываются, чтобы равняться выходному напряжению источника. (d) Рассчитайте мощность, рассеиваемую каждым резистором. (e) Найдите выходную мощность источника и покажите, что она равна общей мощности, рассеиваемой резисторами.

Стратегия и решение для (а)

Общее сопротивление — это просто сумма отдельных сопротивлений, определяемая следующим уравнением:

[латекс] \ begin {array} {lll} {R} _ {\ text {s}} & = & {R} _ {1} + {R} _ {2} + {R} _ {3} \ \ & = & 1.00 \ text {} \ Omega + 6.00 \ text {} \ Omega + 13.0 \ text {} \ Omega \\ & = & 20.0 \ text {} \ Omega \ end {array} \\ [/ latex].

Стратегия и решение для (b)

Ток определяется по закону Ома, В = IR . Ввод значения приложенного напряжения и общего сопротивления дает ток для цепи:

[латекс] I = \ frac {V} {{R} _ {\ text {s}}} = \ frac {12.0 \ text {V}} {20.0 \ text {} \ Omega} = 0.60 \ text {A }\\[/латекс].

Стратегия и решение для (c)

Напряжение — или падение IR — на резисторе определяется законом Ома.Ввод значения тока и значения первого сопротивления дает

.

В 1 = IR 1 = (0,600A) (1,0 Ом) = 0,600 В.

Аналогично

В 2 = IR 2 = (0,600A) (6,0 Ом) = 3,60 В

и

V3 = IR 3 = (0,600 A) (13,0 Ом) = 7,80 В.

Обсуждение для (c)

Три капли IR добавляют к 12.0 В, прогноз:

V 1 + V 2 + V 3 = (0,600 + 3,60 + 7,80) V = 12,0 В.

Стратегия и решение для (d)

Самый простой способ рассчитать мощность в ваттах (Вт), рассеиваемую резистором в цепи постоянного тока, — это использовать закон Джоуля , P = IV , где P — электрическая мощность. В этом случае через каждый резистор протекает одинаковый полный ток.Подставляя закон Ома V = IR в закон Джоуля, мы получаем мощность, рассеиваемую первым резистором, как

P 1 = I 2 R 1 = (0,600 A) 2 (1,00 Ом) = 0,360 Вт

Аналогично

P 2 = I 2 R 2 = (0,600 A) 2 (6,00 Ом) = 2,16 Вт

и

P 3 = I 2 R 3 = (0.{2}} {R} \\ [/ latex], где В, — это падение напряжения на резисторе (а не полное напряжение источника). Будут получены те же значения.

Стратегия и решение для (e)

Самый простой способ рассчитать выходную мощность источника — использовать P = IV , где В, — напряжение источника. Это дает

P = (0,600 A) (12,0 В) = 7,20 Вт.

Обсуждение для (e)

Обратите внимание, что по совпадению общая мощность, рассеиваемая резисторами, также равна 7.20 Вт, столько же, сколько мощность, выдаваемая источником. То есть

P 1 + P 2 + P 3 = (0,360 + 2,16 + 4,68) W = 7,20 Вт

Мощность — это энергия в единицу времени (ватты), поэтому для сохранения энергии требуется, чтобы выходная мощность источника была равна общей мощности, рассеиваемой резисторами.

Основные характеристики резисторов серии

  1. Последовательные сопротивления добавить: R с = R 1 + R 2 + R 3 +….
  2. Одинаковый ток протекает последовательно через каждый резистор.
  3. Отдельные последовательно включенные резисторы не получают полное напряжение источника, а делят его.

На рисунке 3 показаны резисторы параллельно , подключенные к источнику напряжения. Резисторы включены параллельно, когда каждый резистор подключен непосредственно к источнику напряжения с помощью соединительных проводов с незначительным сопротивлением. Таким образом, к каждому резистору приложено полное напряжение источника. Каждый резистор потребляет такой же ток, как если бы он один был подключен к источнику напряжения (при условии, что источник напряжения не перегружен).Например, автомобильные фары, радио и т. Д. Подключены параллельно, так что они используют полное напряжение источника и могут работать полностью независимо. То же самое и в вашем доме, или в любом другом здании. (См. Рисунок 3 (b).)

Рис. 3. (a) Три резистора, подключенных параллельно батарее, и эквивалентное одиночное или параллельное сопротивление. (б) Электроснабжение в доме. (Источник: Dmitry G, Wikimedia Commons)

Чтобы найти выражение для эквивалентного параллельного сопротивления R p , давайте рассмотрим протекающие токи и их связь с сопротивлением.Поскольку каждый резистор в цепи имеет полное напряжение, токи, протекающие через отдельные резисторы, равны [латекс] {I} _ {1} = \ frac {V} {{R} _ {1}} \\ [/ latex] , [латекс] {I} _ {2} = \ frac {V} {{R} _ {2}} \\ [/ latex] и [латекс] {I} _ {3} = \ frac {V} {{R} _ {3}} \\ [/ латекс]. Сохранение заряда подразумевает, что полный ток I , производимый источником, является суммой этих токов:

I = I 1 + I 2 + I 3 .

Подстановка выражений для отдельных токов дает

[латекс] I = \ frac {V} {{R} _ {1}} + \ frac {V} {{R} _ {2}} + \ frac {V} {{R} _ {3}} = V \ left (\ frac {1} {{R} _ {1}} + \ frac {1} {{R} _ {2}} + \ frac {1} {{R} _ {3}} \ справа) \\ [/ латекс].

Обратите внимание, что закон Ома для эквивалентного одиночного сопротивления дает

[латекс] I = \ frac {V} {{R} _ {p}} = V \ left (\ frac {1} {{R} _ {p}} \ right) \\ [/ latex].

Члены в скобках в последних двух уравнениях должны быть равны. Обобщая для любого количества резисторов, общее сопротивление R p параллельного соединения связано с отдельными сопротивлениями на

[латекс] \ frac {1} {{R} _ {p}} = \ frac {1} {{R} _ {1}} + \ frac {1} {{R} _ {2}} + \ гидроразрыв {1} {{R} _ {\ text {.} 3}} + \ text {.} \ Text {…} \\ [/ latex]

Это соотношение приводит к общему сопротивлению R p , которое меньше наименьшего из отдельных сопротивлений. (Это видно в следующем примере.) При параллельном подключении резисторов от источника течет больше тока, чем протекает по любому из них по отдельности, поэтому общее сопротивление ниже.

Пример 2. Расчет сопротивления, тока, рассеиваемой мощности и выходной мощности: анализ параллельной цепи

Пусть выходное напряжение батареи и сопротивления в параллельном соединении на Рисунке 3 будут такими же, как в ранее рассмотренном последовательном соединении: В = 12.0 В, R 1 = 1,00 Ом, R 2 = 6,00 Ом и R 3 = 13,0 Ом. а) Каково полное сопротивление? (б) Найдите полный ток. (c) Рассчитайте токи в каждом резисторе и покажите, как они складываются, чтобы равняться общему выходному току источника. (d) Рассчитайте мощность, рассеиваемую каждым резистором. (e) Найдите выходную мощность источника и покажите, что она равна общей мощности, рассеиваемой резисторами.

Стратегия и решение для (а)

Общее сопротивление для параллельной комбинации резисторов находится с помощью следующего уравнения.Ввод известных значений дает

[латекс] \ frac {1} {{R} _ {p}} = \ frac {1} {{R} _ {1}} + \ frac {1} {{R} _ {2}} + \ frac {1} {{R} _ {3}} = \ frac {1} {1 \ text {.} \ text {00} \ text {} \ Omega} + \ frac {1} {6 \ text {. } \ text {00} \ text {} \ Omega} + \ frac {1} {\ text {13} \ text {.} 0 \ text {} \ Omega} \\ [/ latex].

Таким образом,

[латекс] \ frac {1} {{R} _ {p}} = \ frac {1.00} {\ text {} \ Omega} + \ frac {0 \ text {.} \ Text {1667}} {\ текст {} \ Omega} + \ frac {0 \ text {.} \ text {07692}} {\ text {} \ Omega} = \ frac {1 \ text {.} \ text {2436}} {\ text { } \ Omega} \\ [/ латекс].

(Обратите внимание, что в этих расчетах каждый промежуточный ответ отображается с дополнительной цифрой.) Мы должны перевернуть это, чтобы найти полное сопротивление R p . Это дает

[латекс] {R} _ {\ text {p}} = \ frac {1} {1 \ text {.} \ Text {2436}} \ text {} \ Omega = 0 \ text {.} \ Text { 8041} \ text {} \ Omega \\ [/ latex].

Общее сопротивление с правильным количеством значащих цифр составляет R p = 0,804 Ом

Обсуждение для (а)

R p , как и предполагалось, меньше наименьшего индивидуального сопротивления.

Стратегия и решение для (b)

Полный ток можно найти из закона Ома, заменив полное сопротивление R p . Это дает

[латекс] I = \ frac {V} {{R} _ {\ text {p}}} = \ frac {\ text {12.0 V}} {0.8041 \ text {} \ Omega} = \ text {14} \ text {.} \ text {92 A} \\ [/ latex].

Обсуждение для (б)

Ток I для каждого устройства намного больше, чем для тех же устройств, подключенных последовательно (см. Предыдущий пример).Схема с параллельным соединением имеет меньшее общее сопротивление, чем резисторы, включенные последовательно.

Стратегия и решение для (c)

Отдельные токи легко вычислить по закону Ома, поскольку каждый резистор получает полное напряжение. Таким образом,

[латекс] {I} _ {1} = \ frac {V} {{R} _ {1}} = \ frac {12.0 \ text {V}} {1.00 \ text {} \ Omega} = 12.0 \ text {A} \\ [/ латекс].

Аналогично

[латекс] {I} _ {2} = \ frac {V} {{R} _ {2}} = \ frac {12.0 \ text {V}} {6.00 \ text {} \ Omega} = 2 \ text {.} \ text {00} \ text {A} \\ [/ latex]

и

[латекс] {I} _ {3} = \ frac {V} {{R} _ {3}} = \ frac {\ text {12} \ text {.} 0 \ text {V}} {\ text {13} \ text {.} \ Text {0} \ text {} \ Omega} = 0 \ text {.} \ Text {92} \ text {A} \\ [/ latex].

Обсуждение для (c)

Общий ток складывается из отдельных токов:

I 1 + I 2 + I 3 = 14,92 A.

Это соответствует сохранению заряда.{2}} {13.0 \ text {} \ Omega} = 11.1 \ text {W} \\ [/ latex].

Обсуждение для (d)

Мощность, рассеиваемая каждым резистором параллельно, значительно выше, чем при последовательном подключении к тому же источнику напряжения.

Стратегия и решение для (e)

Общую мощность также можно рассчитать несколькими способами. Выбрав P = IV и введя полный ток, получим

P = IV = (14,92 A) (12,0 В) = 179 Вт.

Обсуждение для (e)

Суммарная мощность, рассеиваемая резисторами, также 179 Вт:

P 1 + P 2 + P 3 = 144 Вт + 24,0 Вт + 11,1 Вт = 179 Вт

Это соответствует закону сохранения энергии.

Общее обсуждение

Обратите внимание, что как токи, так и мощность при параллельном подключении больше, чем для тех же устройств, подключенных последовательно.

Основные характеристики резисторов, подключенных параллельно
  1. Параллельное сопротивление определяется из [latex] \ frac {1} {{R} _ {\ text {p}}} = \ frac {1} {{R} _ {1}} + \ frac {1} { {R} _ {2}} + \ frac {1} {{R} _ {3}} + \ text {…} \\ [/ latex], и оно меньше любого отдельного сопротивления в комбинации.
  2. Каждый резистор, включенный параллельно, имеет одинаковое полное напряжение источника. (В системах распределения электроэнергии чаще всего используются параллельные соединения для питания бесчисленных устройств, обслуживаемых одним и тем же напряжением, и для того, чтобы они могли работать независимо.)
  3. Не каждый параллельный резистор получает полный ток; они делят это.

Сочетания последовательного и параллельного

Более сложные соединения резисторов иногда представляют собой просто комбинации последовательного и параллельного. Они часто встречаются, особенно если учитывать сопротивление провода. В этом случае сопротивление провода включено последовательно с другими сопротивлениями, включенными параллельно. Комбинации последовательного и параллельного подключения можно свести к одному эквивалентному сопротивлению, используя технику, показанную на рисунке 4.Различные части идентифицируются как последовательные или параллельные, уменьшаются до их эквивалентов и далее уменьшаются до тех пор, пока не останется единственное сопротивление. Процесс более трудоемкий, чем трудный.

Рис. 4. Эта комбинация из семи резисторов имеет как последовательные, так и параллельные части. Каждое из них идентифицируется и приводится к эквивалентному сопротивлению, а затем уменьшается до достижения единого эквивалентного сопротивления.

Самая простая комбинация последовательного и параллельного сопротивления, показанная на рисунке 4, также является наиболее поучительной, поскольку она используется во многих приложениях.Например, R 1 может быть сопротивлением проводов от автомобильного аккумулятора к его электрическим устройствам, которые подключены параллельно. R 2 и R 3 могли быть стартером и светом салона. Ранее мы предполагали, что сопротивление провода незначительно, но, когда это не так, оно имеет важные последствия, как показывает следующий пример.

Пример 3. Расчет сопротивления,

IR Падение, ток и рассеиваемая мощность: объединение последовательных и параллельных цепей

На рис. 5 показаны резисторы из двух предыдущих примеров, подключенные другим способом — комбинацией последовательного и параллельного подключения.Мы можем считать R 1 сопротивлением проводов, ведущих к R 2 и R 3 . (а) Найдите полное сопротивление. (b) Что такое падение IR в R 1 ? (c) Найдите текущие значения от I 2 до R 2 . (d) Какую мощность рассеивает R 2 ?

Рис. 5. Эти три резистора подключены к источнику напряжения, так что R 2 и R 3 параллельны друг другу, и эта комбинация включена последовательно с R 1 .

Стратегия и решение для (а)

Чтобы найти полное сопротивление, отметим, что R 2 и R 3 находятся параллельно, и их комбинация R p последовательно с R 1 . Таким образом, полное (эквивалентное) сопротивление этой комбинации составляет

.

R tot = R 1 + R p .

Сначала мы находим R p , используя уравнение для параллельных резисторов и вводя известные значения:

[латекс] \ frac {1} {{R} _ {\ text {p}}} = \ frac {1} {{R} _ {2}} + \ frac {1} {{R} _ {3 }} = \ frac {1} {6 \ text {.} \ text {00} \ text {} \ Omega} + \ frac {1} {\ text {13} \ text {.} 0 \ text {} \ Omega} = \ frac {0.2436} {\ text {} \ Омега} \\ [/ латекс].

Инвертирование дает

[латекс] {R} _ {\ text {p}} = \ frac {1} {0,2436} \ text {} \ Omega = 4.11 \ text {} \ Omega \\ [/ latex].

Таким образом, общее сопротивление равно

.

R tot = R 1 + R p = 1,00 Ом + 4,11 Ом = 5,11 Ом.

Обсуждение для (а)

Общее сопротивление этой комбинации является промежуточным между значениями чистой серии и чистой параллели (20.0 Ом и 0,804 Ом соответственно), найденные для тех же резисторов в двух предыдущих примерах.

Стратегия и решение для (b)

Чтобы найти падение IR в R 1 , отметим, что полный ток I протекает через R 1 . Таким образом, его падение IR составляет

В 1 = ИК 1

Мы должны найти I , прежде чем сможем вычислить V 1 .Полный ток I находится с помощью закона Ома для цепи. То есть

[латекс] I = \ frac {V} {{R} _ {\ text {tot}}} = \ frac {\ text {12.0} \ text {V}} {5.11 \ text {} \ Omega} = 2.35 \ text {A} \\ [/ latex].

Вводя это в выражение выше, мы получаем

В 1 = IR 1 = (2,35 А) (1,00 Ом) = 2,35 В.

Обсуждение для (б)

Напряжение, приложенное к R 2 и R 3 , меньше общего напряжения на величину В 1 .Когда сопротивление провода велико, это может существенно повлиять на работу устройств, представленных R 2 и R 3 .

Стратегия и решение для (c)

Чтобы найти ток через R 2 , мы должны сначала найти приложенное к нему напряжение. Мы называем это напряжение В p , потому что оно приложено к параллельной комбинации резисторов. Напряжение, приложенное как к R 2 , так и к R 3 , уменьшается на величину В 1 , и поэтому оно составляет

V p = V V 1 = 12.0 В — 2,35 В = 9,65 В.

Теперь ток I 2 через сопротивление R 2 находится по закону Ома:

[латекс] {I} _ {2} = \ frac {{V} _ {\ text {p}}} {{R} _ {2}} = \ frac {9.65 \ text {V}} {6.00 \ текст {} \ Omega} = 1,61 \ text {A} \\ [/ latex].

Обсуждение для (c)

Ток меньше 2,00 А, который протекал через R 2 , когда он был подключен параллельно к батарее в предыдущем примере параллельной цепи.

Стратегия и решение для (d)

Мощность, рассеиваемая на R 2 определяется на

P 2 = ( I 2 ) 2 R 2 = (1,61 A) 2 (6,00 Ом) = 15,5 Вт

Обсуждение для (d)

Мощность меньше 24,0 Вт, рассеиваемых этим резистором при параллельном подключении к источнику 12,0 В.

Одним из следствий этого последнего примера является то, что сопротивление в проводах снижает ток и мощность, подаваемую на резистор.Если сопротивление провода относительно велико, как в изношенном (или очень длинном) удлинителе, то эти потери могут быть значительными. Если потребляется большой ток, падение IR в проводах также может быть значительным.

Например, когда вы роетесь в холодильнике и включается мотор, свет холодильника на мгновение гаснет. Точно так же вы можете увидеть тусклый свет в салоне, когда вы запускаете двигатель вашего автомобиля (хотя это может быть связано с сопротивлением внутри самой батареи).

То, что происходит в этих сильноточных ситуациях, показано на рисунке 6. Устройство, обозначенное как R 3 , имеет очень низкое сопротивление, поэтому при его включении протекает большой ток. Этот увеличенный ток вызывает большее падение IR в проводах, представленных R 1 , уменьшая напряжение на лампочке (которое составляет R 2 ), которое затем заметно тускнеет.

Рис. 6. Почему гаснет свет при включении большого прибора? Ответ заключается в том, что большой ток, потребляемый двигателем прибора, вызывает значительное падение напряжения в проводах и снижает напряжение на свету.

Проверьте свое понимание

Можно ли любую произвольную комбинацию резисторов разбить на последовательную и параллельную? Посмотрите, сможете ли вы нарисовать принципиальную схему резисторов, которые нельзя разбить на комбинации последовательно и параллельно.

Решение Нет, есть много способов подключения резисторов, которые не являются комбинациями последовательного и параллельного, включая петли и переходы. В таких случаях правила Кирхгофа, которые будут включены в Правила Кирхгофа, позволят вам проанализировать схему.

Стратегии решения проблем для последовательных и параллельных резисторов
  1. Нарисуйте четкую принципиальную схему, обозначив все резисторы и источники напряжения. Этот шаг включает список известных проблем, поскольку они отмечены на вашей принципиальной схеме.
  2. Определите, что именно необходимо определить в проблеме (определите неизвестные). Письменный список полезен.
  3. Определите, включены ли резисторы последовательно, параллельно или в комбинации последовательно и параллельно.Изучите принципиальную схему, чтобы сделать эту оценку. Резисторы включены последовательно, если через них должен последовательно проходить один и тот же ток.
  4. Используйте соответствующий список основных функций для последовательных или параллельных подключений, чтобы найти неизвестные. Есть один список для серий, а другой — для параллелей. Если ваша проблема представляет собой комбинацию последовательного и параллельного соединения, уменьшайте ее поэтапно, рассматривая отдельные группы последовательных или параллельных соединений, как это сделано в этом модуле и примерах. Особое примечание: при нахождении R необходимо соблюдать осторожность.
  5. Проверьте, являются ли ответы разумными и последовательными. Единицы и числовые результаты должны быть разумными. Общее последовательное сопротивление должно быть больше, а общее параллельное сопротивление, например, должно быть меньше. Мощность должна быть больше для одних и тех же устройств, подключенных параллельно, по сравнению с последовательными и так далее.

Сводка раздела

Концептуальные вопросы

1. Переключатель имеет переменное сопротивление, которое почти равно нулю в замкнутом состоянии и очень велико в разомкнутом состоянии, и он включен последовательно с устройством, которым он управляет.Объясните влияние переключателя на рис. 7 на ток в разомкнутом и замкнутом состоянии.

Рис. 7. Переключатель обычно включается последовательно с источником сопротивления и напряжения. В идеале переключатель имеет почти нулевое сопротивление в замкнутом состоянии, но имеет чрезвычайно большое сопротивление в разомкнутом состоянии. (Обратите внимание, что на этой диаграмме скрипт E представляет напряжение (или электродвижущую силу) батареи.)

2. Какое напряжение на разомкнутом переключателе на Рисунке 7?

3. На разомкнутом переключателе есть напряжение, как на Рисунке 7.Почему же тогда мощность, рассеиваемая разомкнутым переключателем, мала?

4. Почему мощность, рассеиваемая замкнутым переключателем, как на Рисунке 7, мала?

5. Студент в физической лаборатории по ошибке подключил электрическую лампочку, батарею и выключатель, как показано на рисунке 8. Объясните, почему лампочка горит, когда выключатель разомкнут, и гаснет, когда он замкнут. (Не пытайтесь — батарея сильно разряжается!)

Рис. 8. Ошибка подключения. Включите этот переключатель параллельно устройству, обозначенному [латекс] R [/ латекс].(Обратите внимание, что на этой диаграмме скрипт E представляет напряжение (или электродвижущую силу) батареи.)

6. Зная, что сила электрического шока зависит от величины тока, протекающего через ваше тело, вы бы предпочли, чтобы он был включен последовательно или параллельно с сопротивлением, таким как нагревательный элемент тостера, если он шокирован им? Объяснять.

7. Были бы ваши фары тусклыми при запуске двигателя автомобиля, если бы провода в вашем автомобиле были сверхпроводниками? (Не пренебрегайте внутренним сопротивлением батареи.) Объяснять.

8. Некоторые гирлянды праздничных огней соединены последовательно для экономии затрат на проводку. В старой версии использовались лампочки, которые при перегорании прерывали электрическое соединение, как открытый выключатель. Если одна такая лампочка перегорит, что случится с остальными? Если такая цепочка работает от 120 В и имеет 40 одинаковых лампочек, каково нормальное рабочее напряжение каждой? В более новых версиях используются лампы, которые при перегорании замыкаются накоротко, как замкнутый выключатель. Если одна такая лампочка перегорит, что случится с остальными? Если такая цепочка работает от 120 В и в ней осталось 39 идентичных лампочек, каково тогда рабочее напряжение каждой?

9.Если две бытовые лампочки мощностью 60 и 100 Вт подключить последовательно к бытовой электросети, какая из них будет ярче? Объяснять.

10. Предположим, вы проводите физическую лабораторию, в которой вас просят вставить резистор в цепь, но все прилагаемые резисторы имеют большее сопротивление, чем запрошенное значение. Как бы вы соединили доступные сопротивления, чтобы попытаться получить меньшее запрошенное значение?

11. Перед Второй мировой войной некоторые радиостанции получали питание через «шнур сопротивления», который имел значительное сопротивление.Такой резистивный шнур снижает напряжение до желаемого уровня для ламп радиоприемника и т.п., и это экономит расходы на трансформатор. Объясните, почему шнуры сопротивления нагреваются и тратят энергию при включенном радио.

12. У некоторых лампочек есть три уровня мощности (не включая ноль), получаемые от нескольких нитей накала, которые индивидуально переключаются и соединяются параллельно. Какое минимальное количество нитей необходимо для трех режимов мощности?

Задачи и упражнения

Примечание. Можно считать, что данные, взятые из цифр, имеют точность до трех значащих цифр.

1. (а) Каково сопротивление десяти последовательно соединенных резисторов сопротивлением 275 Ом? (б) Параллельно?

2. (a) Каково сопротивление последовательно соединенных резисторов 1,00 × 10 2 Ом, 2,50 кОм и 4,00 кОм? (б) Параллельно?

3. Какое наибольшее и наименьшее сопротивление можно получить, соединив резисторы на 36,0 Ом, 50,0 Ом и 700 Ом?

4. Тостер на 1800 Вт, электрическая сковорода на 1400 Вт и лампа на 75 Вт подключены к одной розетке в цепи 15 А, 120 В.(Три устройства работают параллельно, если они подключены к одной розетке.) а) Какой ток потребляет каждое устройство? (b) Перегорит ли эта комбинация предохранитель на 15 А?

5. Фара мощностью 30,0 Вт и стартер мощностью 2,40 кВт обычно подключаются параллельно в систему на 12,0 В. Какую мощность потребляли бы одна фара и стартер при последовательном подключении к батарее 12,0 В? (Не обращайте внимания на любое другое сопротивление в цепи и любое изменение сопротивления в двух устройствах.)

6.(a) Для батареи 48,0 В и резисторов 24,0 Ом и 96,0 Ом найдите для каждого из них ток и мощность при последовательном соединении. (b) Повторите, когда сопротивления включены параллельно.

7. Ссылаясь на пример комбинирования последовательных и параллельных цепей и рисунок 5, вычислите I 3 двумя следующими способами: (a) по известным значениям I и I 2 ; (б) используя закон Ома для R 3 . В обеих частях явно показано, как вы следуете шагам, описанным в описании стратегии решения проблем для последовательных и параллельных резисторов выше.

Рис. 5. Эти три резистора подключены к источнику напряжения, так что R 2 и R 3 параллельны друг другу, и эта комбинация включена последовательно с R 1 .

8. Ссылаясь на рисунок 5: (a) Вычислите P 3 и обратите внимание на его сравнение с P 3 , найденным в первых двух примерах задач в этом модуле. (b) Найдите полную мощность, отдаваемую источником, и сравните ее с суммой мощностей, рассеиваемых резисторами.

9. См. Рисунок 6 и обсуждение затемнения света при включении тяжелого прибора. (a) Учитывая, что источник напряжения составляет 120 В, сопротивление провода составляет 0,400 Ом, а номинальная мощность лампы составляет 75,0 Вт, какая мощность будет рассеиваться лампой, если при включении двигателя через провода пройдет в общей сложности 15,0 А? Предположите незначительное изменение сопротивления лампы. б) Какая мощность потребляет двигатель?

Рис. 6. Почему гаснет свет при включении большого прибора? Ответ заключается в том, что большой ток, потребляемый двигателем прибора, вызывает значительное падение напряжения в проводах и снижает напряжение на свету.

10. Линия электропередачи на 240 кВ, имеющая 5,00 × 10 2 , подвешена к заземленным металлическим опорам с помощью керамических изоляторов, каждый из которых имеет сопротивление 1,00 × 10 9 Ом (рис. 9 (а)). Какое сопротивление на землю у 100 изоляторов? (b) Рассчитайте мощность, рассеиваемую 100 из них. (c) Какая доля мощности, переносимой линией, составляет это? Ясно покажите, как вы следуете шагам, описанным в описании стратегии решения проблем для последовательных и параллельных резисторов выше.

Рис. 9. Высоковольтная (240 кВ) линия электропередачи 5,00 × 10 2 подвешена к заземленной металлической опоре электропередачи. Ряд керамических изоляторов обеспечивает сопротивление 1,00 × 10 9 Ом каждый.

11. Покажите, что если два резистора R 1 и R 2 объединены, и один из них намного больше другого ( R 1 >> R 2 ): (a ) Их последовательное сопротивление почти равно большему сопротивлению R 1 .(b) Их параллельное сопротивление почти равно меньшему сопротивлению R 2 .

12. Необоснованные результаты Два резистора, один с сопротивлением 145 Ом, подключены параллельно, чтобы получить общее сопротивление 150 Ом. а) Каково значение второго сопротивления? б) Что неразумного в этом результате? (c) Какие предположения необоснованны или непоследовательны?

13. Необоснованные результаты Два резистора, один из которых имеет сопротивление 900 кОм, соединены последовательно, чтобы получить общее сопротивление 0.500 МОм. а) Каково значение второго сопротивления? б) Что неразумного в этом результате? (c) Какие предположения необоснованны или непоследовательны?

Глоссарий

серия:
последовательность резисторов или других компонентов, включенных в цепь один за другим
резистор:
компонент, обеспечивающий сопротивление току, протекающему через электрическую цепь
сопротивление:
вызывает потерю электроэнергии в цепи
Закон Ома:
соотношение между током, напряжением и сопротивлением в электрической цепи: В = IR
напряжение:
электрическая потенциальная энергия на единицу заряда; электрическое давление, создаваемое источником питания, например аккумулятором
падение напряжения:
потеря электроэнергии при прохождении тока через резистор, провод или другой компонент
ток:
поток заряда через электрическую цепь мимо заданной точки измерения
Закон Джоуля:
взаимосвязь между потенциальной электрической мощностью, напряжением и сопротивлением в электрической цепи, определяемая следующим образом: [latex] {P} _ {e} = \ text {IV} [/ latex]
параллельно:
разводку резисторов или других компонентов в электрической цепи, так что каждый компонент получает одинаковое напряжение от источника питания; часто изображается на диаграмме в виде лестницы, где каждый компонент находится на ступеньке лестницы

Избранные решения проблем и упражнения

1.(а) 2,75 кОм (б) 27,5 Ом

3. (а) 786 Ом (б) 20,3 Ом

5. 29,6 Вт

7. (а) 0,74 А (б) 0,742 А

9. (а) 60,8 Вт (б) 3,18 кВт

11. (a) [латекс] \ begin {array} {} {R} _ {\ text {s}} = {R} _ {1} + {R} _ {2} \\ \ Rightarrow {R} _ {\ text {s}} \ приблизительно {R} _ {1} \ left ({R} _ {1} \ text {>>} {R} _ {2} \ right) \ end {array} \\ [/ латекс]

(b) [латекс] \ frac {1} {{R} _ {p}} = \ frac {1} {{R} _ {1}} + \ frac {1} {{R} _ {2} } = \ frac {{R} _ {1} + {R} _ {2}} {{R} _ {1} {R} _ {2}} \\ [/ latex],

, так что

[латекс] \ begin {array} {} {R} _ {p} = \ frac {{R} _ {1} {R} _ {2}} {{R} _ {1} + {R} _ {2}} \ приблизительно \ frac {{R} _ {1} {R} _ {2}} {{R} _ {1}} = {R} _ {2} \ left ({R} _ {1 } \ text {>>} {R} _ {2} \ right) \ text {.} \ end {array} \\ [/ latex]

13. (a) –400 кОм (b) Сопротивление не может быть отрицательным. (c) Считается, что последовательное сопротивление меньше, чем у одного из резисторов, но должно быть больше, чем у любого из резисторов.

21.1 Последовательные и параллельные резисторы — College Physics

Сводка

  • Нарисуйте цепь с резисторами, включенными параллельно и последовательно.
  • Рассчитайте падение напряжения тока на резисторе, используя закон Ома.
  • Сравните способ расчета общего сопротивления для резисторов, включенных последовательно и параллельно.
  • Объясните, почему полное сопротивление параллельной цепи меньше наименьшего сопротивления любого из резисторов в этой цепи.
  • Вычислите общее сопротивление цепи, которая содержит смесь резисторов, включенных последовательно и параллельно.

Большинство схем имеет более одного компонента, называемого резистором , который ограничивает поток заряда в цепи. Мера этого предела расхода заряда называется сопротивлением . Простейшие комбинации резисторов — это последовательное и параллельное соединение, показанное на Рисунке 1.Общее сопротивление комбинации резисторов зависит как от их индивидуальных значений, так и от способа их подключения.

Рис. 1. (a) Последовательное соединение резисторов. (б) Параллельное соединение резисторов.

Когда резисторы серии ? Резисторы включены последовательно всякий раз, когда поток заряда, называемый током , должен проходить через устройства последовательно. Например, если ток течет через человека, держащего отвертку, и попадает в Землю, то $ latex \ boldsymbol {R_1} $ на рисунке 1 (a) может быть сопротивлением вала отвертки, $ latex \ boldsymbol {R_2} $ сопротивление его ручки, $ latex \ boldsymbol {R_3} $ сопротивление тела человека и $ latex \ boldsymbol {R_4} $ сопротивление его обуви.

На рисунке 2 показаны резисторы, последовательно подключенные к источнику напряжения . Кажется разумным, что полное сопротивление является суммой отдельных сопротивлений, учитывая, что ток должен проходить через каждый резистор последовательно. (Этот факт был бы преимуществом для человека, желающего избежать поражения электрическим током, который мог бы уменьшить ток, надев обувь с высоким сопротивлением на резиновой подошве. Это могло бы стать недостатком, если бы одно из сопротивлений было неисправным шнуром с высоким сопротивлением. прибор, уменьшающий рабочий ток.)

Рисунок 2. Три резистора, подключенных последовательно к батарее (слева) и эквивалентному одиночному или последовательному сопротивлению (справа).

Чтобы убедиться, что последовательно включенные сопротивления действительно складываются, давайте рассмотрим потерю электроэнергии, называемую падением напряжения , в каждом резисторе на Рисунке 2.

Согласно закону Ома , падение напряжения $ latex \ boldsymbol {V} $ на резисторе при протекании через него тока рассчитывается с использованием уравнения $ latex \ boldsymbol {V = IR} $, где $ latex \ boldsymbol {I} $ равняется току в амперах (A), а $ latex \ boldsymbol {R} $ — сопротивлению в омах $ latex \ boldsymbol {(\ Omega)} $.Другой способ думать об этом: $ latex \ boldsymbol {V} $ — это напряжение, необходимое для протекания тока $ latex \ boldsymbol {I} $ через сопротивление $ latex \ boldsymbol {R} $.

Таким образом, падение напряжения на $ latex \ boldsymbol {R_1} $ составляет $ latex \ boldsymbol {V_1 = IR_1} $, на $ latex \ boldsymbol {R_2} $ — $ latex \ boldsymbol {V_2 = IR_2} $, и что через $ latex \ boldsymbol {R_3} $ стоит $ latex \ boldsymbol {V_3 = IR_3} $. Сумма этих напряжений равна выходному напряжению источника; то есть

$ латекс \ boldsymbol {V = V_1 + V_2 + V_3}.$

Это уравнение основано на сохранении энергии и сохранении заряда. Электрическая потенциальная энергия описывается уравнением $ latex \ boldsymbol {PE = qV} $, где $ latex \ boldsymbol {q} $ — электрический заряд, а $ latex \ boldsymbol {V} $ — напряжение. Таким образом, энергия, подводимая источником, равна $ latex \ boldsymbol {qV} $, а энергия, рассеиваемая резисторами, равна

.

$ латекс \ boldsymbol {qV_1 + qV_2 + qV_3}. $

Связи: законы сохранения

Вывод выражений для последовательного и параллельного сопротивления основан на законах сохранения энергии и сохранения заряда, которые утверждают, что общий заряд и полная энергия постоянны в любом процессе.Эти два закона непосредственно участвуют во всех электрических явлениях и будут многократно использоваться для объяснения как конкретных эффектов, так и общего поведения электричества.

Эти энергии должны быть равны, потому что в цепи нет другого источника и другого назначения для энергии. Таким образом, $ latex \ boldsymbol {qV = qV_1 + qV_2 + qV_3} $. Плата $ latex \ boldsymbol {q} $ отменяется, в результате чего получается $ latex \ boldsymbol {V = V_1 + V_2 + V_3} $, как указано. (Обратите внимание, что одинаковое количество заряда проходит через батарею и каждый резистор за заданный промежуток времени, поскольку нет емкости для хранения заряда, нет места для утечки заряда и заряд сохраняется.)

Теперь подстановка значений отдельных напряжений дает

$ латекс \ boldsymbol {V = IR_1 + IR_2 + IR_3 = I (R_1 + R_2 + R_3)}. $

Обратите внимание, что для эквивалентного сопротивления одиночной серии $ latex \ boldsymbol {R_s} $ мы имеем

$ латекс \ boldsymbol {V = IR_s}. $

Это означает, что полное или эквивалентное последовательное сопротивление $ latex \ boldsymbol {R_s} $ трех резисторов равно

$ латекс \ boldsymbol {R_s = R_1 + R_2 + R_3} $.

Эта логика действительна в общем для любого количества резисторов, включенных последовательно; таким образом, полное сопротивление $ latex \ boldsymbol {R_s} $ последовательного соединения равно

$ латекс \ boldsymbol {R_s = R_1 + R_2 + R_3 + \ dots} $

, как предлагается.Поскольку весь ток должен проходить через каждый резистор, он испытывает сопротивление каждого, а последовательно соединенные сопротивления просто складываются.

Пример 1: Расчет сопротивления, тока, падения напряжения и рассеиваемой мощности: анализ последовательной цепи

Предположим, что выходное напряжение батареи на рисунке 2 равно $ latex \ boldsymbol {12.0 \; \ textbf {V}} $, а сопротивление равно $ latex \ boldsymbol {R_1 = 1.00 \; \ Omega} $, $ latex \ boldsymbol {R_2 = 6.00 \; \ Omega} $ и $ latex \ boldsymbol {R_3 = 13.0 \; \ Omega} $. а) Каково полное сопротивление? (б) Найдите ток. (c) Рассчитайте падение напряжения на каждом резисторе и покажите, как они складываются, чтобы равняться выходному напряжению источника. (d) Рассчитайте мощность, рассеиваемую каждым резистором. (e) Найдите выходную мощность источника и покажите, что она равна общей мощности, рассеиваемой резисторами.

Стратегия и решение для (а)

Общее сопротивление — это просто сумма отдельных сопротивлений, определяемая следующим уравнением:

$ latex \ begin {array} {r @ {{} = {}} l} \ boldsymbol {R_s} & \ boldsymbol {R_1 + R_2 + R_3} \\ [1em] & \ boldsymbol {1.00 \; \ Omega + 6.00 \; \ Omega + 13.0 \; \ Omega} \\ [1em] & \ boldsymbol {20.0 \; \ Omega}. \ end {array}

долл. США

Стратегия и решение для (b)

Ток определяется по закону Ома $ latex \ boldsymbol {V = IR} $. Ввод значения приложенного напряжения и общего сопротивления дает ток для цепи:

$ latex \ boldsymbol {I =} $ latex \ boldsymbol {=} $ latex \ boldsymbol {= 0.600 \; \ textbf {A}}. $

Стратегия и решение для (c)

Напряжение — или падение $ latex \ boldsymbol {IR} $ — в резисторе определяется законом Ома.Ввод значения тока и значения первого сопротивления дает

.

$ латекс \ boldsymbol {V_1 = IR_1 = (0.600 \; \ textbf {A}) (1.0 \; \ Omega) = 0.600 \; \ textbf {V}}. $

Аналогично

$ латекс \ boldsymbol {V_2 = IR_2 = (0.600 \; \ textbf {A}) (6.0 \; \ Omega) = 3.60 \; \ textbf {V}} $

и

$ латекс \ boldsymbol {V_3 = IR_3 = (0.600 \; \ textbf {A}) (13.0 \; \ Omega) = 7.80 \; \ textbf {V}}. $

Обсуждение для (c)

Три капли $ latex \ boldsymbol {IR} $ добавляют к $ latex \ boldsymbol {12.0 \; \ textbf {V}} $, прогноз:

$ латекс \ boldsymbol {V_1 + V_2 + V_3 = (0,600 + 3,60 + 7,80) \; \ textbf {V} = 12,0 \; \ textbf {V}}. 2 R_1 = (0.2} {R}} $, где $ latex \ boldsymbol {V} $ — это падение напряжения на резисторе (а не полное напряжение источника). Будут получены те же значения.

Стратегия и решение для (e)

Самый простой способ рассчитать выходную мощность источника — использовать $ latex \ boldsymbol {P = IV} $, где $ latex \ boldsymbol {V} $ — напряжение источника. Это дает

$ латекс \ boldsymbol {P = (0.600 \; \ textbf {A}) (12.0 \; \ textbf {V}) = 7.20 \; \ textbf {W}}. $

Обсуждение для (e)

Обратите внимание, что по совпадению общая мощность, рассеиваемая резисторами, также равна 7.20 Вт, столько же, сколько мощность, выдаваемая источником. То есть

$ латекс \ boldsymbol {P_1 + P_2 + P_3 = (0,360 + 2,16 + 4,68) \; \ textbf {W} = 7.20 \; \ textbf {W}}. $

Мощность — это энергия в единицу времени (ватты), поэтому для сохранения энергии требуется, чтобы выходная мощность источника была равна общей мощности, рассеиваемой резисторами.

Основные характеристики резисторов серии

  1. Сопротивления серии добавляют: $ latex \ boldsymbol {R_s = R_1 + R_2 + R_3 + \ dots} $.
  2. Одинаковый ток протекает последовательно через каждый резистор.
  3. Отдельные последовательно включенные резисторы не получают полное напряжение источника, а делят его.

На рисунке 3 показаны резисторы параллельно , подключенные к источнику напряжения. Резисторы включены параллельно, когда каждый резистор подключен непосредственно к источнику напряжения с помощью соединительных проводов с незначительным сопротивлением. Таким образом, к каждому резистору приложено полное напряжение источника.

Каждый резистор потребляет такой же ток, как если бы он один был подключен к источнику напряжения (при условии, что источник напряжения не перегружен).Например, автомобильные фары, радио и т. Д. Подключены параллельно, так что они используют полное напряжение источника и могут работать полностью независимо. То же самое и в вашем доме, или в любом другом здании. (См. Рисунок 3 (b).)

Рис. 3. (a) Три резистора, подключенных параллельно батарее, и эквивалентное одиночное или параллельное сопротивление. (б) Электроснабжение в доме. (предоставлено Дмитрием Г., Wikimedia Commons)

Чтобы найти выражение для эквивалентного параллельного сопротивления $ latex \ boldsymbol {R _ {\ textbf {p}}} $, давайте рассмотрим протекающие токи и их связь с сопротивлением.Поскольку каждый резистор в цепи имеет полное напряжение, токи, протекающие через отдельные резисторы, равны $ latex \ boldsymbol {I_1 = \ frac {V} {R_1}} $, $ latex \ boldsymbol {I_2 = \ frac {V} { R_2}} $ и $ latex \ boldsymbol {I_3 = \ frac {V} {R_3}} $. Сохранение заряда подразумевает, что полный ток $ latex \ boldsymbol {I} $, производимый источником, является суммой этих токов:

$ латекс \ boldsymbol {I = I_1 + I_2 + I_3}. $

Подстановка выражений для отдельных токов дает

Обратите внимание, что закон Ома для эквивалентного одиночного сопротивления дает

$ латекс \ boldsymbol {I =} $ латекс \ boldsymbol {= V} $

Члены в скобках в последних двух уравнениях должны быть равны.Обобщая для любого количества резисторов, общее сопротивление $ latex \ boldsymbol {R_p} $ параллельного соединения связано с отдельными сопротивлениями на

$ latex \ boldsymbol {=} $ $ latex \ boldsymbol {+} $ $ latex \ boldsymbol {+} $ $ latex \ boldsymbol {+ \ cdots} $

Это соотношение приводит к общему сопротивлению $ latex \ boldsymbol {R_p} $, которое меньше наименьшего из отдельных сопротивлений. (Это видно в следующем примере.) При параллельном подключении резисторов от источника течет больше тока, чем протекает по любому из них по отдельности, поэтому общее сопротивление ниже.

Пример 2: Расчет сопротивления, тока, рассеиваемой мощности и выходной мощности: анализ параллельной цепи

Пусть выходное напряжение батареи и сопротивления в параллельном соединении на рисунке 3 будут такими же, как и в ранее рассмотренном последовательном соединении: $ latex \ boldsymbol {V = 12.0 \; \ textbf {V}} $, $ latex \ boldsymbol { R_1 = 1.00 \; \ Omega} $, $ latex \ boldsymbol {R_2 = 6.00 \; \ Omega} $ и $ latex \ boldsymbol {R_3 = 13.0 \; \ Omega} $. а) Каково полное сопротивление? (б) Найдите полный ток.(c) Рассчитайте токи в каждом резисторе и покажите, как они складываются, чтобы равняться общему выходному току источника. (d) Рассчитайте мощность, рассеиваемую каждым резистором. (e) Найдите выходную мощность источника и покажите, что она равна общей мощности, рассеиваемой резисторами.

Стратегия и решение для (а)

Общее сопротивление для параллельной комбинации резисторов находится с помощью следующего уравнения. Ввод известных значений дает

Таким образом,

(Обратите внимание, что в этих расчетах каждый промежуточный ответ отображается с дополнительной цифрой.)

Мы должны перевернуть это, чтобы найти полное сопротивление $ latex \ boldsymbol {R_p} $. Это дает

$ latex \ boldsymbol {R_p =} $ latex \ boldsymbol {\ Omega = 0.8041 \; \ Omega}. $

Суммарное сопротивление с правильным количеством значащих цифр равно $ latex \ boldsymbol {R_p = 0.804 \; \ Omega} $

.

Обсуждение для (а)

$ latex \ boldsymbol {R_p} $, как и предполагалось, меньше наименьшего индивидуального сопротивления.

Стратегия и решение для (b)

Полный ток можно найти из закона Ома, заменив полное сопротивление $ latex \ boldsymbol {R_p} $.Это дает

$ latex \ boldsymbol {I =} $ latex \ boldsymbol {=} $ latex \ boldsymbol {= 14.92 \; \ textbf {A}} $

Обсуждение для (б)

Текущее значение $ latex \ boldsymbol {I} $ для каждого устройства намного больше, чем для тех же устройств, подключенных последовательно (см. Предыдущий пример). Схема с параллельным соединением имеет меньшее общее сопротивление, чем резисторы, включенные последовательно.

Стратегия и решение для (c)

Отдельные токи легко вычислить по закону Ома, поскольку каждый резистор получает полное напряжение.Таким образом,

$ latex \ boldsymbol {I_1 =} $ latex \ boldsymbol {=} $ $ latex \ boldsymbol {= 12.0 \; \ textbf {A}}. $

Аналогично

$ latex \ boldsymbol {I_2 =} $ latex \ boldsymbol {=} $ $ latex \ boldsymbol {= 2.00 \; \ textbf {A}} $

и

$ latex \ boldsymbol {I_3 =} $ latex \ boldsymbol {=} $ $ latex \ boldsymbol {= 0.92 \; \ textbf {A}}. $

Обсуждение для (c)

Общий ток складывается из отдельных токов:

$ латекс \ boldsymbol {I_1 + I_2 + I_3 = 14.2} {R}} $, так как каждый резистор получает полное напряжение. Таким образом,

$ latex \ boldsymbol {P_1 =} $ latex \ boldsymbol {=} $ $ latex \ boldsymbol {= 144 \; \ textbf {W}}. $

Аналогично

$ latex \ boldsymbol {P_2 =} $ latex \ boldsymbol {=} $ $ latex \ boldsymbol {= 24.0 \; \ textbf {W}} $

и

$ latex \ boldsymbol {P_3 =} $ latex \ boldsymbol {=} $ $ latex \ boldsymbol {= 11.1 \; \ textbf {W}}. $

Обсуждение для (d)

Мощность, рассеиваемая каждым резистором параллельно, значительно выше, чем при последовательном подключении к тому же источнику напряжения.

Стратегия и решение для (e)

Общую мощность также можно рассчитать несколькими способами. Выбирая $ latex \ boldsymbol {P = IV} $ и вводя полный ток, получаем

$ латекс \ boldsymbol {P = IV = (14.92 \; \ textbf {A}) (12.0 \; \ textbf {V}) = 179 \; \ textbf {W}}. $

Обсуждение для (e)

Суммарная мощность, рассеиваемая резисторами, также 179 Вт:

$ латекс \ boldsymbol {P_1 + P_2 + P_3 = 144 \; \ textbf {W} + 24.0 \; \ textbf {W} + 11.1 \; \ textbf {W} = 179 \; \ textbf {W}}. $

Это соответствует закону сохранения энергии.

Общее обсуждение

Обратите внимание, что как токи, так и мощность при параллельном подключении больше, чем для тех же устройств, подключенных последовательно.

Основные характеристики параллельных резисторов

  1. Параллельное сопротивление определяется из $ latex \ boldsymbol {\ frac {1} {R_p} = \ frac {1} {R_1} + \ frac {1} {R_2} + \ frac {1} {R_3} + \ cdots } $, и оно меньше любого отдельного сопротивления в комбинации.
  2. Каждый резистор, включенный параллельно, имеет одинаковое полное напряжение источника. (В системах распределения электроэнергии чаще всего используются параллельные соединения для питания бесчисленных устройств, обслуживаемых одинаковым напряжением, и для того, чтобы они могли работать независимо.)
  3. Не каждый параллельный резистор получает полный ток; они делят это.

Более сложные соединения резисторов иногда представляют собой просто комбинации последовательного и параллельного. Они часто встречаются, особенно если учитывать сопротивление провода.В этом случае сопротивление провода включено последовательно с другими сопротивлениями, включенными параллельно.

Комбинации последовательного и параллельного соединения можно свести к одному эквивалентному сопротивлению, используя метод, показанный на рисунке 4. Различные части идентифицируются как последовательные или параллельные, сокращаются до их эквивалентов и далее уменьшаются до тех пор, пока не останется единственное сопротивление. Процесс более трудоемкий, чем трудный.

Рисунок 4. Эта комбинация из семи резисторов имеет как последовательные, так и параллельные части.Каждое из них идентифицируется и приводится к эквивалентному сопротивлению, а затем уменьшается до достижения единого эквивалентного сопротивления.

Самая простая комбинация последовательного и параллельного сопротивления, показанная на рисунке 5, также является наиболее поучительной, поскольку она используется во многих приложениях. Например, $ latex \ boldsymbol {R_1} $ может быть сопротивлением проводов от автомобильного аккумулятора к его электрическим устройствам, которые подключены параллельно. $ latex \ boldsymbol {R_2} $ и $ latex \ boldsymbol {R_3} $ могут быть стартером и светом салона.Ранее мы предполагали, что сопротивление провода незначительно, но, когда это не так, оно имеет важные последствия, как показывает следующий пример.

Пример 3: Расчет сопротивления, $ latex \ boldsymbol {IR} $, падения, тока и рассеиваемой мощности: объединение последовательных и параллельных цепей

На рис. 5 показаны резисторы из двух предыдущих примеров, подключенные другим способом — комбинацией последовательного и параллельного подключения. Мы можем рассматривать $ latex \ boldsymbol {R_1} $ как сопротивление проводов, ведущих к $ latex \ boldsymbol {R_2} $ и $ latex \ boldsymbol {R_3} $.(а) Найдите полное сопротивление. (б) Что такое падение $ latex \ boldsymbol {IR} $ в $ latex \ boldsymbol {R_1} $? (c) Найдите текущий $ latex \ boldsymbol {I_2} $ через $ latex \ boldsymbol {R_2} $. (d) Какую мощность рассеивает $ latex \ boldsymbol {R_2} $?

Рисунок 5. Эти три резистора подключены к источнику напряжения, так что R 2 и R 3 параллельны друг другу, и эта комбинация последовательно с R 1 .

Стратегия и решение для (а)

Чтобы найти полное сопротивление, отметим, что $ latex \ boldsymbol {R_2} $ и $ latex \ boldsymbol {R_3} $ находятся параллельно, а их комбинация $ latex \ boldsymbol {R_p} $ находится последовательно с $ latex \ boldsymbol { R_1} $. Таким образом, полное (эквивалентное) сопротивление этой комбинации составляет

.

$ латекс \ boldsymbol {R _ {\ textbf {tot}} = R_1 + R_p}. $

Сначала мы находим $ latex \ boldsymbol {R_p} $, используя уравнение для параллельных резисторов и вводя известные значения:

Инвертирование дает

$ latex \ boldsymbol {R_p =} $ latex \ boldsymbol {\ Omega = 4.11 \; \ Omega}. $

Таким образом, общее сопротивление равно

.

$ латекс \ boldsymbol {R _ {\ textbf {tot}} = R_1 + R_p = 1.00 \; \ Omega + 4.11 \; \ Omega = 5.11 \; \ Omega}. $

Обсуждение для (а)

Общее сопротивление этой комбинации является промежуточным между значениями чистой серии и чисто параллельной ($ latex \ boldsymbol {20.0 \; \ Omega} $ и $ latex \ boldsymbol {0.804 \; \ Omega} $, соответственно), найденными для тех же резисторы в двух предыдущих примерах.

Стратегия и решение для (b)

Чтобы найти падение $ latex \ boldsymbol {IR} $ в $ latex \ boldsymbol {R_1} $, отметим, что полный текущий $ latex \ boldsymbol {I} $ течет через $ latex \ boldsymbol {R_1} $.Таким образом, его падение $ latex \ boldsymbol {IR} $ равно

.

$ латекс \ boldsymbol {V_1 = IR_1}. $

Мы должны найти $ latex \ boldsymbol {I} $, прежде чем сможем вычислить $ latex \ boldsymbol {V_1} $. Полный ток $ latex \ boldsymbol {I} $ находится с помощью закона Ома для схемы. То есть

$ latex \ boldsymbol {I =} $ latex \ boldsymbol {=} $ latex \ boldsymbol {= 2.35 \; \ textbf {A}}. $

Вводя это в выражение выше, мы получаем

$ латекс \ boldsymbol {V_1 = IR_1 = (2.35 \; \ textbf {A}) (1.00 \; \ Omega) = 2.35 \; \ textbf {V}}. $

Обсуждение для (б)

Напряжение, приложенное к $ latex \ boldsymbol {R_2} $ и $ latex \ boldsymbol {R_3} $, меньше общего напряжения на величину $ latex \ boldsymbol {V_1} $. Большое сопротивление провода может существенно повлиять на работу устройств, представленных $ latex \ boldsymbol {R_2} $ и $ latex \ boldsymbol {R_3} $.

Стратегия и решение для (c)

Чтобы найти ток через $ latex \ boldsymbol {R_2} $, мы должны сначала найти приложенное к нему напряжение.Мы называем это напряжение $ latex \ boldsymbol {V _ {\ textbf {p}}} $, потому что оно применяется к параллельной комбинации резисторов. Напряжение, приложенное как к $ latex \ boldsymbol {R_2} $, так и к $ latex \ boldsymbol {R_3} $, уменьшается на сумму $ latex \ boldsymbol {V_1} $, и поэтому оно составляет

.

$ латекс \ boldsymbol {V_p = V — V_1 = 12.0 \; \ textbf {V} — 2.35 \; \ textbf {V} = 9.65 \; \ textbf {V}}. $

Теперь текущий $ latex \ boldsymbol {I_2} $ через сопротивление $ latex \ boldsymbol {R_2} $ находится по закону Ома:

$ latex \ boldsymbol {I_2 =} $ latex \ boldsymbol {=} $ $ latex \ boldsymbol {= 1.2 (6.00 \; \ Omega) = 15.5 \; \ textbf {W}}. $

Обсуждение для (d)

Мощность меньше 24,0 Вт, рассеиваемых этим резистором при параллельном подключении к источнику 12,0 В.

Одним из следствий этого последнего примера является то, что сопротивление в проводах снижает ток и мощность, подаваемую на резистор. Если сопротивление провода относительно велико, как в изношенном (или очень длинном) удлинителе, то эти потери могут быть значительными. Если протекает большой ток, падение $ latex \ boldsymbol {IR} $ в проводах также может быть значительным.

Например, когда вы роетесь в холодильнике и включается мотор, свет холодильника на мгновение гаснет. Точно так же вы можете увидеть тусклый свет в салоне, когда вы запускаете двигатель вашего автомобиля (хотя это может быть связано с сопротивлением внутри самой батареи).

Что происходит в этих сильноточных ситуациях, показано на рисунке 6. Устройство, обозначенное $ latex \ boldsymbol {R_3} $, имеет очень низкое сопротивление, поэтому при его включении протекает большой ток.Этот повышенный ток вызывает большее падение $ latex \ boldsymbol {IR} $ в проводах, представленных $ latex \ boldsymbol {R_1} $, что снижает напряжение на лампочке (которое составляет $ latex \ boldsymbol {R_2} $), что потом заметно тускнеет.

Рис. 6. Почему свет тускнеет, когда включен большой прибор? Ответ заключается в том, что большой ток, потребляемый двигателем прибора, вызывает значительное падение IR в проводах и снижает напряжение на свету.

Проверьте свое понимание

1: Можно ли любую произвольную комбинацию резисторов разбить на последовательную и параллельную комбинации? Посмотрите, сможете ли вы нарисовать принципиальную схему резисторов, которые нельзя разбить на комбинации последовательно и параллельно.

Стратегии решения проблем для последовательных и параллельных резисторов

  1. Нарисуйте четкую принципиальную схему, пометив все резисторы и источники напряжения. Этот шаг включает список известных проблем, поскольку они отмечены на вашей принципиальной схеме.
  2. Определите, что именно необходимо определить в проблеме (определите неизвестные). Письменный список полезен.
  3. Определите, включены ли резисторы последовательно, параллельно или в комбинации последовательно и параллельно.Изучите принципиальную схему, чтобы сделать эту оценку. Резисторы включены последовательно, если через них должен последовательно проходить один и тот же ток.
  4. Используйте соответствующий список основных функций для последовательных или параллельных подключений, чтобы найти неизвестные. Есть один список для серий, а другой — для параллелей. Если ваша проблема представляет собой комбинацию последовательного и параллельного соединения, уменьшайте ее поэтапно, рассматривая отдельные группы последовательных или параллельных соединений, как это сделано в этом модуле и примерах. Особое примечание: при нахождении $ latex \ boldsymbol {R_p} $ необходимо с осторожностью относиться к обратному значению.
  5. Проверьте, являются ли ответы разумными и последовательными. Единицы и числовые результаты должны быть разумными. Общее последовательное сопротивление должно быть больше, а общее параллельное сопротивление, например, должно быть меньше. Мощность должна быть больше для одних и тех же устройств, подключенных параллельно, по сравнению с последовательными и так далее.
  • Полное сопротивление электрической цепи с последовательно включенными резисторами является суммой отдельных сопротивлений: $ latex \ boldsymbol {R_s = R_1 + R_2 + R_3 + \ cdots} $
  • Каждый резистор в последовательной цепи имеет одинаковое количество тока, протекающего через него.
  • Падение напряжения или рассеиваемая мощность на каждом отдельном резисторе в серии различается, и их общая сумма складывается с входом источника питания.
  • Общее сопротивление электрической цепи с резисторами, подключенными параллельно, меньше наименьшего сопротивления любого из компонентов и может быть определено по формуле: $ latex \ boldsymbol {=} $ $ latex \ boldsymbol {+} $ $ latex \ boldsymbol {+} $ $ latex \ boldsymbol {+ \; \ cdots} $
  • Каждый резистор в параллельной цепи имеет одинаковое полное напряжение источника.
  • Ток, протекающий через каждый резистор в параллельной цепи, различается в зависимости от сопротивления.
  • Если более сложное соединение резисторов представляет собой комбинацию последовательного и параллельного соединения, оно может быть уменьшено до одного эквивалентного сопротивления, идентифицируя его различные части как последовательные или параллельные, уменьшая каждую до своего эквивалента и продолжая до тех пор, пока в конечном итоге не будет достигнуто единое сопротивление. .

Концептуальные вопросы

1: Переключатель имеет переменное сопротивление, близкое к нулю в замкнутом состоянии и очень большое в разомкнутом, и он включен последовательно с устройством, которым он управляет.Объясните влияние переключателя на рис. 7 на ток в разомкнутом и замкнутом состоянии.

Рис. 7. Переключатель обычно включается последовательно с источником сопротивления и напряжения. В идеале переключатель имеет почти нулевое сопротивление в замкнутом состоянии, но имеет чрезвычайно большое сопротивление в разомкнутом состоянии. (Обратите внимание, что на этой диаграмме скрипт E представляет напряжение (или электродвижущую силу) батареи.)

2: Какое напряжение на разомкнутом переключателе на рисунке 7?

3: На разомкнутом переключателе есть напряжение, как на Рисунке 7.Почему же тогда мощность, рассеиваемая разомкнутым переключателем, мала?

4: Почему мощность, рассеиваемая замкнутым переключателем, как на рисунке 7, мала?

5: Студент в физической лаборатории по ошибке подключил электрическую лампочку, батарею и выключатель, как показано на рис. 8. Объясните, почему лампочка горит, когда выключатель разомкнут, и гаснет, когда он замкнут. (Не пытайтесь — батарея сильно разряжается!)

Рис. 8. Ошибка подключения: этот переключатель подключен параллельно устройству, представленному R .(Обратите внимание, что на этой диаграмме скрипт E представляет напряжение (или электродвижущую силу) батареи.)

6: Зная, что сила удара зависит от величины тока, проходящего через ваше тело, вы бы предпочли быть последовательно или параллельно с сопротивлением, таким как нагревательный элемент тостера, если он поражен током? Объяснять.

7: Были бы ваши фары тусклыми при запуске двигателя автомобиля, если бы провода в вашем автомобиле были сверхпроводниками? (Не пренебрегайте внутренним сопротивлением батареи.) Объяснять.

8: Некоторые гирлянды праздничных огней подключаются последовательно для экономии затрат на проводку. В старой версии использовались лампочки, которые при перегорании прерывали электрическое соединение, как открытый выключатель. Если одна такая лампочка перегорит, что случится с остальными? Если такая цепочка работает от 120 В и имеет 40 одинаковых лампочек, каково нормальное рабочее напряжение каждой? В более новых версиях используются лампы, которые при перегорании замыкаются накоротко, как замкнутый выключатель. Если одна такая лампочка перегорит, что случится с остальными? Если такая цепочка работает от 120 В и в ней осталось 39 идентичных лампочек, каково тогда рабочее напряжение каждой?

9: Если две бытовые лампочки мощностью 60 Вт и 100 Вт подключить последовательно к бытовой электросети, какая из них будет ярче? Объяснять.

10: Предположим, вы выполняете физическую лабораторию, которая просит вас вставить резистор в цепь, но все прилагаемые резисторы имеют большее сопротивление, чем запрошенное значение. Как бы вы соединили доступные сопротивления, чтобы попытаться получить меньшее запрошенное значение?

11: Перед Второй мировой войной некоторые радиостанции получали питание через «шнур сопротивления», который имел значительное сопротивление. Такой резистивный шнур снижает напряжение до желаемого уровня для ламп радиоприемника и т.п., и это экономит расходы на трансформатор.Объясните, почему шнуры сопротивления нагреваются и тратят энергию при включенном радио.

12: У некоторых лампочек есть три уровня мощности (не включая ноль), получаемые от нескольких нитей накала, которые индивидуально переключаются и подключаются параллельно. Какое минимальное количество нитей необходимо для трех режимов мощности?

Проблемные упражнения

Примечание. Можно считать, что данные, взятые из цифр, имеют точность до трех значащих цифр.

1: (a) Каково сопротивление десяти последовательно соединенных резисторов $ latex \ boldsymbol {275 — \; \ Omega} $? (б) Параллельно?

2: (a) Какое сопротивление $ латекс \ boldsymbol {1.2 — \; \ Omega} $ подключены $ latex \ boldsymbol {2.50 — \; \ textbf {k} \ Omega} $ и резистор $ latex \ boldsymbol {4.00 — \; \ textbf {k} \ Omega} $ последовательно? (б) Параллельно?

3: Какие наибольшее и наименьшее сопротивления вы можете получить, подключив $ latex \ boldsymbol {36.0 — \; \ Omega} $, $ latex \ boldsymbol {50.0 — \; \ Omega} $ и $ латексный \ boldsymbol {700 — \; \ Omega} $ резистор вместе?

4: Тостер на 1800 Вт, электрическая сковорода на 1400 Вт и лампа на 75 Вт подключены к одной розетке в цепи 15 А, 120 В.(Три устройства работают параллельно, если они подключены к одной розетке.) а) Какой ток потребляет каждое устройство? (b) Перегорит ли эта комбинация предохранитель на 15 А?

5: Фара на 30,0 Вт и стартер на 2,40 кВт обычно подключаются параллельно в систему на 12,0 В. Какую мощность потребляли бы одна фара и стартер при последовательном подключении к батарее 12,0 В? (Не обращайте внимания на любое другое сопротивление в цепи и любое изменение сопротивления в двух устройствах.)

6: (a) Учитывая 48.Аккумулятор 0 В и резисторы $ latex \ boldsymbol {24.0 — \; \ Omega} $ и $ latex \ boldsymbol {96.0 — \; \ Omega} $, найдите для каждого из них ток и мощность при последовательном соединении. (b) Повторите, когда сопротивления включены параллельно.

7: Ссылаясь на пример, объединяющий последовательные и параллельные схемы и рисунок 5, вычислите $ latex \ boldsymbol {I_3} $ двумя следующими способами: (a) по известным значениям $ latex \ boldsymbol {I} $ и $ latex \ boldsymbol {I_2} $; (б) используя закон Ома для $ latex \ boldsymbol {R_3} $.В обеих частях явно показано, как вы выполняете шаги, указанные в Стратегиях решения проблем для последовательных и параллельных резисторов.

8: Ссылаясь на рисунок 5: (a) Вычислите $ latex \ boldsymbol {P_3} $ и обратите внимание на его сравнение с $ latex \ boldsymbol {P_3} $, найденным в первых двух примерах задач в этом модуле. (b) Найдите полную мощность, отдаваемую источником, и сравните ее с суммой мощностей, рассеиваемых резисторами.

9: См. Рисунок 6, где обсуждается затемнение света при включении тяжелого прибора.9 — \; \ Omega} $ сопротивление. Рис. 9. (а) Какое сопротивление относительно земли у 100 изоляторов? (b) Рассчитайте мощность, рассеиваемую 100 из них. (c) Какая доля мощности, переносимой линией, составляет это? Ясно покажите, как вы выполняете шаги, указанные в Стратегиях решения проблем для последовательных и параллельных резисторов.

Рис. 9. Линия электропередачи высокого напряжения (240 кВ), несущая 5,00 × 10 2 подвешен к заземленной металлической опоре электропередачи. Ряд керамических изоляторов обеспечивает л.00 × 10 9 Ом по сопротивления каждый.

11: Покажите, что если два резистора $ latex \ boldsymbol {R_1} $ и $ latex \ boldsymbol {R_2} $ объединены, и один из них намного больше другого ($ latex \ boldsymbol {R_1 >> R_2} $) : (a) Их последовательное сопротивление почти равно большему сопротивлению $ latex \ boldsymbol {R_1} $. (б) Их параллельное сопротивление почти равно меньшему сопротивлению $ latex \ boldsymbol {R_2} $.

12: Необоснованные результаты

Два резистора, один из которых имеет сопротивление $ latex \ boldsymbol {145 \; \ Omega} $, подключены параллельно, чтобы получить общее сопротивление $ latex \ boldsymbol {150 \; \ Omega} $.а) Каково значение второго сопротивления? б) Что неразумного в этом результате? (c) Какие предположения необоснованны или непоследовательны?

13: необоснованные результаты

Два резистора, один из которых имеет сопротивление $ latex \ boldsymbol {900 \; \ textbf {k} \ Omega} $, соединены последовательно, чтобы получить общее сопротивление $ latex \ boldsymbol {0.500 \; \ textbf {M}. \ Omega} $. а) Каково значение второго сопротивления? б) Что неразумного в этом результате? (c) Какие предположения необоснованны или непоследовательны?

Глоссарий

серии
последовательность резисторов или других компонентов, включенных в цепь один за другим
резистор
компонент, обеспечивающий сопротивление току, протекающему через электрическую цепь
сопротивление
вызывает потерю электроэнергии в цепи
Закон Ома
взаимосвязь между током, напряжением и сопротивлением в электрической цепи: $ latex \ boldsymbol {V = IR} $
напряжение
электрическая потенциальная энергия на единицу заряда; электрическое давление, создаваемое источником питания, например аккумулятором
падение напряжения
потеря электроэнергии при прохождении тока через резистор, провод или другой компонент
текущий
поток заряда через электрическую цепь мимо заданной точки измерения
Закон Джоуля
взаимосвязь между потенциальной электрической мощностью, напряжением и сопротивлением в электрической цепи, определяемая по формуле: $ latex \ boldsymbol {P_e = IV} $
параллельно
разводку резисторов или других компонентов в электрической цепи, так что каждый компонент получает одинаковое напряжение от источника питания; часто изображается на диаграмме в виде лестницы, где каждый компонент находится на ступеньке лестницы

Решения

Проверьте свое понимание

1: Нет, есть много способов подключения резисторов, которые не являются комбинациями последовательного и параллельного, включая петли и переходы.В таких случаях правила Кирхгофа, которые будут представлены в главе 21.3 «Правила Кирхгофа», позволят вам проанализировать схему.

Задачи Упражнения

1: (a) $ latex \ boldsymbol {2.75 \; \ textbf {k} \ Omega} $

(б) $ latex \ boldsymbol {27.5 \; \ Omega} $

3: (a) $ latex \ boldsymbol {786 \; \ Omega} $

(б) $ latex \ boldsymbol {20.3 \; \ Omega} $

5: 29,6 Вт

7: (а) 0.74 А

(б) 0,742 А

9: (а) 60,8 Вт

(б) 3,18 кВт

11: (a) $ latex \ begin {array} {c} \ boldsymbol {R_s = R_1 + R_2} \\ [1em] \ boldsymbol {\ Rightarrow R_s \ приблизительно R_1 (R_1 >> R_2)} \ end {array}

долл. США

(b) $ latex \ boldsymbol {=} $ $ latex \ boldsymbol {+} $ $ latex \ boldsymbol {=} $

, так что

$ latex \ boldsymbol {R_p =} $ $ latex \ boldsymbol {\ приблизительно} $ $ latex \ boldsymbol {= R_2 \; (R_1 >> R_2)}.$

13: (a) $ latex \ boldsymbol {-400 \; \ textbf {k} \ Omega} $

(b) Сопротивление не может быть отрицательным.

(c) Считается, что последовательное сопротивление меньше, чем у одного из резисторов, но должно быть больше, чем у любого из резисторов.

Серия

и параллельные резисторы

  • Изучив этот раздел, вы сможете:
  • Рассчитайте значения общего сопротивления в цепях с последовательным сопротивлением.
  • Используйте соответствующие формулы для расчета сопротивления в цепях с параллельным сопротивлением.
  • • Вычисление суммы обратных величин.
  • • Произведение над суммой.
  • Рассчитайте значения общего сопротивления в последовательных / параллельных сетях.

Расчеты в последовательно- и параллельных резисторных цепях

Компоненты, включая резисторы в цепи, могут быть соединены вместе двумя способами:

ПОСЛЕДОВАТЕЛЬНО, так что один и тот же ток течет через все компоненты, но на каждом из них может существовать разная разность потенциалов (напряжение).

ПАРАЛЛЕЛЬНО, так что одинаковая разность потенциалов (напряжение) существует на всех компонентах, но каждый компонент может проводить разный ток.

Рис. 4.2.1 Резисторы серии

Рис. 4.2.2 Параллельные резисторы

В любом случае (для резисторов) полное сопротивление той части цепи, которая содержит резисторы, может быть рассчитано с использованием методов, описанных ниже.

Возможность рассчитать суммарное (общее) значение резисторов таким способом позволяет легко вычислить неизвестные значения сопротивления, тока и напряжения для довольно сложных схем с использованием относительно простых методов.Это очень полезно при поиске неисправностей.

ПЕРЕД ДАЛЬНЕЙШЕЙ ДАЛЬНОСТЬЮ ПОПРОБУЙТЕ ИСПОЛЬЗОВАНИЕ ФОРМУЛ ДЛЯ РАСЧЕТА ОБЩИХ ЗНАЧЕНИЙ СЕРИЙНЫХ И ПАРАЛЛЕЛЬНЫХ РЕЗИСТОРОВ.

Для резисторов в серии:

Общее сопротивление двух или более резисторов, подключенных последовательно , определяется простым сложением индивидуальных значений резисторов, чтобы найти общую сумму (R TOT ):

Для резисторов, включенных параллельно:

Для расчета общего сопротивления цепи, в которой используются параллельные резисторы, можно использовать следующую формулу.

Обратите внимание, однако, что эта формула НЕ дает вам общего сопротивления R TOT . Это дает вам ВЗАИМОДЕЙСТВИЕ R TOT или:

Это совсем другое значение — и НЕ является полным сопротивлением. Он делится на 1, деленный на TOT . Чтобы получить правильное значение для R TOT (которое будет обратным 1 / R TOT , т. Е. TOT /1, просто нажмите соответствующую клавишу на вашем калькуляторе (отмеченную 1 / x или x-1) .

Другой способ расчета параллельных цепей.

Общее сопротивление двух резисторов, включенных параллельно , которое не включает обратных величин, определяется по формуле:

Эту формулу часто называют «произведение над суммой».

Рассчитывает только ДВА резистора параллельно? Ну да, но это не большая проблема. Если имеется более двух параллельных резисторов, просто выберите два из них и определите общее сопротивление для этих двух — затем используйте это общее сопротивление, как если бы это был один резистор, и составьте еще одну пару с третьим резистором.Определите новую сумму и так далее, пока вы не включите все параллельные резисторы в этой конкретной сети.

О, еще кое-что, что нужно помнить о произведении над суммой, видите скобки вокруг суммы (нижняя часть) формулы? Это означает, что вы должны решить это, прежде чем использовать его для разделения продукта (верхняя часть) на. Если вы этого не сделаете, ваш ответ будет неправильным.

Звучит сложно? Не совсем, это просто вопрос повторения, и на практике вы не часто встречаетесь с множеством параллельных сетей с гораздо более чем двумя резисторами.Тем не менее, какую формулу вы выберете, зависит от вас, взаимная или сумма продукта.

Подсказки

Использование обратного метода

Если вы используете МЕТОД ВЗАИМОДЕЙСТВИЯ для параллельных цепей, НЕ ЗАБУДЬТЕ, когда вы добавили обратные величины отдельных резисторов — вы должны снова найти обратную величину. 1 / R1 + 1 / R2 + 1 / R3 = 1 / R TOT и чтобы найти R TOT , вы должны найти обратное 1 / R TOT .

Упрощающие схемы

Для комбинированных последовательных и параллельных цепей сначала определите участок цепи (последовательный или параллельный).Затем перерисуйте схему, заменив участок, сопротивление которого вы нашли, одним резистором. Теперь у вас есть упрощенная схема, в которой можно найти R TOT .

Вы можете использовать формулу «произведение на сумму»:

Для цепей с более чем двумя параллельными резисторами просто определите два параллельных резистора одновременно, используя формулу произведения на сумму, а затем перерисуйте схему, заменив два резистора одним резистором, значение которого является объединенным сопротивлением двух .

Теперь вы можете использовать первое комбинированное значение в качестве единственного резистора со следующим параллельным резистором и так далее. Таким образом, можно выработать большое количество параллельных резисторов с использованием произведения на сумму.

Когда все параллельные резисторы одинакового номинала.

Если подключено несколько одинаковых параллельных резисторов, общее сопротивление будет равно номиналу резистора, умноженному на обратную величину количества резисторов.

, т. Е. Два параллельных резистора 12 кОм имеют общее сопротивление

.

12K x 1/2 = 6K

Три параллельно включенных резистора 12 кОм имеют суммарное сопротивление

.

12K x 1/3 = 4K и т. Д.

Проверяю ваш ответ

Суммарное значение любого количества параллельных резисторов всегда будет МЕНЬШЕ, чем значение наименьшего отдельного резистора в сети. Используйте этот факт, чтобы проверить свои ответы.

Серия

и параллельная комбинация

Попробуйте несколько вычислений на основе последовательной и параллельной цепей резисторов. Для этого вам просто нужно использовать информацию на этой странице и на странице «Советы по расчету резисторов». Вас просят вычислить общее сопротивление для каждой цепи.Вы можете выбрать, какую формулу использовать

Вы также можете получить помощь по математике, загрузив нашу бесплатную брошюру «Советы по математике».

Прежде чем начать, подумайте об этих нескольких советах. Они упростят задачу, если вы будете внимательно им следовать.

1. Разработайте ответы с помощью карандаша и бумаги; перерисуйте схему, над которой работаете.

2. Конечно, ответ — это не просто число, это будет определенное количество Ом, не забудьте указать правильную единицу (например.грамм. Ω, KΩ или MΩ) или ваш ответ не имеет смысла.

3. Когда вы вводите значения в калькулятор, преобразуйте все значения KΩ или MΩ в Ом с помощью клавиши EXP. Если вы здесь ошибетесь, то получите действительно глупые ответы, в тысячи раз слишком большие или слишком маленькие.

Итак, вы прочитали эти инструкции, и вы готовы к работе. Вот способ решить типичную проблему на бумаге, чтобы (со временем) вы не запутались.

Пример последовательной и параллельной цепи.

Хорошо, есть что вспомнить, так почему бы не попробовать несколько практических вопросов в Resistors Module 4.5 по определению полного сопротивления некоторых цепей резисторов?

Решено: ИССЛЕДОВАНИЕ 2: ТОК И НАПРЯЖЕНИЕ В ПАРАЛЛЕЛЬНЫХ ЦЕПЯХ …

  1. наука
  2. физика
  3. вопросы и ответы по физике
  4. ИССЛЕДОВАНИЕ 2: ТОК И НАПРЯЖЕНИЕ В ПАРАЛЛЕЛЬНЫХ ЦЕПЯХ Есть два основных способа подключения…

Показать текст изображения

Ответ эксперта

100% (1 оценка) Sol: Q2-1: D и E параллельны, как видно из конфигурации, один вывод D подключен к одному выводу E и то же самое верно и для других клемм, и эти комбинированные клеммы относятся к полному ответу Предыдущий вопрос Следующий вопрос

ИССЛЕДОВАНИЕ 2: ТОК И НАПРЯЖЕНИЕ В ПАРАЛЛЕЛЬНЫХ ЦЕПИ Существует два основных способа подключения резисторов, лампочек или других элементов в цепи — последовательно и параллельно.До сих пор вы подключали лампочки последовательно. Чтобы делать прогнозы, связанные с более сложными цепями, нам нужно более точное определение последовательного и параллельного. Они кратко изложены во вставке ниже. Последовательное соединение: совместное использование одного перехода Два резистора или лампочки включены последовательно, если они соединены так, что приходящий ток, проходящий через один резистор или лампочку, проходит через другой. То есть, существует только один путь для тока, последовательное соединение 1 Параллельное соединение: два общих соединения Два резистора или лампочки подключены параллельно, если их клеммы соединены вместе так, что каждый конец резистора или лампочки подключен напрямую. к одному концу другого резистора или лампы, e.Gujunction 1 на схеме. Аналогичным образом другие концы соединяются вместе (стык 2). соединение 2. Параллельно Давайте сравним поведение цепи с двумя лампочками, включенными параллельно, с цепью с одной лампочкой. (См. Рисунок 2-5 ниже.) A 3) (b) (a) Рисунок 2-5: Две разные цепи с идентичными компонентами — (a) цепь с одной лампочкой и (b) цепь с двумя одинаковыми лампочками, подключенными параллельно друг к другу и параллельно аккумулятору. Обратите внимание, что если лампы A, D и E идентичны, то схема на Рисунке 2-6 эквивалентна схеме 2-5 (a), когда переключатель разомкнут (как показано), и эквивалентна схеме 2-5 (b), когда выключатель замкнут.Физика в реальном времени: Лаборатория активного обучения + D3 E Рис. 2-6: Когда переключатель разомкнут, лампочка D подключается параллельно к батарее. Когда переключатель замкнут, лампочки D и E подключаются параллельно друг другу и параллельно батарее. Вопрос 2-1: Объясните, как определить из конфигурации схемы, что D&E включены параллельно, когда переключатель замкнут. Прогноз 2-1: При замкнутом переключателе спрогнозируйте, как яркость лампочек D и E в параллельной цепи на рисунке 2-5 (b) будет сравниваться с яркостью лампы A в цепи с одной лампой на рисунке 2-5 (a).Как D и E будут сравниваться друг с другом? Оцените яркость всех трех лампочек. Объясните причины своих прогнозов. Упражнение 2-1: Яркость лампочек в параллельной цепи Создайте схему на рис. 2-6 и опишите ваши наблюдаемые рейтинги яркости лампочки D с разомкнутым переключателем и D и E с замкнутым переключателем. Вопрос 2-2: совпадают ли полученные рейтинги с вашим прогнозом? Если нет, можете ли вы объяснить, какие предположения, которые вы делали, кажутся ложными? Предсказание 2-2: На основании ваших наблюдений за яркостью, что вы предсказываете относительно относительного количества тока, протекающего через каждую лампочку при параллельном подключении, т.е.е., лампочки D и E на Рисунке 2-5 (b)? Прогноз 2-3: Основываясь на ваших наблюдениях за яркостью, как вы думаете, включение переключателя на рис. 2-6 влияет на ток через лампочку D?

Цепи серии

Представьте себе электрический ток, выходящий из батареи. Если резисторы подключены в такой способ, которым часть тока может проходить через один резистор, а остальная часть ток может пройти через другой резистор, тогда цепь будет параллельной схема .

I T — полный ток параллельной цепи. Вы бы измерили этот ток в любом месте до или после трехканального разветвителя, ведущего к трем резисторам. В между переходом и R 1 , вы бы измерили I 1 . Между перекрестком и R 2 , Вы бы измерили I 2 и т. д.

Поскольку общий ток, I T , делится на три разные группы электронов, путешествующих каждый своим путем,

I T = I 1 + I 2 + I 3 +.

В параллельных цепях все резисторы, независимо от их сопротивления, испытывают одинаковое падение напряжения или разность потенциалов, потому что все они имеют одинаковые точки входа и выхода (переходы).

V T = V 1 = V 2 = V 3 = V n

Если разделить формулу тока по соотношению напряжений получаем:

или R T = [R 1 -1 + R 2 -1 + R 3 -1 +] -1

Пример 1

а. Какое полное сопротивление цепи?

R T = [R 1 -1 + R 2 -1 + R 3 -1 ] -1

R T = [12 -1 + 12 -1 +12 -1 ] -1 = 4 Вт

г. Какой общий ток?

I T = V / R T = 12/4 = 3 A

г.Какое напряжение ( В 1 ) будет измеряется на каждом отдельном резисторе?

12 В (напряжение постоянно параллельно.)

г. Какой ток отводится каждый резистор?

I 1 = V / R 1 = 12/12 = 1 А. Остальные тоже нарисуйте по 1 А, всего 3 А.

Пример 2

рисунок кажется запутанным, но обратите внимание, что это параллельная схема, потому что у электронов есть выбор.На стыке (показано красной точкой) электроны следуйте либо зеленому маршруту, либо оранжевому маршруту.

Используйте I

1 = 1A; I 2 = 0,5 А; R 1 = 10Вт.

  1. Найдите V 2 .

Помните, что параллельное напряжение постоянно. Итак, если мы найдем V 1 , мы узнаем V 2 .

В 1 = I 1 R 1 = 1 (10) = 10 В.

В 2 = В 1 = 10 В.

  1. Найдите R 2 .

R 2 = V / I 2 = 10 / 0,5 = 20 Вт.

  1. Используйте два метода, чтобы получить R T .

(1) R T = [R 1 -1 + R 2 -1 ] -1 = [20 -1 + 10 -1 ] -1 = 6.7 W.

(2) I T = I 3 = I 1 + I 2 = 1 + 0,5 = 1,5 A.

R T = V / I T = 10 / 1,5 = 6,7 W.

Пример 3

В параллельной цепи какой эффект дает добавление дополнительных резисторов иметь по общему току?

Ток увеличивается !

В цепи серии добавление резисторов увеличивает общее сопротивление и, таким образом, снижает ток.Но в случае с параллельная схема, потому что добавление дополнительных резисторов параллельно создает больше вариантов а снижает общее сопротивление . Если такая же батарея подключена к резисторы, ток увеличится. Не убежден? Попробуйте:

[10 -1 + 10 -1 ] -1 = 5 Вт, но добавьте подключите резистор параллельно, и вы получите [10 -1 + 10 -1 + 10 -1 ] -1 = всего 3,3 Вт. чем меньше сопротивление, тем выше общий ток.

Еще одна интересная особенность параллельных цепей заключается в том, что если один компонент отключен, другие пути все еще жизнеспособны, так что электроны могут продолжать течь по цепи.

alexxlab

Добавить комментарий

Ваш адрес email не будет опубликован. Обязательные поля помечены *